Barkley Adult ALL, FNP-Barkley Ped Questions

¡Supera tus tareas y exámenes ahora con Quizwiz!

When diagnosing depression in an adolescent, which of the following is the most important question to ask? A. "Have you ever thought about or attempted to hurt yourself?" B. "Have you ever been treated for depression?" C. "Have you ever experienced delusions or hallucinations?" D. "Do you regularly take prescription medication for depression?"

A. "Have you ever thought about or attempted to hurt yourself?"

A premature infant should typically be given palivizumab (Synagis) for the prevention of RSV bronchiolitis if gestation is less than: A. 29 weeks B. 33 weeks C. 35 weeks D. 40 weeks

A. 29 weeks

According to the psychosocial development model, at which age would an infant be least likely to cry when she is handed to her new babysitter? A. 4 months B. 6 months C. 8 months D. 10 months

A. 4 months

Anthony, age 62, presents to your practice with ptosis dysphagia, extremity weakness. Which of the following lab results would be most helpful in establishing a diagnosis based on your suspicion of myasthenia gravis?

Acetylcholine receptor antibodies in serum

Your patient asks, "I really thought I was having a heart attack. How can you tell the difference between angina and having a myocardial infarction?" Which of the following responses would provide the patient the most accurate information?

"Pain of angina can usually be relieved by resting or lying down."

An Adolescent has experienced a rupture in the growth plate at the tibial tuberosity as a result of stress on the resulting condition. Which of these statements would you most likely make?

"The pain will often be worsened by running, jumping, and climbing stairs."

What percentage of the patient's bill for physician services is typically covered by medicare

80%

What is the male to female transmission rate of gonorrhea on initial exposure?

80-90%

Winston, a 42 year old male, is an HIV-positive patient whose tuberculosis skin test returns with an elevation of 5 mm. After confirming a diagnosis of TB, you prescribe a traditional drug regimen. For what minimum period of time is Winston expected to continue his regiment?

9 months

Which of these values is within the normal range for mean corpuscular volume? A. 77 fL B. 92fL C. 101 fL D. 113 fL

92fL

Jerome, an 11 year old, wakes up every morning @ 6 AM. At what time should he go to bed in order to get the minimum recommended amount of sleep for his age? A. 8 PM B. 9 PM C. 10 PM D. 11 PM

C. 10 PM (School age children should get 8 hours of sleep w/max of 11 hours.)

Which of the following values most likely indicates the normal visual acuity of a 5 year old? A. 20/80 B. 20/50 C. 20/30 D. 20/20

C. 20/30 (3 — 20/50; 4 — 20/40; 5 — 20/30)

You note that an infant waves "bye-bye" when leaving the clinic. This is an unexpected finding by at least what age? A. 2 to 4 months B. 4 to 6 months C. 8 to 9 months D. One year

C. 8 to 9 months (p. 10)

You are consulting a mother who is worried about her 8 mom old male infant. She states that he has a fever, & wants to give him ibuprofen. You recommend giving him 90 mg of ibuprofen q6h. Based on the above repayment guidelines, what is the most likely weight of this infant? A. 7.5 kg B. 8.18 kg C. 9 kg D. 10 kg

C. 9 kg

You are consulting a mother who is worried about her 8-month-old male infant. She states that he has a fever, & she wants to give him ibuprofen. You recommend giving him 90 mg of ibuprofen every 6 hours. Based on the above treatment guidelines, what is the most likely weight of this infant? A. 7.5 kg B. 8.18 kg C. 9 kg D. 10 kg

C. 9 kg (The suggested ibuprofen regimen for managing fever in infants > 6 months is 10 mg/kg every 6 hours; thus, a 90 mg dose of ibuprofen would be recommended for an infant weighing 9 kg. The dosing regimens for acetaminophen is 10-15 mg/kg every 4 hours.)

Which of the following BP percentiles on the "BP levels for boys & girls by age & height percentile table" published by the National Institutes for Health serves as the hypertension threshold in pediatric patients? A. 50th B. 90th C. 95th D. 99th

C. 95th (p. 68)

In general, how frequently would a typical pediatric pt w/mild persistent asthma use a rescue inhaler? A. < 2 days/week B. Several times per day C. > 2 days/week D. Once per day

C. > 2 days/week

At least how many months apart should hepatitis A doses of vaccines be administered? A. @ least 1 month apart B. @ least 3 months apart C. @ least 6 months apart D. @ least 12 months apart

C. @ least 6 months apart (The 2 doses should be given between the ages of 1 & 2 & they should be 6 months apart. The 1st dose protects the body against hepatitis A for 1 year & allows the body to make antibodies against the virus. The 2nd dose acts to protect against hepatitis A for 20 years.)

For which of the following patients is a lumbar puncture most indicated? A. A patient w/a known seizure disorder B. A patient w/streptococcal pneumonia C. A feverish patient w/seizures D. A nauseous patient w/an erythematous tympanic membrane

C. A feverish patient w/seizures (p. 130)

Of the following patients, which does not need to be reported to state or local authorities? A. A low risk patient whose tuberculin test results is < 15mm B. Accident gunshot wound C. A patient whose serology results are anti-HAV & IgG D. Non-rabid dog bite

C. A patient whose serology results are anti-HAV & IgG

A mother brings her 18 mo old daughter to your practice w/complaints of watery diarrhea. The mother says that she has noticed her child "has not looked too well". She also says that her daughter has been refusing to eat & has been unable to keep her food down. The mother also reports hearing loud rumbling noises coming from the child's stomach. Based on these findings, which finding would you most expect to see in this patient during your exam? A. Enlarged abdominal mass B. Visible peristaltic waves C. Abdominal distention D. Positive psoas sign

C. Abdominal distention (Abdominal distention is an expected finding of AGE, which often produces watery diarrhea, hyperactive bowel sounds, non-bilious vomiting, & anorexia in infants. An enlarged abd mass, on the other hand, may be a sign of neuroblastoma, but this condition is accompanied by profuse sweating & tachycardia. Although AGE may present w/peristaltic rushes, peristaltic waves are more likely a sign of pyloric stenosis. A positive psoas sign indicates appendicitis, no AGE.)

Mrs. Jones often catches her 13yo student, George, staring into space, daydreaming, & not paying attention in class. When she calls out to him to get his attention, he ignores her. He usually stares off for about 10-20 seconds, coming back to attention w/o any memory of the episode. George's behavior is most indicative of which of the following seizures? A. Tonic seizure B. Atonic seizure C. Absence D. Tonic-clonic seizure

C. Absence

Which form of leukemia accounts for about 20% of all childhood leukemia's & occurs primarily in infants & adolescents? A. Chronic myelogenous leukemia (CML) B. Chronic lymphocytic leukemia (CLL) C. Acute myelogenous leukemia (AML) D. Acute lymphocytic leukemia (ALL)

C. Acute myelogenous leukemia (AML) (AML is a form of leukemia that accounts for about 20% of all leukemia's & occurs primarily in infants & adolescents. ALL accounts for about 75% of pediatric leukemia cases, rather than 20%, & peak incidence occurs in children < 5yo. CML & CLL rarely occur in children.)

A 35-year-old woman is complaining of gradual weight gain, lack of energy, and amenorrhea. The urine pregnancy test is negative. The CBC shows a hemoglobin of 13.5 g/dL and an MCV of 84. The NP suspects that the patient may have hypothyroidism. The TSH is 10mU/L. Which of the following is the next step in the evaluation? Select one: a. Check the thyroid panel b. Check the total T3 and T4 levels c. Check for Antithyroid peroxidase antibodies d. Recheck the TSH in 4-6 months

Check the thyroid panel

Chlamydial conjunctivitis would most likely be treated with all of the following except: erythromycin doxycycline ceftriaxone azithromycin

Ceftriaxone

Which of the following medications would be the best first-line treatment for a patient with gonococcal pharyngitis?

Ceftriaxone

A male complains of a painful ulcer on his genitals, ulcer has a red halo. What antibiotic would be prescribed to treat chancroid ?

Ceftriaxone Azithromycin Ciprofloxacin Erythromycin

Which following medications you will prescribe for Chlamydia trachomatis caused PID?

Ceftriaxone and doxycycline with metronidazole

Tyler, age 20, fell during a basketball game and injured his left quadricep. In addition to recommending rest and immobilization, you want to help him with the pain. Which of these medications is least likely to see use in the initial treatment of Tyler's injuries? A. Naproxen B. Celecoxib C. Metaxalone D. Tramadol

Celecoxib

A 42-year-old secretary is diagnosed with carpal tunnel syndrome, and you are advising her on proper forms of management. Which of the following medications would be best for early treatment of her symptoms?

Celecoxib , and other NSAIDs may see use as the first step in pain management for carpal tunnel syndrome.

A 14-year-old boy has been complaining for several months of recurrent bloating, stomach upset, and occasional loose stools. He has difficulty gaining weight and is short for his age and symptoms are worse after eating large amounts of crackers, cookies, and breads. She denies seeing blood in the boy's stool. Which of the following conditions is most likely?

Celiac disease

In smallpox cases, initial lesions classically appear on oral mucosa, face, or forearms. Which form does the distribution of lesions usually take as the lesions continue to spread?

Centrifugal (concentrated on the face and extremities)

Your patient is admitted to the hospital with slurred speech and tingling in his right armand leg. An hour later, he appears to have fully recovered. However, he is alarmed to learn that about one-third of patients who have such attacks will experience which of the following within the next 5 years?

Cerebral infarction

Cervical cyanosis

Chadwicks Sign - first trimester

As a nurse practitioner, you know that an intrauterine pregnancy typically presents with the following specific indications in the first trimester except: amenorrhea urinary frequency breast tenderness change in skin pigmentation

Change in skin pigmentation (2nd trimester)

Which of the following patient findings constitute the strongest indication for pharmacologic revascularization?

Chest pain that has lasted for 3 hours, an ST segment elevation of 0.3 mV in the precordial lead, followed by an ST segment elevation of 0.3 mV in lead I

epididymitis usually presents with

Chills, malaise, fever, dysuria, urgency and frequency. Pain, swelling, redness of the scrotum, and patients may also experience low back pain, perineal pain. Positive prem sign

What STD is caused by a parasitic pathogen?

Chlamydia trachomatis is a parasitic intracellular obligate

Samuel, a 45 year old male, presents to the clinic with a cough, headache, excessive sweating, fever, sore throat, and soreness in the chest. A chest x-ray reveals the presence of infiltrates. Given the most likely diagnosis, which atypical pathogen that may have caused his condition?

Chlamydophila Mycoplasma Legionella

Whch of the following drugs is not a phosphodiesterase inhibitor? a. Sildenafil b. vardebafil c. Chlorophyllin d. Tadalafil

Chlorophyllin

Joe, a 54 year old male, comes to your practice with complaints of frequent drainage in his ear and decreased hearing for about 2 years. You examine his ear with an otoscope, and discover that his tympanic membrane is perforated. In addition, his ear canal is filled with mucous and peeling layers of scaly epithelium. Which conditions does the patient most likely have?

Cholesteatoma

A 31-year-old patient comes to your office complaining of hearing loss but only in the right ear. When examining her, you notice some erosion on her eardrum in the middle ear. Which of the following is the most likely diagnosis and treatment?

Cholesteatoma; surgery

Which of the following physiological changes is least likely a result of the effects of aging in older adults? a. Increased intestinal transit time b. Decreased liver size c. Constipation d. Impaired defecation signal

Constipation

Jason, age 18, comes to your practice with depression purple marks under his skin; however, the skin and bone are not broken an a quick physical exam does not detect any dislocation in the underlying joint Which of the following would best describe Jason's injury?

Contusion

You want to assess cerebral function in a patient experiencing memory loss. All of the following are main components of the examination that specifically assess cerebral function except: a. Thought processes b. Cognition c. Coordination d. Appearance

Coordination

Which of the following medications is least recommended for the tx of enuresis? A. Desmopressin B. Oxbutynin C. Flavoxate D. Nitofurantoin

D. Nitofurantoin

While assessing a patient during a physical, you find a small elevated area on her lower back. The area is a firm lesion of approx 1.5 cm; it feels smooth to the touch, is not filled w/fluid, & does not appear to extend below the epidermis. This morphology best describes which type of lesion? A. Wheal B. Tumor C. Cyst D. Nodule

D. Nodule

All of the following are typical physiological changes in the central nervous system of the elderly population except a. Decreased beta-adrenergic responsiveness (peripheral nervous system) b. Increased alpha responses c. Decreased dopamine receptors d. Increased muscarinic parasympathetic

Decreased beta-adrenergic responsiveness (peripheral nervous system)

Which of the following factors most strongly contributes to the development of anemia of chronic disease?

Decreased erythrocyte life span

Which of the following is considered an objective finding in patients who have a case of suppurative otitis media?

Decreased mobility of the tympanic membrane as measured by tympanogram

The first day of your pregnant patient's last normal menstrual period was on March 24. Using Naegele's rule, you estimate the date of confinement to occur on:

December 31

Which of the following methods would be most helpful in counteracting the somogyi effect in a diabetic patient

Decrease the patient's at bedtime dose of insulin

Joy, a 30-year-old female, is worried about the "mood" she has experienced for the past few weeks. She says that she has "felt really sad" nearly every day and wants to know if she is clinically depressed. Which of the following findings are required to confirm this diagnosis?

Diminished pleasure in almost all activities

What is the typical initial dosage of ethanol estradiol for combined oral contraceptives?

Dose of 35 mcg or less

Low alpha-fetoprotein could indicate

Down syndrome

The practitioner knows that which of the following prophylactic drugs is not used in pediatric migraine management? a. Doxepin b. Bisoprolol c. Ibuprofen d. Amitriptyline

Doxepin (tricyclic antidepressant)

Which of these choices would best treat a Rickettsia rickettsii disease that results and that your patient having a maculopapular rash, abdominal pain, joint pain, and flu like symptoms?

Doxycycline

Marge & Homer have brought their 3-day old son, Bartholomew, in for a WCE. Which of the following would you be least likely to conduct at this visit? A. Measure head circumference B. A tuberculosis screening C. Examine mouth & throat D. A jaundice examination

B. A tuberculosis screening (p.34)

Robin & Mindy bring in their 8-week old son, Jonathan, because he is irritable & his skin is warm to the touch. Taking his temperature, you discover it is 102.0. Which of the following is the most likely cause of the patient's signs and symptoms? A. Influenza A B. Group A Beta-hemolytic streptococci C. Respiratory syncytial virus D. Epstein-Barr

B. Group A Beta-hemolytic streptococci (p. 387)

All of the following are definitive reasons to re-examine and possibly replace a diaphragm as a contraceptive method except: a. Use of oil-based lubricants b. Wear and tear c. Being diagnosed with vulvovaginitis d. Gaining weight exceeding 20 lb

Being diagnosed with vulvovaginitis

A worried mother brings her 2yo daughter whose slight symptoms of a cold have progressed into a prominent cough, quick breathing, & wheezing. Which of the following would be your best diagnosis? A. Reflux B. Bronchiolitis C. Strep throat D. Seasonal allergy

B. Bronchiolitis

You have a 2yo patient who has an upper respiratory infection. Findings include a fever of 102 & signs of respiratory distress, including nasal flaring, grunting, & prolonged expiration a physical exam indicates a palpable liver & spleen. You would most expect to see which of the following is the patient's signs & symptoms are the result of a condition that could've caused all of these? A. Productive cough B. Chest x-ray with hyperinflated lungs C. Slight white blood cell á w/eosinophilia D. Peak flow < 60 L/min

B. Chest x-ray with hyperinflated lungs (p. 110-11)

An 18 mo old child's feet have an inward-turned appearance. Which of the following would most likely not be associated w/the patient's condition? A. Metatarsus adductus B. Genu valgum C. Adducted great toe D. Tibial torsion

B. Genu valgum (p. 40)

When diagnosing a heart defect with an x-ray, increased pulmonary vascular markings are oft-noted finding. Which of the following is most associated with the acyanotic atrial septal defect? A. Left ventricular hypertrophy B. Grade II systolic ejection murmur heard at left upper sternal border C. Holosystolic thrill felt at the left lower sternal border D. "Egg on a string" with cardiomegaly

B. Grade II systolic ejection murmur heard at left upper sternal border

A child diagnosed with eczema was treated with over-the-counter 1% hydrocortisone ointment. During a follow-up visit, the mother reports that the rash does not seem to be improving. What level steroid should the nurse practitioner prescribe next?

Group 6

a care map identifes all of the following points relevant to the management of a condition

Guidelines for planning and managing care delivered by all disciplines, identifies common patient-related problems of a specific case type, and sets out day to day goals that the patient must achieve as well as the final desired clinical outcome

Classes of medications that contribute to erectile dysfunction?

H2 receptor blocker, antihypertensives, diuretics

Which of these serologic markers would provide the first definitive evidence of a current hepatitis B viral infection?

HBsAg

Which statement most accurately reflects the relationship between HIV and CD4 cells?

HIV uses CD4 cells as receptors and reservoirs

What test is use to confirm HIV

HIV-1/2 antigen/antibody combination immunoassay, if positive proceed to HIV-1/HIV-2 antibody differentiation immunoassay

Of the following choices, the pracitioner recognizes which two as the most common pathogens responsible for sinusitis in the adult?

Haemophilus influenzae and Streptococcus pneumoniae

Peak expiratory flow is calculated using which factors? (Select all that apply.)

Height, Age, Gender Think HAG

Which of these symptoms would be least indicative of lower back pain? numbness along inner surface of thighs ataxic gait bladder problems Heightened proprioception

Heightened proprioception

A patient in the 3rd trimester of pregnancy presents with a swollen face and complaints of nausea and headaches that she says originate at the back of her head. You note that the patient's skin in slightly yellow, which is most likely a sign of what pregnancy-related complication?

Hemolysis, elevated liver enzymes and low platelets syndrome

types of pleural effusion

Hemorrhagic Transudate Empyema Exudate

Which variant of hepatitis is a blood borne RNA virus that is most commonly associated with intravenous drug use?

Hepatitis C

Which of the following is not a common finding of acute left-sided heart failure? a. Coarse rales in all lung fields b. Hepatomegaly c. Wheezing, frothy cough d. An S3 gallop sound

Hepatomegaly

The metabolic syndrome, also know as syndrome X, is associated with the risk of DM 2. It is characterized by a group of predisposing factors that include obesity and hypertension. What additional risk factors are associated with syndrome X?

High triglyceride levels and low HDL levels

According to the Eighth Joint National Committee hypertension guidelines, which of the following is used to estimate 10-year and lifetime atherosclerotic cardiovascular disease risks?

High-density lipid cholesterol, Race, Diabetes status, systolic BP, race, total cholesterol, and history of smoking.

A patient taking oral contraceptive may be at increase risk for which of the following conditions as the patient's age, dose, and length of therapy increase?

Hypertension

While performing a fundoscopic exam on your patient, you notice several arterioles that cross a venule, producing bulging on either side. Your patient, a 57-year-old African-American male, is slightly overweight and admits to smoking approximately one pack of cigarettes a day. Which of the following is the most likely cause of the findings?

Hypertension

A patient w/heart disease has headaches & dizziness. He also states that he has visual & breathing problems. Based on these findings, what would be the most likely diagnosis for this patient? A. Rheumatic fever B. Hypertension C. Kawasaki disease D. Pulmonic stenosis

Hypertension (p. 68)

The most common comorbidities that occur with type 2 diabetes mellitus are:

Hypertension, hyperlipidemia, obesity

Which of the following conditions is the most common cause of sudden death among athletes?

Hypertrophic cadiomyopathy

You are examining a chest x-ray of a patient with chronic right-sided heart failure. Which of the following would you expect to find?

Kerley B lines, Pulmonary edema, Pleural effusions

Which of these signs would indicate meningitis in a patient?

Kernig's sign Pain upon extension of the leg when the hip is flexed 90°

Which of these would not be considered a first line treatment for inflammation of the glans penis caused by candida albicans? Miconazole Ketoconazole Clotrimazole Fluconazole

Ketoconazole

Fetal monitoring commonly involves the conjunction of a non-stress test, biophysical profile, and which other diagnostic procedure?

Kick count

A 52 year old female comes to your practice with complaints of breathlessness and a cough accompanied by excessive phlegm. She produces a sputum sample, which appears clear upon inspection. You order a pulmonary function test; in reviewing the results, you find evidence indicating both an increased functional residual capacity and an increased total lung capacity. Which respiratory diseases would be the most likely diagnosis?

Emphysema

Which of these conditions does not usually warrant the need for progestin- only pills? a. Migraine headaches b. Hypertension c. Endometriosis d. Obesity

Endometriosis (combo pill decreased pain)

Which vaccine helps prevent and protect against hep B?

Energix- Recombivax-B, Heplisav-B. Twinrix is a combination of Hep-A and Hep-B

After a car accident, Renee, a 35-year-old female, needs surgery. You are called to coordinate her care. She has refused a blood transfusion because she is a Jehovah's Witness. However, her husband asks that you give her blood if needed. Of the following, which is the correct choice of action?

Ensure proper documentation and follow Renee's refusal of a transfusion.

You are a part of a task team charged with evaluating the quality of health care in every day practice. Since you round on your primary care patients at a community hospital, to accomplish this task, you choose to apply the standards of Quality Improvement, Quality Assurance, and Continuous Process Improvement. These standards would best identify all of the following indicators related to aspects of care except: a. Establishing thresholds for evaluation b. Collecting data to identify problems in health care delivery c. Ensuring that nurses maintain proper licensing d. Assessing the effectiveness of clinic documentation

Ensuring that nurses maintain proper licensing

How often should prenatal checks be traditionally scheduled for a patient who is 30 weeks pregnant?

Every 2 weeks (28 to 36 weeks)

Undesirable side effects of oral contraceptives such as depression, fatigue, and decreased libido usually primary result from:

Excessive progesterone

Of the following, which is a necessary guideline for interviewing adolescents in a clinical setting?

Excluding the parents from the interview with the adolescent

A patient comes to your practice with a knee that keeps "locking," making it difficult for him to walk. You perform the apple's grind test on the patient to determine the presence of soft tissue damage. What would be the best method for carrying out this test?

Flexing the knee 90 degrees, and then applying pressure to the heel while rotating the lower leg. (pain or click is positive)

Which method is most commonly used in completed suicides?

Firearms

The measles, mumps, and rubella (MMR) vaccine must be administered in two doses. When should the MMR vaccines typically be given?

First dose: 12 to 15 months old; second dose: 6 years old

A sexually active young adult is diagnosed with Chlamydia trachomatis and treated with doxycycline 100 mg BID 7 days. During a follow-up visit, the patient reports a new onset of right upper quadrant pain and tenderness on palpation. Alanine aminotransferase (ALT) is 43 U/L, and aspartate aminotransferase (AST) is 24 U/L. The nurse practitioner suspects:

Fitz-Hugh-Curtis syndrome (PID)

A patient starting the contraceptive patch places her first patch on the first Sunday of the month. She would replace this patch with a new one on each of the following Sundays except?

Fourth Sunday. The patient is removed and replaced a week later

After observing a red reflex, a fundoscopic exam should proceed from the optic disc and end at which part of the eye?

Fovea centralis

An adult patient presents with fatigue. The patient has a thyroid-stimulating hormone (TSH) level of 11.3 mIU/L. Which result leads the nurse practitioner to start the patient on levothyroxine (Synthroid)?

Free T4 level of 0.4 ng/dL

The practitioner knows that the peak age go onset for xanthelasma is:

From 40-50 years of age

Adam, age 26, has a seizure that initially originates in his arms before spreading to the entire left side of his body. He remains conscious through the entire seizure. Based on this type of seizure, which of the following would you most expect Adam to also experience?

Flashing lights (simple partial seizure)

Your patient has severe restrictive lung disease. The nurse practitioner would expect that all of the following pulmonary function test results should be significantly lower than normal.

Functional residual capacity Forced vital capacity Residual volume

A patient who presents with white purulent penile discharge and six maculopapular pustular lesions. He also complains of pain in his testicles and frequent nausea. what is his most likely diagnosis?

Gonorrhea, Lesions are not always present but when they are they are painful and there can be as many as 5-40.

Certain STDs can cause males to have urethritis and in rare cases this a can lead to epididymitis and male infertility. What STD?

Gonorrhea. Treat with ceftriaxone and Doxy

Softening of the cervix and the vagina indicates which sign?

Goodell's sign

A female presents with an ankle sprain. She is unable to walk unassisted, an egg-shaped swelling is identified around the affected area. The patient's ankle sprain should be classified as which of the following?

Grade 3

You are examining Victor, age 67, and identify a heart murmur. This murmur is fairly loud and can be heard with one corner of the stethoscope off the chest wall. You are most likely listening to which of the following murmurs?

Grade IV/VI

Harriet, age 54, is waiting in the clinic when her muscles become tense. She loses consciousness and develops consistent contractions. The episode lasts for about three minutes, and you notice that she loses control of her bladder during the event. How should the nurse practitioner document these findings?

Grand mal seizure

A 36-year-old patient presents with concerns over experiencing uncontrollable fine tremors in her hands, feeling anxious and nervous, and that her emotions seem to have been fluctuating "up and down" over the past 2 weeks. Her complaints are most likely associated with which conditions?

Graves disease

A 36-year-old patient presents to your office with concerns over experiencing uncontrollable fine tremors in her hands. When you ask her about other concerns, she tells you she has been feeling anxious and nervous, and that her emotions seem to have been fluctuating "up and down" over the past 2 weeks. Her complaints are most likely associated with which condition?

Graves' disease

Joseph, 31, complains of a few painless, "smooth, raised bumps" on his penis. He says that the bumps are fairly firm and match the color of his skin but "do not hurt at all." Without further examining the patient, with what condition can you diagnose the patient?

Molluscum contagiosum

A patient presents to your practice with pain and loss of movement in his right knee. As you are performing the physical exam, you notice a loss of the patellar tendon reflex (knee jerk) and difficulty moving his quadriceps. The patient's pain most likely stems from which of the following?

L3-L4 disk compression

When performing an eye exam, if you have difficulty visualizing the macula, what instruction should you give the patient?

Look into the light

You are considering medications to manage a patient's gastroenteritis. Which of the following agents should not be used if the patient's symptoms include bloody stools? a. Ciprofloxacin b. Loperamide c. Doxycycline d. Metronidazole

Loperamide

Which of the following lab values would most likely confirm a diagnosis of hyperthyroidism?

Low TSH, increased free T3, normal T4

A 70-year-old man with open-angle glaucoma is prescribed timolol (Betimol) ophthalmic drops. All of the following are contraindications to Betimol ophthalmic drops, except: Overt heart failure or sinus bradycardia History of asthma Second- or third-degree atrioventricular (AV) block Migraine headaches

Migraine headaches

A 45-year old female, presents to the clinic with rounded, firm, painless bumps along her lower abdomen and genital region. The papules are 3 mm and pearly-white. Which is the most common treatment for the most likely condition?

Molluscum contagiosum and cryoanesthesia with liquid nitrogen is the best form of removal.

Upon examination, you notice that Alex. an obese 63 year old male, has moderate dyspnea and purulent sputum. His lungs are normal upon percussion. Laboratory results reveal an increased hematocrit level. Given the most likely diagnosis, which of the following drugs would you be least likely to prescribe for the patient's condition? a. Ipratopium bromide b. Albuterol c. Budesonide d. Montelukast

Montelukast (not for chronic bronchitis)

A 7-year-old patient is brought to you complaining of difficulty drooling and swallowing. During the examination you note that the epiglottis is inflamed. Which of the following pathogens is usually not associated with this type of inflammation? Streptococci Pneumococci Moraxella catarrhalis Haemophilus influenzae

Moraxella catarrhalis

Regarding the incidence of myasthenia gravis, which of the following is most accurate?

More common in women

Which of the following musculoskeletal disorders would most likely compel a referral for cryogenic neuroablation?

Morton's neuroma

Of the following,which statement is most accurate regarding suicide?

Most people who state intent to commit suicide actually follow through

A patient complaining os intense itching in her vaginal area and yellowish-green discharge with smells like old gabage. Inspecting the vulvovaginal region, you find erythema and red spots around the vagina. You order a wet prep test and expect which of the following results?

Motile trichomonads

A patient with the following lesion presents to the clinic. A diagnosis of syphills would be most likely confirmed if the lesion are

Mucous

Which of the following is indicative of gallbladder inflammation?

Murphy's maneuver

During a physical examination, the nurse practitioner palpates the right upper quadrant while the patient takes a deep inspiration. Which diagnostic test is being performed?

Murphy's sign

Tension and psychogenic headaches are commonly associated with what mechanism of headache pain?

Muscular contraction

Which of the following gynecologic cancers has the highest mortality rate?

Ovarian

A patient is at risk for secondary hypothyroidism if they have which condition?

Pituitary adenoma

A patient in her 28th week of pregnancy arrives to your clinic concern of vaginal bleeding. Upon questioning, she tells you that the bleeding is not accompanied with any pain and that the bleeding occurred after vaginal intercourse. Based on these signs, you should screen the patient for which of the following conditions?

Placenta previa

Of the following, which medical imaging technique is most routinely used to rule out other bone or joint conditions in patients with suspected cases of bursitis?

Plain X-rays

Your male patient says that he experiences severe, sharp pain in the bottom of his right foot when he wakes up in the morning, which subsides throughout the day. He describes a dull pain in the heel and a throbbing ache in the arch of his foot.

Plantar fasciitis

All of the following musculoskeletal disorders affect women at the higher rate than men except: A. Rheumatoid arthritis B. Carpal tunnel syndrome C. Morton's neuroma D. Plantar fasciitis

Plantar fasciitis. Plantar fasciitis is a particularly common skeletal disorder among runners, but there is no higher prevalence in a specific gender.

Trimethoprim-Sulfamethoxale is given to AIDs patients to prophylactically to prevent what disease

Pneumocysitis Pneumonia

An HIV-positive male who is in your care has been complaining of shortness of breath and a serious cough. Given this information, which condition should you be most cautious to monitor for in this patient?

Pneumocystis jiroveci pneumonia

Which test is used to differentiate prior exposure to the hepatitis C virus from a current infection?

Polymerase chain reaction

An obese patient is usually at an increased risk of developing all the following except: A. Osteoarthritis B. Plantar fasciitis C. Low back pain D. Polymyalgia reumatica

Polymyalgia reumatica

A woman comes to your practice with complaints of stiffness in her shoulder and "this terrible ache" in her hips. You also find that she has anemia and mild fever. Which of the following is the patient most likely experiencing?

Polymyalgia rheumatica

Which finding would provide the most definitive diagnosis for tubercolosis? a. Acid-fast bacili on a sputum smear b. Positive purified protein derivative c. Homogenous infiltrates in the upper lobes on an x-ray d. Positive culture for tubercle bacili

Positive culture for tubercle bacili

An 18 year old college student presents to urgent care with complaints of severe periumbilical pain. While being evaluated, the patient is lying very still with his right thigh drawn to his chest. Upon extension of the right thigh, the patient complains of strong pain. Which of the following signs should you determine is positive?

Psoas

Which of the following contraceptives almost always contains the chemicals nontoxynol-9 and octoxynol?

Spermicides

A patient with an intrauterine device learns that she is pregnant. If the device is not removed, which of these complications is most likely to occur?

Spontaneous abortion

The discharge from allergic conjunctivitis can best be described as which of the following?

Stringy and white

Which has the fastest onset and longest duration for erectile dysfunction?

Tadalafil or Avanafil works in 15min. Take with or without food

A patient presents for follow-up with the nurse practitioner after receiving a diagnosis of acute diverticulitis. The nurse practitioner is educating the patient about how to avoid complications. What instructions should be included? (Select all that apply.) Maintain a high-fiber diet. Take amoxicillin-clavulanate 875/125 mg BID as ordered Take a laxative every other day Use psyllium fiber supplementation daily Report fever to healthcare provider immediately

Take amoxicillin-clavulanate 875/125 mg BID as ordered Report fever to healthcare provider immediately

Whch Tanner stage of breast development is characterized by projection of the areola and nipple as a secondary mound?

Tanner stage 4

What is the primary role of a polymerase chain reaction (PCR) in the treatment of a patient with HIV?

The PCR helps quantitate the viral count of HIV in an infected person

Power of attorney must usually meet a criteria before it will be honored by most institutions?

The document must form a part of the patient's health care directive

Which of the following is most true regarding the treatment of hypothyroidism with levothyroxine?

The dosage should be given every day

An otherwise healthy 45 year old patient with atypical pneymonia would best be treated with which medication?

amoxicillin, doxy, or azithromycin

What is an example of battery

angrily striking a patient on the shoulder when he or she will not stand still

What type of drug should elderly patients take 1 hour before or 4 hours after other medications

antacids

Which of the following is the most frequent site of nose bleeds?

anterior septum

ST-segment elevation in leads V1-V6 indicates which type of infarction?

anterior wall

Most stage-based theories of development focus primarily on: a. The continuity of development b. The discontinuity of development c. Persistence of inherent personality characteristics d. The influence of context on development

b. The discontinuity of development (These theories address deviations from developmental progress norms.)

A 79-year-old male having trouble with my urine. His prostrate-specific antigen was noted to be 6.0 ng/ml. The nurse practitioner considers a referral for which of the following tests at this time?

Transrectal ultrasound

While treating 29-year-old Sue for vulvovaginitis, you inquire about her sexual history. She explains that she has been sexually active for about ten years. There are many causes of vulvovaginitis, but only one is a sexually transmitted disease. Which of the following is it?

Trichomoniasis

Josephine, age 63, experiences severe pain in her face, claiming that it feels like "an electric shock." In discussing her history, she mentions that her gait has become stiff and unpredictable as of late, and reports experiencing blurred vision. Which of the following is the most likely diagnosis?

Trigeminal neuralgia

Which effect will the nurse practitioner take into consideration when prescribing oral pharmacologic treatment for a 2-week-old infant with omphalitis?

Undeveloped blood-brain barrier

Menopause often presents with the following

breast reduction, sleep disturbance, stress incontinence, increase melanin, thinning of the baginia, dry skin, decreased skin elasticity

According to the most recent CDC recommendations for treating PID, which of the following drugs would not be paired with doxycycline? a. cefoxitin b. probenecid c. metronidazole d. butoconazole

butoconazole

A menopausal patient starting hormone therapy may not need as much daily intake of which supplement?

calcium

Which of the following is NOT a Dermatophyte infection? Candida balanitis Tinea capitus Ringworm Onychomycosis

candida balanitis

The image below illustrates which bacterial issue?

carbuncle

A 62 year old male patient presents with cloudy vision in a single eye. In addition, he complains of glare coming off of bright lights and difficulty with night vision. Which of the following is the most likely diagnosis?

cataracts

CDC recommendations for treating PID

ceftriaxone 500 mg IM in a single dose6 PLUS doxycycline 100 mg orally 2x/day for 14 days WITH metronidazole 500 mg orally 2x/day for 14 days OR cefoxitin 2 gm IM in a single dose AND probenecid 1 gm orally, administered concurrently in a single dose PLUS doxycycline 100 mg orally 2x/day for 14 days WITH metronidazole 500 mg orally 2x/day for 14 days

A 30 year old with yellow-green penile discharge, testicular pain, and nausea and vomiting. What medication should be included for uncomplicated gonorrhea

ceftriaxone 500mg IM plus doxy

A 21-year-old male comes to you complaining of a painful bump with a small, red-looking halo on his penis. The affected area is about 5 mm in diameter, soft, and tender to the touch. He states that the halo has been there for about a week and is painful.

chancroid

most common leukemia in adults

chronic lymphocytic leukemia

Rib notching due to collateral circulation which also commonly presents with cardiomegaly is a distinguishing feature of

coarctation of the aorta

Which of the following statements is true regarding abruptio placenta?

cocaine, alcohol, and cigarette use contribute to the incidence of abruptio placentae

What is the name of the occupational splint used for carpal tunnel syndrome

cock up splint

A 35-year-old Sarah says that she and her partner are In a committed relationship and are looking for a long-lasting, effortless birth control method. She has an allergy to estrogen-based birth control. Under the circumstances, which of these choices should you recommend?

etonogestrel implant

obstructive lung disease

example Bronchiectasis, COPD, asthma, cystic fibrosis

Which of the following is most likely to be found on the fundoscopic examination in a patient with untreated POAG?

excessive cupping of the optic disk

Undesirable side effects of oral contraceptives such as breast tenderness, nausea, headache, irritability, bloating, usually primary result from:

excessive estrogen

Which of following symptoms is associated with PID?

fever, chills, dysuria, dyspareunia,

Which of the following is a finding of acute bacterial prostatitis?

fever, chills, low back pain, dysuria, nocturne, increased urgency or frequency of urination

What is typically considered the seven key principles governing ethical conduct for nurse practitioners?

fidelity, veracity, justice, autonomy, beneficence, utilitarianism, nonmaleficence,

Tretinoin cream is specifically used as the first-line treatment for two kinds of warts?

filiform and flat

Of the following causes of conductive hearing loss, which is the most common?

foreign body

A nurse practitioner is reviewing Pap results of a 30-year-old patient. Results indicate a normal Pap screen with positive human papillomavirus (HPV), strain 16. Which follow-up testing is recommended?

colposcopy

A 24-year-old patient comes to your office with a sore throat, nasal congestion, and a cough. The patient repeatedly clears her watery congestion into a tissue. The patient does not have a fever, and a physical exam is unremarkable. Given only this information, what is the most likely cause of her condition?

common cold

You are performing a research study to determine the effects of smoking. You collect data from two sample groups. The first group consists of smokers between the ages of 21 and 45, and the second group consists of smokers between the ages of 65 and 80.which type of research does this study best exemplify?

cross-sectional

Victor, a stocky 40 year old male, with complaints of difficulty breathing and "endless amounts of gunk whenever [he] cough[s]." During the visit, he coughs up a substantial amount of yellow phlegm. A blood test reveals an increased hematocrit level, and a physical examination detects lungs that are normal upon percussion. Given the most likely condition of Chronic bronchitis, which of the following findings would you expect?

decreased forced expiratory volume in 1 second increased total lung capacity Increased functional residual capacity Increased residual volume

What finding is most often directly linked to low protein intake

decreased healing time

A male patient diagnosed with depression threatens to kill himself. To keep the patient from harming himself, the nurse practitioner may legally perform all of the following action except: a. Order medication to incapacitate the patient to prevent him from harming himself. b. Immediately restrain the patient without informing other staff. c.Restrain the patient with a reasonable amount of properly checked restraints. d.Temporarily commit the patient, then transfer him to care of his relatives regardless of mental state

d.Temporarily commit the patient, then transfer him to care of his relatives regardless of mental state

What is the best first line treatment choice to prescribe to a 79 year old patient with chronic hypertension and heart failure

enalapril

Key diagnostic findings in Primary open-angle glaucoma (POAG) include which of the following?

intraocular pressure greater than 25 mm Hg.

While going over lab results, you explain that chelation therapy is not recommended because the toddler's venous blood level concentrations do not reach which threshold 35 45 65 67

level 35

A fair skin 47-year-old undergoing a periodic health assessment presents with pink patches on her upper chest. She says that the patches appeared about a year ago. Within the past couple of weeks, though, the patches have started to itch. Which would best treat the patient's most likely condition?

liquid nitrogen

Which diabetic med should be avoided in patient with gastroparesis

liragkutide (victoza)

Which first-line treatment should be administered when treating a patient with chronic obstructive pulmonary disease (COPD) category C?

long-acting muscarinic agonist (LAMA)

You inform your patient that he is in the latent stage of syphillis. What signs and symptoms would best characterize this stage of the disease?

no signs or symptoms

How and where does the obturator sign present in pediatric patient with appendicitis?

pain with internal rotation of the right hip

The patient has lymphogranuloma venereum LGV (L3 immunotype of chlamydia trachoma )What sign or symptom would be most expected from the suspected condition

painless ulcers with tender regional adenopathy . Treat with Doxy, azithromycin, or erythromycin

Which of the following would least qualify as an advanced activity of daily living? Routine phone calls and visits with neighbors grocery shopping and making dinner Teaching music at the community center Reading poetry

grocery shopping and making dinner

You suspect that your patient , Susana, has sickle cell a chronic, normocytic,hemolytic anemia. Select the best description of the diagnostic procedure that would best confirm the most likely condition?

hemoglobin electrophoresis on citrate agar gel

If a patient is experiencing a painful, blistering eruption in a dermatolmal distribution that resembles the following, which is the most likely diagnosis ?

herpes zoster

What two drugs are typically prescribed to induce an elective abortion within the first 49 days of pregnancy?

prostaglandin and mifepristone

Diastolic heart failure also results in decreased cardiac output; however, this is a result of the heart's inability to

relax and fill

The practitioner knows that all of the following agents are commonly used to treat candida intertrigo except: Ciclopirox Itraconazole Selenium Sulfide Cornstarch

selenium sulfide

What would be associate with hypersominia in an elderly patient

sleep apnea, chronic use of hypnotic medications, unusual daytime sleepiness as a symptoms

The FNP understands that when determining whether to incorporate a new procedure into a clinical practice based on the findings of a recent quantitative research study, which of the following should be considered? (Select 2 responses.)

statistical significance of the findings methodological limitations of the study

As a nurse practitioner, you know that the most common cause of cellulitis in outpatients is ?

streptococccus progenies

What is the most common etiology of pneumonia in children 1-3

streptococcus pneumonia

Emotional symptoms of premenstrual syndrome

tension, irritability, confusion, mood swings

A boot-shaped heart is a characteristic x-ray finding of

tetralogy of Fallot.

Values of MCV=72, MCHC=30%, and reticulocyte count = 3%, respectively suggestive of

thalassemia

The most important clue for ulcerative colitis is bloody stools that are covered with mucus and pus along with

the systemic symptoms fatigue, low-grade fever.

Periumbilical to right lower quadrant pain occurs more frequently in CD, and left lower quadrant pain is more classically associated with

ulcerative colitis (UC).

You order a regimen of antibiotics to treat tuberculosis, which includes 15 mg/kg of ethambutol. With this medication as part of the regimen, which of the following conditions should the patient be tested for ?

visual acuity and red-green color perception

Which of the following supplements is used to potentially slow cognitive decline in AD? A. vitamin B12 B. vitamin E C. ginkgo biloba D. St. John's wort

vitamin E

Which of the following is more likely to cause a pregnancy loss in the second trimester?

A patient with uterus didelphys

Which of these is covered under the auspices of the Danforth Amendment?

A patient's right to refuse care when admitted to a federally- funded institution

A 71-year-old male with an active lifestyle presents to your practice with complaints of pain in his right shoulder. Which of these statement from the patient would most strongly indicate bursitis?

"It hurts when I move my arms over my head."

A premature infant should typically be given palivizumab for the prevention of RSV bronchitis if gestation is less than

29 weeks

A nurse practitioner evaluates a 16-year-old male patient for a routine exam. He completed a single human papillomavirus (HPV) 4-valent recombinant vaccination four years ago. Which comment from the parent indicates an understanding of the recommended vaccination schedule?

"Because the series was started prior to age 15, he needs a second dose of the 9-valent vaccine today to complete the vaccination series."

Going by the standard CAGE mnemonic, which of the following questions is least helpful in the diagnosis for alcohol abuse? a. "Have you ever felt the need to cut down on your drinking? " b. "Have you ever justified having a drink for any reason? " c. "Have you ever felt guilt about your drinking? " d. "Have people annoyed you by criticizing your drinking?"

"Have you ever justified having a drink for any reason?"

Sally, a 27-year-old female, reports to your clinic with complaints of recurring headaches.When you ask her to describe the nature of the pain, she says: "I don't know, it just hurts." You decide to ask her about signs and symptoms that accompany the headaches. All of these statements would suggest Sally is experiencing cluster headaches except: a. "It feels like my nostrils just close up. "b. "I keep crying and I don't know why. "c. "My nose just starts running like a waterfall. "d. "I feel like the light starts stabbing at my eyes."

"I feel like the light starts stabbing at my eyes.

Which of these statements would most likely suggest that your female patient was predisposed to the hepatitis A virus?

"I recently returned from a trip to Haiti doing hurricane relief work. Had some good oysters over there"

A 32 year old Asian female is in the emergency department after being assaulted by her husband. Her history shows that she has been seen 2 times within the past 6 weeks for bruises and facial abrasions. Which of the following statements best demonstrates the nurse practitioner's ethical responsibility to this patient before discharge?

"I want you to know that this is not a normal relationship, and I would like for you to talk with someone here today about your family relationship."

Which of these patient statements would most warrant a diagnosis of tension headaches?

"It feels like something is squeezing my head."

Jennifer, an 18 year old female is seeking emergency contraception. She claims the condom broke last night during the act. You believe that the levonorgestrel (plan B) would best address her concerns. Which of the following statements would be most accurate?

"You may experience menstrual irregularities during your next cycle" . (Plan B can be given up to 72 hours )

Scheduling of prenatal visits

0-28 every 4 weeks 28-36 every 2 weeks 36-delivery every week

Second trimester

13-27 weeks

Trimesters of pregnancy

1st 0-12 weeks 2nd 13-27 weeks 3rd 28-40 weeks

In general, depression must be present for at least how long to qualify for a dysthymia diagnosis?

2 years

When are Leopold maneuvers first able to be performed?

20 weeks

The typical first-year rate of spermicides is approximately

21%

A 73-year-old male patient presents with bilateral edema in the legs and feet, weight gain of 15 lb, and decreased urine output. The nurse practitioner will order:

24-hour creatinine level (Acute renal failure)

At which point during pregnancy should women expect to see the most significant abdominal growth?

28 weeks

The practitioner teaches a patient that an eruption of pityriasis rosacea typically lasts for how long>?

4 to 8 weeks?

Which of the following values best represents a normal red blood cell concentration in a 50 year old male patient?

48% (40-54%)

A 30-year-old African-American patient complains of being constantly out of breath even in the absence of strenuous physical activity. He has also noticed the sudden onset of severe pain in his back and chest. Lab tests show a hemoglobin level of 13 g/dL. Given your suspicion of anemia, you know that which of the following would be least appropriate to order as a course of treatment?

5 units of packed red blood cells

In a healthy person, what percentage of the body's total daily physiological insulin secretion is released as basally?

50-60%

The span of the normal adult liver is:

6 to 15 cm in the midclavicular line

Which cranial nerve controls hearing and equilibrium?

CN VIII

What feature best distinguishes the characteristic murmur of patent ductus arteriosus?

A "machinery" noise at the left upper sternal border

Which of these patients represents a demographic for which suicide is the second leading cause of death? a. A 50 year old Caucasian male b. A 77 year old African American female c. A 17 year old African American male d. A 22 year old Caucasian female

A 17 year old African American male

All of the following patients should be screened for diabetes mellitus except: Select one: a. An obese man of Hispanic descent b. An overweight middle-aged Black woman whose mother has type 2 diabetes c. A woman who delivered an infant weighing 9.5 lb d. A 30-year-old White male with hypertension

A 30-year-old White male with hypertension

Which of the following patients would you expect to have the highest risk of developing multiple sclerosis?

A 35-year-old Irish female

In which of the following patients would the NuvaRing be contraindicated?

A 36 year old female who smokes

You are treating four patients who experience recurring headaches. Which of the following patients is most likely to be experiencing cluster headaches?

A 45-year-old man whose headache lasts for 1 hour and usually returns a month later.

Which of these patients would be most likely to be affected by a cluster headache?

A 55-year-old male

Which of these patients is most at risk of developing dementia?

A 62-year-old with ventricular dilation

In July, Marian brings in her 5-year-old son Jensen, who has just started presenting w/the varicella zoster virus. His contraction of the virus was on purpose. Marian brought Jensen to a "chickenpox party" where a child w/the virus can spread it to the other kids so that all the children can get the virus now & not when school is in session. Marion asks you for a treatment that specifically reduces the duration of his signs & symptoms. What is the best treatment recommendation for her? A. Acyclovir B. Antihistamine C. Acetaminophen D. Calamine

A Acyclovir (p. 87)

Which statement is true statement regarding hypertension?

A classic presentation of hypertension includes a suboccipital, "pulsating" headache, usually in the morning, which resolves throughout the day.

The mechanism of action of metformin (Glucophage) is as:

A drug that increases insulin action in the peripheral tissues and reduces hepatic glucose production

Patients diagnosed with hyperthyroidism face a variety of treatment options, depending on etiology, severity, and other factors. Radioactive iodine therapy is one such option that should be specifically considered in which of the following patients

A female diagnosed with Graves' disease

Of the following, who is most likely to experience symptoms of premenstrual syndrome/premenstrual dysphoric disorder?

A fit, healthy 33-year-old with a high stress job

Metabolic Syndrome (Syndrome X)

A genetic metabolic disorder characterized by diabetes, hypertension, atherosclerosis, centrally distributed obesity, and elevated blood lipids

When using the TNM Classification of Malignant Tumors staging system to classify the spread to regional lymph nodes, all of the following grades would be appropriate except: A grade of "4" A grade of 0 A grade of X A grade of 3

A grade of "4" (only goes to 3)

Which of the following patients has the lowest risk of developing colon cancer? a. A female whose aunt and grandmother both developed ovarian cancer b. A hispanic male with a history of peptic ulcer disease c. A female with a high-fat diet that includes red meat and refined carbohydrates d. A male with a history of recurrent inflammatory bowel disease

A hispanic male with a history of peptic ulcer disease

You are assessing for egophony in a patient with suspected pneumonia. You ask the patient to produce a lone E sound. What should should you anticipate to osculate when the patient has lung consolidation?

A long A or ay sound

A 75-year-old diabetic male requires treatment for pneumonia. Which of the following drugs is not recommended for this patient? Gemifloxacin Moxifloxacin A respiratory fluoroquinolone A macrolide antibiotic

A macrolide antibiotic

Which of these scenarios best describes the primary component of malpractice?

A nurse practitioner acts hastily in treating an aneurysm, resulting in otherwise avoidable partial paralysis

Which of these patients would most likely need to have her diaphragm refitted?

A patient who has gained approximately 25 lb

Which of the following patients, all of whom have a known history of congenital heart disease is least likely to require prophylactic antibiotics prior to a dental procedure? a. A patient with a heart transplant b. A patient with a ventricular septal defect with patch repair c. A patient with a previous history of endocarditis d. A patient with a partially repaired cyanotic heart defect

A patient with a heart transplant

A 54-year-old former professional athlete is being screened for hypertension, a condition which runs in his family. Which of the following findings is not consistent with a diagnosis of significant hypertension? Epistaxis A S4 heart sound A subparietal headache Blurred vision

A subparietal headache

Of the following cases, which of the following would you recognize as a child whose development is not that of a typical healthy child? A. A 2 ½ year old who usually speaks in 1-word sentences B. A 15-month-old who scribbles with a large crayon C. A 2-year-old jumps with both feet D. The 3 month old who lifts his head & chest when on his stomach

A. A 2 ½ year old who usually speaks in 1-word sentences (p. 11)

Which of your patients would you most expect to present with Burtonian lines? A. A 3yo who frequently spends her afternoons @ her grandfathers stained glass studio B. A 4yo who is living in a 1980's house while it is being renovated C. A 7yo who receives acupuncture to help treat his anxiety D. A 9yo who spends most of his free time playing video games

A. A 3yo who frequently spends her afternoons @ her grandfathers stained glass studio (p. 137)

You are assessing for egophony in a young pt w/suspected pneumonia. You ask the pt to produce a long "E" sound. What sound should you anticipate to auscultation when the pt has lung consolidation? A. A long "A", or "ay" sound B. A short "A", or "ah" sound C. A long "E", or "ee" sound D. A short "E", or "eh" sound

A. A long "A", or "ay" sound

Which of the following patients, all of whom have a known history of congenital heart disease, is least likely to require prophylactic antibiotics prior to a dental procedure? A. A patient w/a heart transplant B. A patient w/a ventricular septal defect w/a patch repair C. A patient w/a previous history of endocarditis D. A patient w/a partially repaired cyanotic heart defect

A. A patient w/a heart transplant

In the correct method for assessing an infant for developmental hip dysplasia, the NP would have the baby on his back with knees flexed &: A. Abduct knees, listening for click as femoral head slips into acetabulum B. Adduct knees, listening for click as femoral head slips into acetabulum C. Abduct knees, feeling femoral head enter acetabulum D. Adduct knees, feeling femoral head slip into acetabulum

A. Abduct knees, listening for click as femoral head slips into acetabulum (Ortolani' s click, commonly used to assess for developmental dysplasia of the hip (DDH), is performed by abducting the knees, then listening & feeling for the femoral head to re-enter the acetabulum. Barlow's maneuver, which also assesses for DDH is done by abducting the knees & feeling the femoral head pop out of , not into, the acetabulum.)

A 16-year-old male presents to your office with a 2 day history of fatigue, cough, & chills. His temp today is 99F. His lung sounds are coarse. The patient has a history of good health but "feels awful" today. Which of the following would be the most appropriate treatment recommendation? A. Acetaminophen B. Levofloxacin C. Ceftriaxone D. Azithromycin

A. Acetaminophen (In healthy patients with viral infections, supportive measures such as hydration & acetaminophen are the most appropriate treatment option. The patient's chills, low fever, and coarse wet lung sounds point to a viral cause rather than bacterial pneumonia. A bacterial infection typically presents with a much higher fever. Azithromycin, levofloxacin, and ceftriaxone are all antibiotics that do not treat this patient's viral infection.)

A mother brings her son to your facility, who has never had serious disease before. The 3yo boy is in preschool & has contracted the Coxsackievirus. At this time, which of the following would be most appropriate to give the child to treat his signs & symptoms? A. Acetaminophen B. Acyclovir C. Amoxicillin D. Penicillin

A. Acetaminophen (p. 97)

You are examining Ryan, a 17 yo male, & note inflamed papules on his face & upper trunk. He admits to using steroids in order to keep up w/his teammates in baseball; these steroids appear to have exacerbated the symptoms. Of the following, which dermatological disorder is the most likely diagnosis? A. Acne B. Psoriasis C. Atopic dermatitis D. Pityriasis rosea

A. Acne

Which of these factors is most likely to increase the risk of Down syndrome in infants? A. Advanced maternal age B. Intrauterine infection during the 2nd trimester C. Alcoholism during pregnancy D. Smoking during pregnancy

A. Advanced maternal age

You are counseling one of your pregnant patients on the prevalence of genetic disorders. If the baby has no predisposition for a specific genetic disorder, his chance of being born w/a major malformation is most likely: A. Approximately 2% B. Approximately 5% C. Approximately 8% D. Approximately 10%

A. Approximately 2%

Which of these dermatological disorders is most commonly associated w/elevated serum IgE levels? A. Atopic dermatitis B. Acne C. Pityriasis rosea D. Varicella zoster virus

A. Atopic dermatitis

You are assessing a 1-year old infant w/gastroenteritis for dehydration. What are some signs that would cause you to judge that the infant is moderately dehydrated? A. BP is normal; cap refill is 2 seconds; pulse is thready; skin turgor is decreased; fontanelle is slightly soft; & UOP < 1 mL/kg/hr B. BP 94/56; HR 120; cap refill is 3 seconds; skin rapidly snaps back to position; fontanelle is firm & slightly curves inward; & UOP is slightly decreased C. BP 70/55; HR 75; cap refill is 4 seconds; skin does not rapidly snap back to position; fontanelle is soft; & UOP < 1 mL/kg/hr D. Her BP, HR, cap refill, skin turgor, & fontanelle, are all normal but UOP is slightly decreased

A. BP is normal; cap refill is 2 seconds; pulse is thready; skin turgor is decreased; fontanelle is slightly soft; & UOP < 1 mL/kg/hr (p. 72)

Which test can be used to diagnose either Hirschsprung's Dz or intussusception? A. Barium enema B. Rectal biopsy C. Ultrasound D. Colon biopsy

A. Barium enema

Robert, a good friend of yours, has called you for advice on obtaining a medical certification. He wants to know his various options. Which statement is true regarding the difference between licensure & certification? A. Certification signifies mastery of specialized knowledge & is granted by a non-governmental agency B. Licensure signifies that a person is qualified to perform a particular role & is granted by a non-governmental agency C. Licensure signifies mastery of specialized knowledge & is granted by a governmental agency D. Certification signifies that a person is qualified to perform a particular role & is granted by a governmental agency

A. Certification signifies mastery of specialized knowledge & is granted by a non-governmental agency (p. 167)

A 5-year-old male has recently been diagnosed w/splenomegaly. Of the following, which is not usually associated with the signs & symptoms? A. Iron deficiency anemia B. Glucose-6-phosphate dehydrogenase deficiency C. Leukemia D. Sickle cell disease

A. Iron deficiency anemia (p. 133)

Jackie, an 8-year-old girl, comes to your clinic complaining of a throbbing headache on the left side of her head. Her parents state their daughter seems to be having episodes of headaches since last week. They say that Jackie has seemed disoriented several times in the past, vomited & fallen into deep sleep. Of the known migraine syndromes, which of the following does Jackie most likely have? A. Confusional migraine B. Classic migraine C. Abdominal migraine D. Common migraine

A. Confusional migraine (Confusional migraines are characterized by periods of confusion & disorientation followed by vomiting & deep sleep. Furthermore, this type of migraine is more common in younger children, such as Jackie. Abdominal migraines are also among the variant migraine syndromes but are characterized by abdominal pain & nausea along with headache, none of which are present in this scenario. "Classic" & "common" migraines are categories of migraine headaches, but, as the question calls for a type of migraine syndrome & not a generalized category, this would not be applicable.)

You are providing nutritional education to the parents of a 12 yo male who has been newly diagnosed w/DM1. All of the following statements regarding proper nutrition would be appropriate except: A. Consume no less than 2,000 calories per day B. Carbohydrates should make up the majority of the diet C. Protein should account for roughly 20% of caloric intake D. The diet s/be high in fiber

A. Consume no less than 2,000 calories per day (The caloric intake of a child w/DM1 is commonly determined by the child's weight & growth patterns. A 2,000 calorie diet may be appropriate for one child, but not for another. Carbohydrates are the main energy source & should make up the majority of daily calories. Protein, which delays the absorption of carbohydrates, should account for about 20% of daily calories. Individuals w/DM1 should generally consume about 25 grams of fiber per 1,000 calories, which equates to a high-fiber diet. High fiber diets may help to control blood sugar levels & A1C.)

Your patient has allergic conjunctivitis. Which of the following treatments would you be least likely to prescribe due to the risk of increased intraocular pressure? A. Corticosteroid eye drops B. Antihistamine eye drops C. A mast cell stabilizer D. Non-steroidal anti-inflammatory eyedrops

A. Corticosteroid eye drops (p. 100)

Claire, 6yo, comes to your clinic w/a low-grade fever & bark-like cough. Her gym teacher sent her home the day before because she seemed as if she was having difficulty catching her breath on the least exertional of activities. Her lungs, however, are clear on auscultation. Which of the following would most account for all of the patient's signs & symptoms? A. Croup B. Group A ß-hemolytic streptococcal infection C. Common cold D. Epiglottitis

A. Croup (p. 108)

A concerned mother brings her 4yo son, Kevin, to your office. The mother is worried about Kevin's weakness & clumsiness compared to other children his age as well as his abnormal posture. Further, she notes unusual movements from her son, such as moving his hands up his legs when rising stand. As you begin laboratory testing to confirm your suspected diagnosis, which of the following diagnostics would you not expect to see? A. Decreased creatine kinase B. Necrotic degenerating fibers C. Abnormal electrocardiogram D. Myopathy

A. Decreased creatine kinase (p. 121)

The NP knows that which of the following prophylactic drugs is not used in pediatric migraine mgmt? A. Doxepin B. Bisoprolol C. Ibuprofen D. Amitriptyline

A. Doxepin

Using the Dubowitz/Ballard exam to estimate the gestational age of a Caucasian newborn, you determine him to be a post-term. Which of the following is not related to the criterion included in the Dubowitz/Ballard exam? A. Elasticity of cartilage in nose B. Thickness & size of breast tissue C Hypertonic flexion of knees D. Plantar creases over sole of feet

A. Elasticity of cartilage in nose

You are taking the medical history of 11-year-old Jake, who complains of regularly been tired. His father states that he has had trouble getting through soccer practice because of muscle fatigue. You note that Jake has gained an abnormal amount of weight but hasn't grown as much as you would have expected for his age. Upon examination, you detect thick tongue. Which of the following should be tested for to verify the most likely diagnosis? A. Elevated TSH & decreased T4 B. Glucose C. Decreased TSH & elevated T3 D. Ketonemia

A. Elevated TSH & decreased T4

All of the following conditions may be present in afebrile patients except: A. Encephalitis B. Acute glaucoma C. Subarachnoid hemorrhage D. Cerebral ischemia

A. Encephalitis (p. 125)

Which of the following is one of the therapeutic treatments used in the symptomatic relief of sickle cell anemia? A. Ensure adequate oxygenation B. Neurodevelopmental monitoring C. Environmental investigation D. Referral to an oncologist

A. Ensure adequate oxygenation

You are examining a newborn who has arrived for her 1st check-up. Her trunk is covered in blotchy red spots of varying sizes, & some spots present w/white pustules over them. Which of the following is the most probable diagnosis? A. Erythema toxicum B. Milia C. Telangiectasia D. Mongolian spots

A. Erythema toxicum

Of the following, which is not the purpose of the Patient Safety and Quality Improvement Act? A. Establish a database to which healthcare providers must report errors B. Resolve patient safety and health care quality issues C. Analyze medical errors D. Establish a voluntary reporting system

A. Establish a database to which healthcare providers must report errors (The purpose of the Patient Safety and Quality Improvement Act is to establish a confidential, liability-free, voluntary database of medical errors so that patient safety organizations can examine the data to help resolve patient safety and health care quality issues.)

You consider screening Sylvester, a 9-year-old African-American boy, for DMII Sylvester presents w/a # of risk factors for DMII; he is obese & has a FHx of the condition. Based on these risk factors, you decide to begin screening the following year, when Sylvester is 10 years old. How often do you subsequently continue screening? A. Every 2 years B. No further screening is necessary C. Every 6 months D. Every year

A. Every 2 years (p. 141)

Which of the following is thought to be a predisposing psychosocial factor for ADHD? A. Family history of alcoholism B. Near-death experience from perinatal asphyxia C. Depression D. Current lack of friends

A. Family history of alcoholism

A couple brings their 4 month old child, Elena, for a regular check up. During the exam, the parents remark about Elena's calm temperament. The mother tells you that Elena "will let anybody hold her". The parents express concern that Elena may even be "too easy-going" & open to strangers. Of the following, what should you tell the parents to allay their fears? A. Fear of strangers usually develops @ about 6 months of age B. Refer to neurologist C. Elena's behavior is abnormal, as she should be afraid of strangers at her age D. Children do not usually develop separation anxiety until 2 years of age

A. Fear of strangers usually develops @ about 6 months of age

A concerned father brings his very ill child to your office. Upon evaluating the child, your assess the child is having pneumonia. Which of the following signs & symptoms would lead you to reach this conclusion? A. Fever, chills, & purulent sputum B. Moderate fever, nonproductive cough, & wheezing C. Belching, hiccoughs, and dysphasia D. Diaphoresis, hyperresonance, chest tightness, & difficulty speaking

A. Fever, chills, & purulent sputum

According to the psychosocial development model, at which age would an infant be least likely to cry when she is handed to her new babysitter? A. Four months B. Six months C. Eight months D. Ten months

A. Four months (An infant would be least likely to cry when introduced to a stranger at around 4 months, as stranger anxiety typically develops at around 6 months. Thus, the 6-month-old, the 8-month-old, & 10-month-old are more likely that the 4-month-old to have developed a fear of strangers, & would be more likely to cry when introduced to a new babysitter)

You have just administered the human papillomavirus vaccine to a 12yo pt. Regarding post vaccination mgmt, which of the following is recommended? A. Have the pt sit for @ least 15 min after vaccination B. Test for antibodies to the human papilloma virus C. Massage the injection site D. Follow up with a 2nd vaccine 2 months after the 1st dose

A. Have the pt sit for @ least 15 min after vaccination

Which of the following is dedicated to identifying disparities that prevent people from obtaining healthcare? A. Healthy People 2020 B. Collaborative practice C. Patient safety and quality improvement act D. Health insurance portability and accountability act

A. Healthy People 2020

Which of the following findings in a newborn would be least likely to indicate an abnormality? A. Heart murmurs B. White forelocks C. Wide fontanelles D. 4 junctional nevi clustered on trunk

A. Heart murmurs (Heart murmurs may present in 85-90% of all newborns; despite this high percentage, structural heart dz typically only presents in 8-10 live births out of 1000. White forelocks may indicate Waardenburg syndrome, whereas wide fontanelles may indicate hydrocephalus, Down syndrome, or hypothyroidism. Although junctional nevi may be present in newborns, groupings of the spots may signal a precursor tuberous sclerosis or generalized neurofibromatosis.)

A mother brings her adopted 6 yo to see you after he bruised his elbow bumping it on the kitchen counter. She became concerned the day after the injury b/c he said his elbow was tingling. After finding a normal CBC, you order a coagulation panel b/c you are most concerned about which disorder? A. Hemophilia B. Platelet disorder C. Anemia D. Leukemia

A. Hemophilia

A father brings his 10-year-old son in for an examination. He states that for the past 3 months, his son has been coughing all night and unable to sleep. The child is very fatigued at school & lacks the stamina to participate in sports. The father is concerned about the child's declining health. In your auscultation of the patient, you note wheezing in both lungs. Which of the following findings best directs you to asthma as the most likely cause? A. History of frequent upper respiratory infections B. Elevated temperature C. Palpable liver and spleen D. Elevated white blood count

A. History of frequent upper respiratory infections

Of the following, which eye disorder is characterized by an abrupt or sudden onset w/painful swelling on the lid margin? A. Hordeolum B. Strabismus C. Conjunctivitis D. Chalazion

A. Hordeolum

An infant present to the clinic w/angry red diaper rash w/satellite lesions. A KOH prep used to examine the lesion is negative. Your tx would consist of which of these methods? A. Hydrocortisone 1% cream B. Zinc oxide ointment C. Acyclovir 5% cream D. Nystatin 100,000 units/gram

A. Hydrocortisone 1% cream

A 15-year-old female comes to your office because she is worried that she may be pregnant. She says that she has been having menstrual irregularities & muscle cramps. She also reports that she is restless & that her hair has become fine. Regardless of her feeling "hungry all the time," you note that she has lost weight since her last checkup. Her tests indicate an elevated T3 level. What is the most likely diagnosis? A. Hyperthyroidism B. Hypothyroidism C. Type II diabetes D. Type I diabetes

A. Hyperthyroidism (p. 143)

Which of the following does not commonly impact temperature stability & regulation in a child? A. Increased subcutaneous tissue w/increased evaporative heat loss B. Decreased body surface area to mass ratio C. Thinner skin D. Increased energy expenditure

A. Increased subcutaneous tissue w/increased evaporative heat loss (Temp stability in children is commonly impacted by their limited, no increased, SQ tissue w/evaporative heat loss, as well as a smaller BSA to mass ratio, thinner skin, & increased energy expenditure. D/t the fact that more energy is needed to facilitate proper growth, less energy is available for thermoregulation. These factors are important b/c they put children @ an increased risk of hypothermia.)

In what area of the brain do childhood tumors predominately occur? A. Infratentorial region B. Supratentorial region C. Tentorium cerebelli D. Occipital lobes

A. Infratentorial region

Which are likely to cause costochondritis

A. Injury to the chest B. Physical strain D. Fibromyalgia

Joey, an 11 mo infant, is brought in by his mother b/c he's been sleeping more than usual. A PE indicates pallor, palpitations, & tachycardia. In discussing recent events w/the mother, she says that she has been alternating between breast milk & formula for Joey; when pressed for time & lacking formula, she gives him whole milk instead. After he responded well to the taste, she replaced breast milk whole cow's milk. Joey most likely has which of these dietary insufficiencies? A. Insufficient iron B. Insufficient folic acid C. Insufficient fiber D. Insufficient Vitamin D

A. Insufficient iron

You are examining a 24 mo old African-American patient whose parents complain that she is "unusually tired". She has no history of chronic illness. The patient's history reveals that she was switched to whole milk @ 12 mo of age & she is currently drinking approximately 32 oz/day. Upon physical examination, you noticed pale conjunctiva but no hepatosplenomegaly. Laboratory results indicate the following: mean corpuscular volume = 76 fL; & mean corpuscular hemoglobin concentration = 31%. What is the most likely diagnosis given this information? A. Iron deficiency anemia B. Thalassemia C. Vitamin B 12 deficiency D. Sickle cell anemia

A. Iron deficiency anemia (p. 133)

When performing an x-ray on a patient with rheumatoid arthritis which findings would expected?

A. Joint swelling B. Osteopenia C. Progressive cortical thinning D. Joint space narrowing

Of the following options, which is not a differential diagnosis for a child with obesity? A. Juvenile idiopathic scoliosis B. Prader Willi syndrome C. Endocrine disease D. Medication-induced obesity (antipsychotics)

A. Juvenile idiopathic scoliosis

A toddler who presents w/iron deficiency anemia is also @ increased risk for lead poisoning d/t pica. After moving to a house built in 1965, the mother brings the child in for venous blood level testing. While going over the results, you explain that chelation therapy is not recommended b/c the toddler's venous blood level concentrations do not reach the threshold to treat with chelation therapy. What is the toddler's lead level? A. Level 35 mcg/dl B. Level 45 mcg/dl C. Level 65 mcg/dl D. Level 75 mcg/dl

A. Level 35 mcg/dl (Chelation therapy is recommended for venous blood level concentrations that meet or exceed 45 mcg/dL. At levels below 45 mcg/dL, removing led sources from the child's environment is considered to be more effective than chelation therapy. At levels above 70 mcg/dL, hospitalization for chelation, hydration & close observation is recommended.)

Your 13-year-old patient, Teresa, complains of cold intolerance & of having recently gained weight. You note that she has thinning hair &, upon further interview, you discover that she has impaired motor coordination. She also experiences hyperactive bowel sounds & constipation. Which drug should you use to treat her condition? A. Levothyroxine B. Metformin C. Thiourea D. Propranolol

A. Levothyroxine

You were treating an infant w/a painful, protuberant abdomen as well as a severe, chronic diarrhea that results in bulky & foul stool. Additionally, her appearance suggests fatigue, pallor, & failure to thrive. Which of the following could most likely cause all of the patient's signs & symptoms? A. Malabsorption B. Hirschsprung's disease C. Appendicitis D. Intussusception

A. Malabsorption (p. 76)

A patient of yours has a tall stature. His arm span is greater than his height. He also has thin extremities with a long narrow face. When you look into his mouth, you notice that he has a high arched, narrow palate. The patient wants to know if he has some sort of growth problem. According to the findings above, you know that the patient most likely has: A. Marfan syndrome B. DiGeorge syndrome C. Turner's syndrome D. Tay-Sachs disease

A. Marfan syndrome

Alex, an 11-year-old male, comes to your clinic for a physical examination. During the exam, you notes that his spine is severely curved in the lumbar & thoracic areas. Although the diagnosis is concurrent, you also know that this finding is most consistent with which of the following genetic conditions? A. Marfan syndrome B. Down syndrome C. Turner's syndrome D. DiGeorge syndrome

A. Marfan syndrome (Kyphoscoliosis, a combination of both kyphosis & scoliosis is a common phenotypic presentation of Marfan syndrome.)

All of the following are viral causes of pharyngitis & tonsillitis except: A. Mumps virus B. Respiratory syncytial virus C. Influenza A & B D. Epstein-Barr virus

A. Mumps virus (Although the mumps virus belongs to the same family as parainfluenza, which can produce pharyngitis & tonsillitis, it produces neither condition; the virus affects the salivary glands, genitals, & nervous system, rather than the throat & pharynx. RSV, EBV, & Influenza A & B are all viral causes of pharyngitis & tonsillitis.)

The term "pervasive developmental disorders" refers to a group of conditions that involve delays in the development of many basic skills. Children with these conditions often are confused in their thinking & generally have problems understanding the world around them. Which of the following is a factor leading to this condition? A. No known factor B. Failure of a parent to respond to their infant's cues C. Lack of maternal infant bonding D. An X-chromosome break

A. No known factor (Until more is known about the causes of pervasive developmental disorders, it is not possible to predict or prevent them. However, this is a child symptoms begin treatment, the better he or she will do it in the long run as early diagnosis & treatment improves outcomes.)

Following a routine, natural birth, you utilize the common appearance, pulse, grimace, activity, & respiration (APGAR) scoring method to determine the overall health of Mabel, a newborn African-American baby. You notice overall normal skin color, but there is a bluish tint on her hands & feet. With regards to her appearance, what is the appropriate Apgar score for Mabel? A. One B. Three C. Zero D. Two

A. One (p. 26)

A 5 yo is brought to your clinic by his mother b/c she is concerned about the multiple bruises on his extremities & back. The boy's mother also states that he looks pale. During your PE, you detect scattered, enlarged lymph nodes. Which of the following actions are you most likely to do? A. Order a peripheral smear B. Refer the patient to a hematologist-oncologist C. Do a bone marrow aspiration & biopsy D. Consult a child protective agency

A. Order a peripheral smear (A peripheral smear s/be ordered to detect malignant cells. If malignant cells are detected, the next step would be a referral to a hematologist-oncologist who would likely perform a bone marrow aspiration & biopsy. Although abuse must be considered, especially w/bruises on the back, enlarge lymph nodes would not be expected in suspected cases of child abuse.)

When performing an x-ray on a patient with osteoarthritis which of the following findings would be expected?

A. Osteophytes B. Narrowing of joint space C. Subchondral bone D. Juxta-articular sclerosis

A 17 yo obese female undergoes a diabetes screening every 2 years. Which set of risk factors r/t the patient's heritage & MHx would best justify this screening? A. Pacific Islander w/polycystic ovarian dz B. Caucasian w/HTN C. Hispanic w/dysmenorrhea D. Native American w/hypotension

A. Pacific Islander w/polycystic ovarian dz (For individuals w/obesity & @ least 2 risk factors, screening for DM should typically be done q2y, beginning around the onset of puberty. 1 risk factor is being Asian/Pacific Islander, AA, Native American, or Hispanic. Signs of insulin resistance, evidenced by polycystic ovarian dz, acanthosis nigricans, HTN, or dyslipidemia, are also risk factors. Caucasians are not generally @ an increased risk of developing DM. Hypotension & dysmenorrhea are also not considered to be risk factors for DM.)

A 7 yo boy, Clark, presents w/a fever of 103. You have seen Clark for WCEs, as well as acute conditions, since he was 2 years old. You know that he has a history of multiple strep infections. Upon your PE, you note the presence of a red skin rash, & lab tests reveal an elevated erythrocyte sedimentation rate. Which of the following is the best treatment plan for the suspected condition? A. Penicillin B. Corticosteroids C. Intravenous gamma globulin D. Acyclovir

A. Penicillin

Which of the following is most likely to be c/by Chlamydia trachomatis? A. Pharyngitis B. Croup C. Infections mononucleosis D. Epiglottitis

A. Pharyngitis

Which of the following WCE screening test is usually performed between 4-6 years of age? A. Purified protein derivative B. Lead screening questionnaires C. Hematocrit D. Cholesterol

A. Purified protein derivative (p. 39)

While assessing a newborn, which of the following should cause you to consider coarctation of the aorta? A. Radio-femoral pulses delay B. Expiratory grunting C. Subcostal and intercostal retractions D. Delayed capillary refill

A. Radio-femoral pulses delay

A 6 yo male presents to your office w/a limp & complaints of pain in his groin. Upon further examination, you notice stiffness in the hip area on the affected side, as well as significant muscle tone loss in the upper thigh d/t limited motion. Which of the following diagnostics would be most helpful in confirming Legg-Calve-Perthes Dz? A. Radiograph B. Ultrasound C. Muscle biopsy D. Electromyography

A. Radiograph

For pts w/inadequate iron intake, most of what composes a pt's blood will be lower than normal in lab findings. What aspect of blood, however, may actually be found to increase? A. Red cell distribution width (RDW) B. Mean corpuscular volume (MCV) C. Mean corpuscular hemoglobin concentration (MCHC) D. Serum iron

A. Red cell distribution width (RDW) (Red cell distribution width (RDW) is increased as RBC may change in shape & size. Serum iron, mean corpuscular volume (MCV), mean corpuscular hemoglobin concentration (MCHC) will all be lower than normal in their lab results d/t lack of iron in the blood.)

Margaret, an 11 yo female diagnosed w/DM1 is experiencing hypoglycemia @ 3AM, & elevated blood sugar @ 7AM. What is the proper treatment for her? A. Reduce of eliminate the dose of insulin before bed B. Increase the dose of metformin before bed C. Add or increase the dose of insulin before bed D. Advise a snack before bed

A. Reduce of eliminate the dose of insulin before bed

Your pt has significant edema & ecchymoses around the elbow. After ordering a radiograph, you notice a fat pad sign on the x-ray but no visible fracture. What would be the best course of action to take regarding treatment? A. Refer the pt to ortho B. Administer NSAIDs C. Employ the RICE method D. Supinate the arm to correct subluxation

A. Refer the pt to ortho

During a physical examination, a newborn is found to have length, weight, & head circumference all at < 10th percentile for her age. All of the following factors should likely be considered as potential sources for her condition except: A. Residing at high altitudes B. Bacterial intrauterine infection C. Inborn errors of metabolism D. Congenital or chromosomal abnormalities

A. Residing at high altitudes (Symmetric intrauterine growth retardation (IUGR) presents w/length, weight, & head circumference < 10th percentile, whereas asymmetric IUGR presents w/weight < 10th percentile but length & head circumference w/in normal range; residing at high altitudes is more likely to produce asymmetric IUGR, d/t decreased O2 available to the fetus, than symmetric IUGR. Bacterial intrauterine infection, inborn errors of metabolism, & congenital or chromosomal abnormalities are known factors that may contribute to symmetric IUGR.)

Which of the following conditions is caused by an RNA virus? A. Rubella B. Erythema infectiosum C. Roseola infantum D. Red measles

A. Rubella (p. 95)

Of the following, which viral infection most commonly present w/Koplik spots? A. Rubeola B. Roseola infantum C. Rubella D. Lyme Dz

A. Rubeola

You are examining an infant w/red-brown vesiculopapular lesions & curved burrows on the head, neck, palms, & soles. The infant appears to have sensations of intense itching. Which of the following dermatologic conditions is the most likely cause of the infant's findings? A. Scabies B. Molluscum contagiosum C. Varicella zoster D. Atopic dermatitis

A. Scabies (p. 92)

Which of the following categories of burns often present as being moist? A. Second B. First & second C. First D. Second & third

A. Second (p. 80)

Which of the following are 2 psychosocial developments that are associated primarily w/adolescents? A. Sense of identity & narcissism B. Narcissism & desire to please adult figures C. Sense of identity & development of self-esteem D. Development of self-esteem & desire to please adult figures

A. Sense of identity & narcissism

When implementing the Guidelines for Adolescent Preventive Services in a WCE with a 16 yr old female, which piece of information would you be most likely to share with her parent? A. She mentions that she is depressed & wonders if life is worth living B. She had unprotected sex w/her boyfriend last month & is late for her period C. She has 3 best girlfriends & they have all started experimenting with marijuana D. She thinks her mother's new boyfriend is creepy

A. She mentions that she is depressed & wonders if life is worth living (Confidentiality s/be assured when implementing GAPS but the NP is expected to disclose information if there is a concern about a patient hurting herself or someone else.)

DEXA is -2.7. The best course of action at this time would be to recommend which of these therapies?

Alendronate

A 16yo male, w/no abnormalities in his medical records, presents w/ unusual body proportions & underdeveloped sexual characteristics. A blood test indicates Klinefelter syndrome. Which of the following is not a typical feature of Klinefelter syndrome? A. Short stature B. Learning disability C. Gynecomastia D. Hypogonadism

A. Short stature (Klinefelter syndrome is a genetic disorder involving only males w/an extra X-chromosome. This disorder is known to cause tall rather than short stature. S/S include hypogonadism, learning disabilities, & gynecomastia. Other manifestations include a simian crease & personality impairment. short stature is not a finding of Klinefelter syndrome; rather it is usually present in those w/Turner syndrome.)

You are examining a 2 1/2 mo infant named Jessica. Which of the following reflexes would not expect Jessica to display? A. Stepping reflex B. Fanning of the toes when bottom of foot is stroked C. Flinging arms to the sides when startled D. Tonic neck reflex

A. Stepping reflex

What is the most common etiology of pneumonia in children 1-3 yo? A. Streptococcus pneumoniae B. Mycoplasma C. Group B Streptococcus D. Haemophilus influenzae

A. Streptococcus pneumoniae

Kasey, age 7 months, is bought to the clinic by her concerned parents. They have been talking to other parents in their parenting group & need reassurance that Kasey is keeping up developmentally. As you observe Kasey, you notice that she responds to her name, consistently babbles, crawls around on the floor, & is able to pick up objects. Which of the following additional milestones would also be expected in a child her age? A. Supports weight on feet B. Holds head steady C. Equal coordination of hands D. Plays independently

A. Supports weight on feet (A child exhibiting the milestones of a 6-9 month old, as evidenced by crawling, babbling, picking up objects, & responding to her name, is likely to be able to support her weight on her feet. The ability to hold her head steady by 2-5 months; play independently & exhibit equal coordination in her hands by 10-12 months.)

Which of the following genetic disorders is most found in the Ashkenazic Jewish population? A. Tay-Sachs disease B. Marfan's syndrome C. XXY syndrome D. Trisomy 21

A. Tay-Sachs disease (p.23)

An Ashkenazic Jewish couple comes to your office with their 8 month old boy. Their child, Ezra, was healthy at birth, but, in the last few weeks, he appeared to be losing his eyesight and hearing. He does not seem to recognize his parents, & he reaches out wildly & without purpose. The baby also does not turn his head when called or respond to sounds. You note that the baby has low muscle tone & seems listless. Which of the following should be a part of your ddx? A. Tay-Sachs dz B. DiGeorge syndrome C. Turner syndrome D. Marfan syndrom

A. Tay-Sachs dz

You were reading a peer-reviewed journal article about predictors of birth length. In 95% of identical experiments, neonates measure 49.2 cm @ birth, plus/minus .05 cm. The value 49.2 plus/minus .05 is known by what statistical term? A. The confidence interval B. The correlation C. The level of significance D. The standard deviation

A. The confidence interval (p. 169)

Which of the following findings would most strongly indicate that a febrile infant is moderately ill? A. The infant has a temperature of 101.8 but smiles often. B. The infant is fussy but calms quickly when offered support. C. The infant appears listless & doesn't feed well. D. The infant has a temperature of 103 but appears alert & active.

A. The infant has a temperature of 101.8 but smiles often. (Moderately ill infants typically display irritable or fussy behavior & have a fever below 102, yet are easy to console, may smile, & continue to feed normally. Infants that present w/ fever above 100.4 but smile, feed normally, & appear alert & active are typically classified as mildly ill. Severely ill infants normally present w/fever > 104, these infants often appear listless, may not feed at all or feed poorly, & are typically recommended for hospital admission.)

Which of the following findings would most strongly indicate that a febrile infant is moderately ill? A. The infant has temp of 101.8 F but smiles often B. The infant is fussy but calms quickly when offered support C. The infant appears listless & does not feed well D. The infant has temp of 103 F but appears alert & active

A. The infant has temp of 101.8 F but smiles often (Moderately ill infants typically display irritable or fussy behavior & have a fever < 102 F, yet are easy to console, may smile, & continue to feed normally. Infants that present w/temp > 100.4 but smiles, feed normally, & appear alert & active are typically classified as mildly ill. Severely ill infants normally present w/ temp > 104, these infants often appear listless, may not feed at all or feed poorly, & are typically recommended for hospital admission)

A NP friend of yours comes to you worried because her office has been receiving a total reimbursement of the Physician Fee Schedule for her services rendered even though she assigned her billing rights to the office. she thinks there may be fraudulent billing & asks for your advice. You tell her that there is nothing to worry about because: A. The office most likely billed under the physician's Medicare provider code. B. Medicare recently changed the rules to allow for total reimbursement of the Physician Fee Schedule for NPs C. Assigned billing rights allows for total reimbursement of the Physician Fee Schedule. D. Legislators have finally recognized the invaluable services provided by NPs & added provisions to reimburse NPs 100% of the Physician Fee Schedule

A. The office most likely billed under the physician's Medicare provider code. (Medicare payments reimburse 85% of the Physician Fee Schedule if the NP directly bills Medicare w/his/her Medicare provider number; however, 100% of the Physician Fee Schedule rate may be reimbursed for services rendered, including those provided by an NP, if the supervising physician provides his/her Medicare provider number for billing purposes. Assigning billing rights to the office does not satisfy the necessary conditions for 100% reimbursement. Currently, legislators & Medicare still allow for only 85% reimbursement of the Physician Fee Schedule to NPs.)

You are treating a overweight 7 yo Hispanic female diagnosed w/HTN. She has a FHx of DM2. Which of the following is true regarding risk factors & screening for DM2 for this patient? A. The patient should not be screened until age 10 or onset of puberty B. The patient s/b screened ASAP d/t her HTN C. The patient should have been screening before the age of 7 D. The patient s/b screened ASAP d/t risk factors r/t her ethnicity

A. The patient should not be screened until age 10 or onset of puberty

Management of symptoms in infants w/GERD is more likely to include which of these in non-severe cases? A. Thickening of feedings B. Histamine 2-receptor antagonist C. A diet that either reduces or eliminates protein D. Switching to hydrolyzed or amino acid-based formula

A. Thickening of feedings

Which asthma pt would you be most likely to treat w/a short course of oral systemic steroids? A. Tinh, 6yo female, whose ratio of FEV1/FVC is 79% B. Nathan, 7yo male, who uses his short-acting beta agonist 3 times/week for symptom control C. Mateo, 3yo male, whose nighttime symptoms wake him up an average of 2 times/month D. Brigid, 10yo female, whose predicted FEV1 is 82%

A. Tinh, 6yo female, whose ration of FEV1/FVC is 79% *A short course of oral systemic corticosteroids should be considered for pediatric pts w/moderate persistent or severe persistent asthma; therefore, a 6yo child whose ratio of FEV1/FVC capacity is 79%, which is an indicator of moderate persistent asthma in this age range, is likely to be prescribed oral systemic corticosteroids. Children w/mild persistent asthma are likely to be prescribed a low-dose ICS. A 7yo who uses a SABA for symptom control > 2 d/wk but not daily has mild persistent asthma. A 3yo who experiences nighttime awakenings 1-2 times/mo also has mild persistent asthma. A 10yo whose predicted FEV1 is 82% would be classified as having mild persistent asthma as well.)

What is the purpose of case management (CM)? A. To mobilize, monitor, & control resources used by patients during illness B. To establish the qualification & mastery of skills of NPs C. To review health care cases so as to improve patient safety D. A management process of monitoring, evaluating, continuous review, & improving quality in health care

A. To mobilize, monitor, & control resources used by patients during illness (CM is a comprehensive & systematic approach to providing quality health care that balances the quality & cost of resources used by the pt during the course of his/her illness. A management process of monitoring, evaluating, continuous review, & improving quality in health care is quality improvement, while establishing the qualification & mastery of skills of NPs is the purpose of certification. Reviewing health care cases so as to improve patient safety is one of the objectives of the The Patient Safety & Quality Improvement Act (PSQIA).)

Which of the following is not true regarding breast-feeding? A. Transfers the daily requirement of vitamin D to the infant B. Decreased illnesses in the infant C. Positively affects the baby's immune system D. Unique nutritional source

A. Transfers the daily requirement of vitamin D to the infant (p. 6 - 400IU/day @ 2 months thru the lifespan)

If a baby has a heart murmur similar to those heard in a patient w/a ventricular septal defect & his echocardiogram indicates right ventricular hypertrophy, what would be the most likely diagnosis? A. Transposition of the great arteries B. Tetralogy of Fallot C. Atrial septal defect D. Patent ductus arteriosus

A. Transposition of the great arteries (p. 65)

Angela brings in her son, Jonathan, a 5 mo old infant. He has been scratching himself constantly, especially now that it is winter. A physical examination indicates very small, firm, pink to flesh-colored discrete papules on his trunk & face. Which of the following is the best treatment specifically for the patient's lesions? A. Tretinoin B. Hydrocortisone C. Triamcinolone D. Desonide

A. Tretinoin (p. 87 Mulluscum Contagiosum)

Which of the following is most likely if you have a patient w/a webbed neck, lymphedema, & low hairline, w/hypertension & learning disabilities? A. Turner's B. Marfan C. Klinefelter's D. DiGeorge

A. Turner's (p. 23)

Which type of diet practiced by adolescents warrants the closest monitoring by parents? A. Vegetarian B. High-protein C. Lactose-free D. Pescetarian

A. Vegetarian (This diet may not provide all nutritional requirements. Alternative sources of protein, fatty acids, calcium, zinc, iron, & vitamins B12 & D may need to be incorporated into the diet.)

If a newborn's mother is HBsAg-positive, when should the NP order the hepatitis B vaccine for the newborn? A. Within 12 hours after birth B. Prior to home discharge C. One week after birth D. Not until 1 year of age

A. Within 12 hours after birth (If mom is HBsAg-positive —give Hep B vax w/in 12 hours of birth. If the mom is a hepatitis B carrier the new born would still need the vax shortly after birth. If mom tests positive and her status was previously unknown — give hepatitis B immune globulin no later than 1 week of age.)

Ms. Swanson, age 60, presents with complaints of constipation and "a painful cramp" in her lower left side. She tells you that she has also been "feeling nauseated and even there up." During the practitioner's physical exam, it is noted that the patient has a low-grade fever, tenderness in the left lower quadrant, and slight abdominal distention. After referring the patient to the emergency room, the nurse practitioner knows that which test is a priority at this time?

Abdominal x-ray

A student asks what factors could place people at high risk of developing alcoholism. You would discuss all of the following factors except: a. Lower level of psycho social development b. Abnormal protein in the brain. c. An alcoholic grandparent d. Low self-esteem

Abnormal protein in the brain (Alzheimer's)

Which of the following drug treaties is associated with rare complication of osteonecrisos of the jaw, especially in patients receiving cancer treatments.?

Alendronate (Fosamax)

Jordan, a 45 year old male, comes to the clinic with a dry cough, weight loss, night sweats, and fatigue. A chest x-ray reveals small, homogeneous infiltrates in the upper love of the right lung. Give the most likely condition, which of the following tests are you most interested in ordering to confirm the most likely diagnosis? a. Complete blood count b. Acid-fast bacilli smear c. Liver function d. Serum creatinine

Acid-fast bacilli smear

Your patient has just returned from competing in a soccer tournament in Denver. He complains of pain in his legs, nausea, tiredness, having trouble concentrating in school, and generally feeling unwell. The patient's history is positive for sickle cell trait. You order baseline studies to assess the patient. Which finding would you most expect to see in your patient?

Acidosis

Which of the following adverse effects is not typically caused by high amounts of estrogen? a. Nausea b. Acne c. Edema d. Breast tenderness

Acne

Which condition will the nurse practitioner monitor for in a geriatric patient taking hydrochlorothiazide and nifedipine (Procardia XL)?

Acquired neutropenia (Hydrochlorothiazide can cause bone marrow suppression)

The parents of a 16-year-old patient are concerned that their child may be depressed. The parents state that their child has increasingly isolated himself and has an overall sad demeanor. While interviewing the child, which of the following guidelines for effective communication is necessary?

Active listening skills and therapeutic response

Most common pediatric leukemia

Acute Lymphocytic leukemia

A 30-year-old female presents with a cough and headache. She reports that she also has been wheezing for nearly a full week. Upon examination, you note a low grade fever and bilateral rhonchi; however, after coughing, her lungs are relatively clear to auscultation. Of the following, which is the most likely diagnosis?

Acute bronchitis

Paroxysms of dry and severe cough that interrupts sleep. Cough: dry to productive. Light-colored sputum. Can last up to 4-6 weeks. No antibiotics. Treat symptoms.

Acute bronchitis

A 35 year old male is presenting with complaints of fatigue and fever. No sign of infection is present Diagnostic tests show decreased red and white blood cells, as well as the presence of circulating blast cells. Which of the following is the most likely type of cancer based on these findings?

Acute lymphoctic leukemia

All of the following statements regarding leukemia in adult patients are true except: A. Leukemia more frequently occurs in males B. Acute myelogenous leukemia constitutes 80% of leukemia in adults C. Acute lymphocytic leukemia is more difficult to cure in adults than in children D. The median survival for chronic myelogenous leukemia is 3 to 4 years.

Acute myelogenous leukemia constitutes 80% of leukemia in adults (of all acute)

A 45-year-old patient with nausea and a fever of 104.4 F. He said the fever appeared earlier in the day, and that he has vomited more than once prior to arriving at your office. During the examination, he states that he feels dizzy and lightheaded whenever he stands up. His vitals indicate that he has low blood pressure. Based on these findings, which of these is the most likely diagnosis?

Addison's disease

A 50-year-old woman of Irish descent presents with history of lethargy, feeling weak, nausea, anorexia with diarrhea and abdominal pain. The woman's' skin appears tanned with hyperpigmentation of the nipple area, the gums and the lips. The electrolyte panel reveals hyperkalemia and hyponatremia. She reports craving salty foods. Which of the following is most likely?

Addison's disease

Jonathan, a 49-year-old-patient complains of frequent dizzy spells. His lab tests yield an elevated erythrocyte sedimentation rate, normal iodine uptake, normal thyroid stimulating hormone levels and low plasma cortisol. In addition, you note the appearance of hyperpigmentation on the patient's skin, particularily in skin creases. What would be the most likely diagnosis?

Addison's disease

Which condition involves damage to the adrenal glands includes metastatic cancer as a possible etiology?

Addison's disease

A mother has just given birth to her infant daughter. The mother's chart indicates that she is positive for HBsAg. Regarding hepatitis B vaccination, which of the following is the correct action to take initially?

Administer the hepatitis B vaccine and 0.5 mL hepatitis B immune globulin within 12 hours.

For a patient diagnosed with bacterial acute otitis media, which of these medications is usually recommended for first-line treatment?

Amoxicillin

A couple brings their 3-year-old boy to you for evaluation because he has been irritable and feverish. Three days prior, they brought him in and were sent home with instructions to give the child acetaminophen. However, his temperature has remained at or slightly above 102° F. The parents are also worried about skin rashes found on the child's trunk and his reddened palms and soles, and point out how red the child's eyes are "from all the crying." Your exam reveals a whitish coat on the child's tongue and very chapped lips. You also see that the skin of the palms and soles is starting to peel. Which of the following is the most appropriate treatment plan for this child's likely condition?

Admit to hospital for intravenous gamma globulin and high dose aspirin

Andre, an 89-year-old male, enters the hospital with pneumonia. Of the choices below, which details the best way and time to deal with an advanced directive?

Advise him as soon as he enters the hospital

You are treating a 55-year old male who is at risk of developing diabetes. When making recommendations about diet, what should you tell the patient about carbohydrate intake?

Advise to take 55-60% of carbohydrate caloric intake

Doug, a 58-year-old male, has recently been diagnosed with cluster headaches. He had previously assumed that he was experiencing migraines and wants to know what the two types of headaches have in common. You tell him that cluster headaches and migraines may share which precipitating factor?

Alcohol ingestion

Which of the following diagnostics is least likely to indicate a diagnosis of rheumatic fever? a. Electrocardiogram b. Aldosterone level c. Echocardiogram d. Throat culture

Aldosterone level

Susan, age 25, presents to the clinic with an altered level of consciousness. Kussmaul's breathing, polyuria, and fruity breath. Upon looking at her medical file you note she has type 1 diabetes. You immediately direct the patient to an emergency room for treatment and further evaluation. Which lab results would you LEAST expect Susan to have? Select one: a. Hyperglycemia b. Ketonuria c. Alkalosis d. Hyperkalemia

Alkalosis (Acidosis)

During a physical exam, Brett mentions that he wants to quit his job but is worried about losing health insurance. You inform him that the Health Insurance Portability and Accountability Act (HIPAA) will provide protection for workers when they change or lose their jobs. This includes certain private information that pertains to Brett's experience with the healthcare system. What is not true about HIPAA?

All sharing of a patient's information requires written authorization.

Which of the following is not a typical advantage of contraceptive rings? a. Alleviation of depression symptoms b. Lighter menstrual periods c. Fewer mood swings than oral contraceptives d. Decreased menstrual cramps

Alleviation of depression symptoms

Short term memory loss is typically an early sign of

Alzheimer's disease

Iris, a 32 year old patient, presents with fever, shaking chills, and malaise. A physical examination reveals lung consolidation, purulent sputum, and an increased fremitus. Further diagnostic evaluation shows a low white blood cell count and infiltrates on a chest x-ray. Give the most likely diagnosis, which medication would be best suited for treating Iris's condition?

Amoxicillin or Doxycycline or azithromycin

What scenario most effectively demonstrates a care of negligence?

An NP waits to administer anticonvulsants to a patient experience multiple prolonged unprovoked seizures, in order to see if they stop on their own.

With regard to S3 and S4 heart sounds, which statement is true?

An S3 sounds like "Kentucky" and is expected during pregnancy but otherwise, it is not a normal finding in the adult.

A patient comes to the clinic with complaints of weakness, fatigue, and difficulty breathing during mild exercise. His lab results indicate a low serum iron level, a low total iron binding capacity level, and a low hemoglobin level. However, his mean corpuscular hemoglobin count and mean corpuscular volume are both normal. What condition would the patient's symptoms and lab results most strongly indicate?

Anemia of chronic disease

Naomi, a 69 year old female presents to your clinic with complaints of fatigue, weakness, and breathlessness when working out. Her medical history reveals that she has renal failure. You order a blood test and the panel shows that her serum iron is low and her serum ferritin is elevated. Which of the following conditions is the most likely diagnosis?

Anemia of chronic disease

Peter, a 55-year-old Caucasian man, has a consistent blood pressure reading of 145/90 mmHg. His medical history reveals he was diagnosed with diabetes a few years prior. Which of the following would most likely be the drug class of choice for Peter at this time?

Angiotensin-converting enzyme inhibitors

The practitioner is considering antihypertensive therapy for a patient. Which statement is most accurate?

Angiotensin-converting enzyme(ACE) inhibitors may cause vasodilation, rash, taste disturbances, hyperkalemia, and renal impairment.

A patient complains of insomnia, indecisiveness, and fatigue. His physical examination indicates that the has lost a considerable amount of weight since his last visit 2 months prior. Which signs or symptoms would best support depression in his case?

Anhedonia

A 27-year old female presents with a ring-shaped skin lesion with a central clearing on her left leg. The skin lesion would most accurately be charted as?

Annular

Which of the following should you most strongly recommend the patient avoid for optimal absorption of the oral ferrous sulfate?

Antacids and tea

In order to correctly perform the pelvic rock test, the practitioner must place his or her hands on the:

Anterior superior iliac spine.

A patient presents to the clinic with the following lesion. Based on the presentation, which is the most likely diagnosis?

Anthrax

Which of the following is the screening test for hepatitis C virus (HCV)?

Anti-HCV

A 56-year-old patient comes to your clinic complaining of pain and pressure over his face, reaching as far back as his teeth. He also has a fever of 102°F, which he has been treating with rest and fluids. When asked when these symptoms occured, the patient said, "about three days ago." Which of the following is the most effective way to treat this patient's complaints? Antibiotics Diazepam for the pain Over-the-counter oral decongestants Over-the-counter antihistamine

Antibiotics

You are discussing treatment options for a patient with severe case irritable bowel syndrome. What three medications would you most strongly consider to help manage this condition?

Anticholinergic agents, antidiarrheal agents, and antidepressant agents

A thin patient with a slight build presents with constant difficulty breathing and clear mucus. A physical exam also indicates an increased chest anteroposterior diameter and hyperresonance on percussion. Given the most likely diagnosis, which class of medications is best suited for long-term treatment?

Anticholinergics (ipratropium bromide)

A patient has just been diagnosed with multiple sclerosis. Your colleague is informing her about the different forms of management and mentions beta interferons, immunosuppressive, and plasmapheresis as viable methods. Which is not a method of managing this disorder?

Anticoagulants

You are seeing a 64 year old female whose complaints include ramping in he lower abdomen that is only relieved by defection. She says that the pain "tends to come and go" and that she often experiences heartburn and fatigue. She adds that her bowel habit sometime alternate between constipation and diarrhea. As you perform the examination, she tells you that she has been working long hours recently to meet an approaching deadline. Based on these findings, which of the following would you most likely suspect as a cause of the patients's most likely condition?

Anxiety

Which of the following is used to confirm a diagnosis of Hashimoto's thyroiditis? a. Serum TSH b. Free T4 c. Antimicrosomal antibody test d. Any of the above

Any of the above

A 44 year old male sent to the hospital for an STD that is characterized by hemiparesis, hemiplegia, cardiac insufficency. Given the most likely stage of tertiary syphilis. Which of the following signs and symptoms would you likely expect?

Aortic aneurysm, leukoplakia, meningitis.

During a routine physical for Leonard, age 56, you note a diastolic murmur. The murmur presents with a blowing at the second left intercostal space. He most likely has which valvular disease?

Aortic regurgitation

Which of the following is not an expected cause of Cushing's syndrome? a. Chronic adminstration of glucocorticoids b. Adrenocorticotropic hormone hypersecretion c. Autoimmune destruction of the adrenal gland d. Adrenal tumors

Autoimmune destruction of the adrenal gland

An older adult patient presents for a physical exam. The patient has a history of hypertension and gastrointestinal bleeding. An irregular heart rhythm is auscultated during the exam, with EKG confirmed atrial fibrillation (AF). The patient's bleeding risk assessment and HAS-BLED score is 1, and the CHA2DS2-VASc score is 4. After a review of risks and benefits of anticoagulation, which should the nurse practitioner consider?

Apixaban (Eliquis)

What test usually assesses for medial or lateral collateral ligament damage or meniscus injury?

Apley's grind test

You are evaluating a 24 year old patient who is being seen for severe abdominal pain and nausea. He says a mild pain originated around his belly button earlier today, and that it has "just gotten worse". For which condition should you assess the patient, and what finding would you most likely expect to see in this patient that would further indicate this condition?

Appendicitis; the patient reports pain with internal rotation of flexed right thigh (Obturator)

Irritable bowel syndrome is a clinical syndrome with uncertain etiology that affects what percentage of the general population?

Approximately 10%-12%

What is the traditionally recommended morning dose of a conventional split-dose mixture for type 1 DM patients?

Approximately 2/3 NPH, 1/3 regular insulin

Kyle, a 32 year old male, is HIV-positive. When initiating a TB skin test, which result would show the minimal amount of elevation that would indicate he is positive for TB?

Approximately 5 mm

In the United States, type 2 diabetes comprises approximately what percentage of diabetic cases?

Approximately 90%

Israel, age 55, presents with chest pain that is exacerbated by exercise but relieved by rest within several minutes. The patient expresses the pain he feels by clenching a fist to his chest. The nurse practitioner suspects angina. An electrocardiogram (ECG) returns normal findings. Which of the following is most true?

Approximately half of all patients with angina present with normal ECG findings, so an exercise test should be conducted for more definitive results.

Your patient complains that his eyes "just aren't the same color anymore." What is the most likely diagnosis, given the picture?

Arcus senilis

A patient is using the NuvaRing as a contraceptive and asks you how long she is allowed to have the ring in her vagina at one time.

As long as 21 days

You are seeing a patient who was diagnosed with hyperthyroidism and had treatment for thyroid storm. You advise the patient that which of the following should be avoided to decrease the risk of developing another thyroid crisis?

Aspirin

Which three statements are accurate concerning a meta-analysis?

Assesses clinical effectiveness of health care interventions. Provides a precise estimate of the treatment effect. Provides highest level of evidence because of statistical analysis and integration of many studie

You are prescribing a patient an initial trial of H2 receptor blockers for management of peptic ulcer disease. When should these be orders?

At bedtime

A 19 year old patient is using a cervical cap as a contraceptive. She asks you how long must the cervical cap be left in the vagina following intercourse. You should tell her which of the following?

At least 6 hours

Julia, a 19 year old female, comes to the clinic with a cough, headache, sore throat, and excessive sweating. After ordering some diagnostic measures, you find an elevated white blood cell count and infiltrates in the lungs. Which condition is Julia most likely experiencing?

Atypical pneumonia

CXR reveals lobar infiltrates. Gradual onset. Low-grade fever. Headache, sore throat, cough, wheezing, rash (sometimes). CXR: interstitial to patchy infiltrates.

Atypical pneumonia No. 1: Mycoplasma pneumoniae

A adult student is seen in the school health clinic with complaints of a hacking nonproductive cough, rhinorrhea, pharyngitis, and malaise for the past 2 weeks. He does not take any medications, denies any allergies, and has no significant medical history. Physical examination reveals a low-grade temperature of 99.9°F, respirations of 16 breaths/min, pulse of 90 beats/min, and scattered rales and wheezing of the lungs. The patient does not appear toxic. The total white blood cell count is 10,500/µL. Which diagnosis is most likely?

Atypical pneumonia. The most common organism causing community-acquired atypical pneumonia is Mycoplasma pneumoniae,

Which of the following best describes a II/VI heart murmur?

Audible but faint

Autism in children is generally identified by 3 years of age. Alice is 30 months of age, and her mother mentions that she is not playing with others at preschool, is unable to pretend play, and has a very limited vocabulary. Her only interest is stacking blocks, at which she is very good. When considering the disorders in the spectrum of pervasive development disorders, which of the following should be your initial consideration in your differential diagnosis?

Autism

A patient has been referred to your practice by an optometrist, who found Lisch nodules in the patient's irises. Which finding would best help to confirm the suspected diagnosis of neurofibromatosis type I ?

Axillary freckling

What would be the correct dose of influenza vaccine for children 6 - 35 months of age? A. 0.15 ML B. 0.25 ML C. 0.10 ML D. 0.20 ML

B. 0.25 ML (p. 18)

What is the maximum APGAR score that can be given to a newborn baby upon assessment? A. 5 B. 10 C. 20 D. 12

B. 10 (p. 25)

By what age would a lack of awareness of cars & animals be 1st considered a developmental delay? A. 1 year B. 2 years C. 3 years D. 4 years

B. 2 years

What is the average age by which toddlers achieve daytime control of bowel & bladder movements? A. 18 months B. 2 years C. 2.5 years D. 3 years

B. 2 years (The age when they are physiologically & psychologically ready for toilet training is 18 months, they are able to control it by 2 years but often are not completely trained. Nighttime control is about 3 years or approx 1 year after daytime control is achieved.)

At what age does taking a blood pressure reading usually become a part of the WCE? A. 2 years B. 3 years C. 4 years D. 5 years

B. 3 years

A parent wants to know when the best time is to give the medicine being prescribed for ADHD. Your instructions are based on your knowledge that the earliest onset of action after the infection of methylphenidate in children is usually: A. 15 minutes B. 30 minutes C. 45 minutes D. 90 minutes

B. 30 minutes

Sam, a toddler, arrives at the clinic to see the NP for a regular check up. The NP asks his name, to which the boy replies, "My name is Sam!". Next, the NP asks Sam whether he is a boy or a girl, and Sam says, "I'm a boy!" The NP then asks Sam what he had for breakfast, & Sam says, "I had Fruit Loops with lots of milk, eggs, & juice, my favorite!" Finally, when the NP asks the boy to sing a nursery rhyme with her, he cannot. Assuming that Sam is on track in his language & communication development, what would you estimate Sam's age to be? A. 4yo B. 3yo C. 5yo D. 1yo

B. 3yo (If Sam is developing at an average pace, then he is most likely 3yo. At this age, one would be able to follow instructions w/2 or 3 steps & say one's name, age, & sex. Additionally a 3yo child should be able to hold a conversation using 2-3 sentences. At 4yo, a child would command greater communication skills & know basic rules of grammar, such as correctly using "he" & "she". A 4yo child should also be able to sing a song or say a poem such as "Itsy-Bitsy Spider". A 5yo child should have a greater grasp of language. He/she should be able to speak clearly and tell a simple story using full sentences. A 5yo child should be able to use future tense & repeat their name & address as well. A 1yo child, however, would have a much smaller grasp of language than a 3yo child. He/she would respond only to simple spoken requests & use simple gestures like shaking the head, waving goodbye, or saying "mama".)

Between what ages should typical pediatric patients 1st receive a purified protein derivative test for tuberculosis? A. 2-4 years B. 4-6 years C. 6-8 years D. 8-10 years

B. 4-6 years

The Bayley-III test is the standard test for the diagnosis of developmental delays in children until what age? A. 48 months B. 42 months C. 36 months D. 24 months

B. 42 months (p. 42)

You are examining a smiling & cooing baby who grabs the stethoscope from around your neck w/1 hand & transfers it to her other hand. Which of the following age ranges is typically when babies develop or begin to develop disability? A. 4 - 5 months B. 5 - 6 months C. 3 - 4 months D. 2 - 3 months

B. 5 - 6 months (p. 11)

Holly, age 4, is at a WCE. During the visit, her weight is recorded as being 40 lb. Assuming expected growth parameters, how much will Holly most likely weigh in 2 years? A. 60 lb B. 52 lb C. 45 lb D. 42 lb

B. 52 lb (School age children typically gain around 5-7 lbs annually. Therefore 40 + 10 = 50 & 40 + 12 = 52 so closest answer is B - 52 lbs.)

You are examining a post-pubertal child w/café au lait macules & are determining if he has neurofibromatosis type 1. Which of the following physical findings would lead you to suspect that he has this disease? A. 6 café au lait macules measuring at least 6 mm & a father w/the condition B. 6 café au lait macules measuring at least 16 mm & 3 iris Lisch nodules C. 7 café au lait macules measuring at least 6 mm & 3 cutaneous neurofibromas D. 4 café au lait macules measuring at least 15 mm & distinct osseous lesions

B. 6 café au lait macules measuring at least 16 mm & 3 iris Lisch nodules (p. 28)

A concerned father says that his daughter recently turned 5 years old, & he has noticed she is beginning to stutter. The NP tells the father that his daughter's condition is probably normal, but you will be watching w/him to see if it persists or if she avoids speaking. As the NP, you know that it is usually acceptable to wait before referring the child for her stuttering until how long? A. 3 months B. 6 months C. 9 months D. 12 months

B. 6 months (It should be evaluated if the child continues stuttering for 6 months or avoids speaking.)

The mother of a 2 month old pt says that their family is relocating to Japan to pursue a long-term business opportunity. You know that the majority of Japan does not practice water fluoridation. You recommend that the mother incorporate fluoride supplementation of 0.25 mg/day into her child's diet, starting at what age? A. 3 months B. 6 months C. 1 year D. 3 years

B. 6 months (The AAP recommends that in areas w/little to no water fluoridation, children s/be started on fluoride supplementation @ 6 months, w/a daily dose of 0.25 mg. Supplementation is not required for the 1st 6 months of life, meaning 3 months of age is too early to introduce fluoride. By 1 year of age, the patient should already be receiving 0.25 mg/day. From 3-6 years of age, children should receive 0.50 mg/day of fluoride.)

What feature best distinguishes the characteristic murmur of patent ductus arteriosus? A. A holosystolic thrill @ the LLSB B. A "machinery" noise @ the LUSB C. An ejection murmur w/radiation to the left interscapular area D. A click @ the LUSB that decreases w/inspiration & increases w/expiration

B. A "machinery" noise @ the LUSB

You ask Samuel, a toddler, to point to one body part & he points directly to his elbow. Samuel's mother states that he just started correctly pointing to body parts last week. If Samuel is properly reaching expected developmental milestones, he would most likely be: A. About 13 months old B. About 18 months old C. About 20 months old D. About 2 years old

B. About 18 months old (A properly developing toddler would be expected to point to his body parts @ 15-18 months old. A 13 month old can typically walk & understand a few words, but would not be expected to point to body parts. A 20 month old & a 2 year old would already be expected to be able to properly indicate parts of their body.)

A mother has just given birth to her infant daughter. The mother's chart indicates that she is positive for HBsAg. Regarding hepatitis B vaccination, which of the following is the correct action to take initially? A. Arrange an appointment for the mother to bring her daughter in for hepatitis B vaccine when her child is 1 - 2 months of age B. Administer the hepatitis B vaccine & 0.5 ML hepatitis B immune globulin within 12 hours C. Test for HBsAg & antibody to HBsAg in the infant D. Administer the hepatitis B vaccine before the child discharges & follow up with another 2 vaccinations given 2 & 6 months of age

B. Administer the hepatitis B vaccine & 0.5 ML hepatitis B immune globulin within 12 hours (Infants born to HBsAg positive mother should receive the hepatitis B vaccine (HepB) & 0.5mL hepatitis B immune globulin w/in 12 hours of birth. All newborn children should receive the 1st dose of HepB before hospital discharge. The 2nd dose, not initial dose, should be given at 1-2 months of age. Administering the hepatitis B vaccine before the child discharges, & then following up w/another 2 vaccinations given at 2 & 6 months of age is that correct dosing series for a child born to HBsAg negative mothers, but incorrect for those born to HBsAg positive mothers. Testing for HBsAg & antibody to HBsAg in the infant is a necessary action only after the child has received 3 or more doses HepB vaccine.)

Which of the following standardized test may be performed by parents w/no formal training? A. Denver Developmental Screening Test II B. Ages & Stages Questionnaire C. Bayley Scales of Infant Development D. Newborn Behavioral Assessment Scale

B. Ages & Stages Questionnaire

Which of the following standardized tests may be performed by parents w/no formal training? A. Denver Developmental Screening Test II B. Ages & Stages Questionnaire C. Bayley Scales of Infant Development D. Newborn Behavioral Assessment Scale

B. Ages & Stages Questionnaire (The Ages & Stages Questionnaire is a standardized test that may be utilized by parents or care takers & does not require a professional. The Denver Developmental Screening Test II & the Bayley Scales Of Infant Development test typically require the employment of professionals. The Newborn Behavioral Assessment Scale may be performed by Brazelton-certified individuals.)

Which of the following diagnostics is least likely to indicate a diagnosis of rheumatic fever? A. EKG B. Aldosterone level C. Echocardiogram D. Throat culture

B. Aldosterone level

Which of these couples in the early stages of their pregnancy would least require a genetic screening for mutations in the HEXA gene? A. An Inuit mother & Louisiana Cajun father B. An African-American mother & a Druze father C. An Ashkenazi Jewish mother & a Scots-Irish father D. A Pinoy mother & a French Canadian father

B. An African-American mother & a Druze father (The mutations at the HEXA gene can result in Tay-Sachs dz, most commonly found in Ashkenazi Jewish, French Canadians, Louisiana Cajuns.)

A child with a h/o rheumatic fever presents with syncope. A grade II, systolic thrill is heard at the right upper sternal border. The NP also hears a consistent systolic ejection click. The child's x-ray indicate left ventricular hypertrophy. Which of the following is the most likely diagnosis? A. Ventricular septal defect B. Aortic stenosis C. Pulmonic stenosis D. Group A Beta-hemolytic streptococcus infection

B. Aortic stenosis

You are examining a 10yo boy who is presenting w/epigastric pain & low grade fever. The pt's mother indicated that the pt has been feeling ill all week & vomited just today before the visit. The pain worsens w/coughing & localizes to the RLQ. Suspicious, you extend the pt's right thigh, which causes him to cry out in pain. Which of the following is the most likely diagnosis? A. Acute gastroenteritis B. Appendicitis C. Testicular torsion D. Gastroesophageal reflux dz

B. Appendicitis

A 5 yo patient is brought to your clinic w/extensive 2nd deg burns. The burns cover the entirety of the back of his head but do not extend any further than that. Approximately how much of his body has been burned? A. ~ 6% B. ~ 7% C. ~ 9% D. ~ 10%

B. Approximately 7% (For a 5 yo child, the back of the head is commonly considered 7% of the surface area of the skin. The surface area of a child's head, thighs, & legs changes during early development, & thus has varying values based on age. The back of the head would typically be considered 10% of total skin surface area in a newborn, 9% in a 1 yo, & 6% in a 10 yo.)

The mother brings her 5yo child, who is coughing out yellow-tinged mucus, to your clinic. Yesterday, both vomiting & diarrhea occurred. During a physical exam, you discover a high fever. The patient appears ill & lethargic, & crackles are heard over the right lower lobe by auscultation. An x-ray reveals infiltrates. Which of the following conditions most accounts for all the patient's signs & symptoms? A. Epiglottitis B. Bacterial pneumonia C. Pulmonary edema D. Sinusitis

B. Bacterial pneumonia (p. 114)

A 9 mo boy is brought to your office by his parents. The infant has a history of irritability, crying, & flexing his legs upwards while kicking his feet in the air. These episodes have become more frequent in the last few hours. A sausage-shaped mass is palpable over the infant's abdomen; you note streaks of blood & mucus in his diaper. Which of the following diagnostics is most likely to be used to confirm the most likely diagnosis? A. Complete blood count B. Barium enema C. Urine catecholamines D. Rectal biopsy

B. Barium enema (p. 74)

The parents of 8-year-old Morris are worried about his adjustment to school. He has not exhibited dysfunctional behavior yet, but his older brother, who is now 20, did. Which of the following is not a developmental warning sign for younger school children? A. Frequent need to stay home from school B. Being a picky eater C. Not working too ability D. Poor adjustment to school

B. Being a picky eater (p.49)

Which test or exam should be performed @ every female adolescent WCE regardless of personal or family medical history? A. Scoliosis assessment B. Blood pressure C. Liver function tests D. A pelvic exam

B. Blood pressure

A 6 yo pt arrives at your clinic w/a painful limp that seemed to develop out of nowhere. While examining the pt, you discover that the internal rotation of the hip causes spasms. Suspecting toxic synovitis, you order a series of lab panels & imaging studies for confirmation. Which of these procedures would not typically be ordered? A. US B. Creatine kinase test C. MRI D. ESR

B. Creatine kinase test (Creatine kinase determination is used to detect inflammation of muscles, not joints, & would serve little use in confirming a dx of toxic synovitis. The presence of inflammation stemming from toxic synovitis is usually detected through a CBC, an ESR, or a C-RP. US is used to detect intracapsular effusion, whereas MRI is used to r/o other potential causes of pain.)

An 8-year-old patient with asthma presents with coughing & shortness of breath. The child was playing tag but immediately left the group of children due to the severity of his symptoms. Which of the following should a NP expect regarding this child's forced expiratory volume in one second? A. Decreased volume of gas remaining in lungs after maximal expiration B. Decreased volume of gas expelled in the first second of the forced vital capacity maneuver C. Increased volume of gas expelled in the first second of the forced vital capacity maneuver D. Maximal air flow rate achieved in forced vital capacity maneuver

B. Decreased volume of gas expelled in the first second of the forced vital capacity maneuver

Which of the following is the first component of quality assurance? A. Collecting & organizing data B. Delineating the scope of care C. Assessing the effectiveness of an action D. Establishing thresholds for evaluations related to the indicators

B. Delineating the scope of care (Once a quality management plan is developed, the next step in quality assurance is to delineate the scope of care, making collecting & organizing data the correct answer. The remaining steps are, in the following order, establishing thresholds for evaluations r/t the indicators, delineating the scope of care, and assessing the effectiveness of an action.)

The mother of a toddler complains that her child is a picky eater, &, despite evidence of a healthy growth pattern, she is concerned about her child's nutrition & diet. Which of the following is most accurate regarding toddler nutrition? A. Parents of toddlers should insist their children clean their plate & consume 90 kcal/kg/day B. Despite being picky eaters, toddlers should consume 100 kcal/kg/day C. Toddlers should consume more protein & a total of 115 kcal/kg/day D. Toddlers are very active & require more calories than infants

B. Despite being picky eaters, toddlers should consume 100 kcal/kg/day (p. 5, 70-100 kcal/kg/day)

Melanie, an 8-month-old baby, comes to the clinic with bluish skin & a cleft palate. The pediatric NP notes that the baby has twitching around her mouth with spasms on her hands and arms. Melanie's parents also report seizures. Upon further observation & interview, the NP learns that the baby is also developmentally delayed in many areas. Which of the following should be a part of your differential diagnosis? A. Tay-Sachs dz B. DiGeorge syndrome C. Marfan syndrome D. Turner syndrome

B. DiGeorge syndrome

During routine screening, a 22 mo old patient shows signs of short stature. The MHx, which reveals poor weight game & recurrent GI symptoms that include vomiting & chronic diarrhea, lead you to suspect celiac dz. Which of the following findings would enable you to establish the dx & determine the etiology of the patient's short stature? A. Human leukocyte antibodies B. Endomysial antibodies C. Serum antinuclear antibodies D. Growth hormone antibodies

B. Endomysial antibodies

A mother brings her 1 yr old to your practice, claiming that the child has had a fever for 6 days. She thought it was a simple illness at first, but she has noticed recent swelling in the hands & feet. An examination shows that the child has inflammation in the lips & oral cavity, as well as redness in both eyes w/out exudate. To rule out Kawasaki dz, which test would best help to isolate the child's most likely condition? A. Hemoglobin & hematocrit level B. Erythrocyte sedimentation rate C. WBC D. Blood urea nitrogen

B. Erythrocyte sedimentation rate

Of the following, which is a necessary guideline for interviewing adolescents in a clinical setting? A. Having the adolescent fill out a questionnaire at the same time. B. Excluding the parents from the interview with the adolescent C. Advising the adolescent that his/her parents are privy to what was discussed in the interview D. Having the parents & the adolescent in your office for a meeting

B. Excluding the parents from the interview with the adolescent

A 1 mo patient presents w/chronic cough, nasal passages filled w/mucus, & a respiratory infection. You find an obstructive pattern in the pulmonary functions test. A chest x-ray indicates atelectasis in the right lung. You try to look for more signs to confirm your suspected diagnosis. Of the following findings, which would not help you confirm the most likely diagnosis? A. Large, liquid, bulky, foul stool B. Fever C. Hepatosplenomegaly D. Salt-tasting skin

B. Fever (p. 115)

A 9yo male & his mother present to your office. The boy explains that he has been having trouble breathing & keeping up with the other children on his soccer team. His mother noticed him wheezing & states, " I'm afraid he might have asthma." Which pulmonary function test results would confirm these suspicions? A. Forced expiratory volume (FEV) in 1 second of 1.4 liters & a forced vital capacity (FVC) of 1.7 liters B. Forced expiratory volume (FEV) in 1 second of 1.8 liters & a forced vital capacity (FVC) of 2.9 liters C. Forced expiratory volume (FEV) in 1 second of 2.8 liters & a forced vital capacity (FVC) of 3.5 liters D. Forced expiratory volume (FEV) in 1 second of 1.0 liters & a forced vital capacity (FVC) of 1.2 liters

B. Forced expiratory volume (FEV) in 1 second of 1.8 liters & a forced vital capacity (FVC) of 2.9 liters (A FEV1 of 1.8 liters & a FVC of 2.9 liters indicates a FEV1/FVC of 62%, well below the threshold of 75-80% ratio of healthy individuals. Low FEV1/FVC ratios usually indicate an obstructive pulmonary dz. In the case of this pt, the low ratio, which indicates moderate obstruction, along with the other symptoms, most likely indicates asthma. The FEV1/FVC ratio of 1.0 liter & 1.2 liters, respectively, is 83%. The FEV1/FVC ratio of 2.8 liters & 3.5 liters, respectively, is 80%. Lastly, the FEV1/FVC ratio of 1.8 liters & 2.9 liters, respectively, is 82%.

Which of the following characteristics is not typically associated w/febrile seizures? A. Fever w/rectal temp of 102 or greater B. Generally manifests in early infancy by 3 months of age C. There is usually a FHx of febrile seizures D. There is typically a loss of consciousness w/this type of seizure

B. Generally manifests in early infancy by 3 months of age

Of the following, which form of conjunctivitis most often presents w/watery discharge? A. Bacterial B. Viral C. Gonococcal D. Allergic

B. Viral

A toddler recently diagnosed as anemic is found to have reticulocyte count of 0.3%. All of the following may explain these findings except: A. Iron deficiency anemia B. Hemolytic anemia C. Folic acid deficiency D. Bone marrow failure

B. Hemolytic anemia (In hemolytic anemia, the destruction of RBCs causes the bone marrow to produce an elevated reticulocyte count. A reticulocyte count of 0.3% is below the normal range of 1-2%. Allow reticulocyte count may indicate IDA, folic acid deficiency, or bone marrow failure.)

You are treating a 24-month-old infant who has completed her PCV13 series. All of the following are possible preexisting conditions that would require administration of one dose of the 23PS formulation except: A. Chronic cardiac dz B. Hepatitis B C. Renal failure D. Asplenia

B. Hepatitis B (Other conditions that may require the 23PS would be, splenic dysfunction, HIV, nephrotic syndrome, pulmonary dz, & DM.)

Donovan has Down's syndrome. Which of the following is a genotypic presentation of Down's syndrome in Donovan? A. Protruding tongue B. Homozygous for a single transverse palmar crease C. Upward slanting eyes D. Inner epicanthal folds

B. Homozygous for a single transverse palmar crease (p. 22)

Your nephew, Ben, is 8 years old & just joined the Cub Scouts. He is proud of his uniform & eager to earn his badges. Ben's latest assignment is to make a birdhouse, & he has been working hard at it all afternoon. Which of Erik Erickson's psychosocial stages would best describe Ben? A. The latency stage B. Industry versus inferiority C. Concrete thinking D. Initiative versus guilt

B. Industry versus inferiority

Which of the following conditions does not have a bacterial etiology? A. Epiglottitis B. Infectious mononucleosis C. Acute otitis media D. Conjunctivitis

B. Infectious mononucleosis

Which of the following is true regarding Turner's syndrome? A. It presents as a result of the mutations in the HEXA gene B. It is the most common sex-chromosome anomaly found in females C. It typically presents with tall stature & kyphoscoliosis D. It typically presents w/increased risk of infection d/t thymus aplasia

B. It is the most common sex-chromosome anomaly found in females (HEXA gene - Tay-Sachs; Tall stature, kyphoscoliosis - Marfan; increased risk of infection d/t thymus aplasia - DiGeorge)

In the middle of a busy morning schedule, a mother brings in her 6-year-old son and says that he has trouble breathing. While examining him, you notice the child frequently opens his mouth & sticks out his tongue as if he were choking. According to the mother, he also has had a high fever that appeared just yesterday. The pulse oximetry placed on his finger reads 90. Which of the following is the best course of action at this time? A. Administer oxygen and an injection of ceftriaxone B. Keep the child in a sitting position & arrange for STAT transport to the hospital C. Administer oxygen & examine the throat for foreign objects D. Lay the child in a comfortable position & give a breathing treatment

B. Keep the child in a sitting position & arrange for STAT transport to the hospital

When treating a child prone to seizures an NP should primarily keep which childhood anatomical feature in mind? A. Smaller circulating blood volume B. Large tongue compared to the oropharynx C. A thin cranium D. Large head in comparison to body proportion

B. Large tongue compared to the oropharynx (Developmentally, the tongue of a child is often comparatively > than the oropharynx, which can protectively cause obstruction during a SZ & may lead to severe repercussions attributed to O2 loss. Children have a smaller circulating blood volume in absolute terms, but this is primarily a concern in cases of blood loss or bacterial infection no SZ. Children have thinner craniums, which would place them @ > risk of head injury if the skill is penetrated' this may present a concern during convulsions, but is < of a concern that the risk of obstruction. Lastly, a child's large head, in comparison to the child's body, accounts for a smaller body surface area when compared w/an adult, but this doesn't greatly influence potential complications from SZ)

You are treating a 14yo soccer player for transient knee pain. His father tells you that the boy's ability to kick has declined recently because of tightness in his hamstrings. He adds that the boy is upset about falling behind his peers on the team & would like to be able to participate again ASAP. Looking @ the boy's chart, you see that he has grown 2 1/2 inches in the last 6 mo. Based on the most likely diagnosis, which of the following treatments is most appropriate for this pt? A. Complete restriction of activity until the end of growth spurt. B. Limit physical activity C. No treatment is necessary D. Surgical excision of bursa & referral to orthopedic surgeon

B. Limit physical activity (The most likely dx is Osgood-Schlatter dz. However, the tightness in the hamstrings warrants a recommendation of limiting activity. In children, Osgood-Schlatter dz is self-limiting, so surgical intervention is almost never recommended. Likewise, complete activity restriction is not recommended until after the child's growth spurt.)

Your pt, a 2yo boy, presents w/ fever of 102F & a noticeable, non-productive cough. You suspect bronchiolitis. Upon exam, you hear unusual crackling in the lungs & note that the liver & spleen are readily palpable. Which of the following findings would you most likely observe in in the diagnostic testing results? A. Elevated WBCs w/eosinophilia B. Lung hyperinflation on the chest x-ray C. Patchy infiltrates on the x-ray D. Minimum of 60mEq/L chloride in 100 mg of sweat

B. Lung hyperinflation on the chest x-ray (The child's presentation is most consistent w/acute bronchiolitis. Hyperinflated lungs on the chest x-ray are an expected finding in bronchiolitis. Patchy infiltrates on the x-ray & a minimum of 60 mEq/L chloride in 100 mg sweat are expected findings in bacterial cystic fibrosis rather than bronchiolitis. Elevated WBC w/eosinophilia is an expected finding in asthma.)

A 6 yo female who is new to this country presents w/a racing heart, rapid breathing, & pale-colored skin. During the PE, you notice splenomegaly & a very bony facial structure, Based on the patient's findings, which of the following would be most likely to present? A. MCV 82 fl; MCHC 32%; reticulocyte count 2% B. MCV 72 fl; MCHC 30%; reticulocyte count 3% C. MCV 70 fl; MCHC 34%; reticulocyte count 1% D. MCV 88 fl; MCHC 35%; reticulocyte count 1%

B. MCV 72 fl; MCHC 30%; reticulocyte count 3% (Values of MCV 72 fl, MCHC 30%, & reticulocyte count 3%, respectively, are suggestive of thalassemia, which may present w/tachypnea, tachycardia, pale skin, splenomegaly, & frontal bossing. Thalassemia is a microcytic, hypochromic anemia w/a high reticulocyte count. Patients w/MCVs of 82 fl & 88 fl fall w/in the normocytic range. The normal range for MCHC is 32-36%, so patients w/a MCHC of either 32% or 36% would be normochromic, not hypochromic. A reticulocyte count of 1-2% is considered normal, not elevated.)

A tall 15yo male comes to your office w/complaints of recent onset of double vision in his right eye, which he says worsens his already poor, nearsighted vision. Upon physical examination, you note that the patient has a hollow chest & an arm span exceeding his tall height. While taking the patient's history, you learn the family history includes unexplained heart related deaths on his father's side. Which of the following choices is the most likely diagnosis? A. Gigantism B. Marfan's syndrome C. Klinefelter syndrome D. Precocious puberty

B. Marfan's syndrome (p. 23)

Key immunizations to offer during adolescence include all the following except: A. Meningococcal B. Measles, mumps, & rubella C. Tetanus, ditheria, & attenuated pertussis D. Human papillomavirus

B. Measles, mumps, & rubella

Which of the following vaccines should not be given to patients who are pregnant? A. Inactivated polio vaccine B. Measles, mumps, rubella vaccine (MMR) C. Influenza vaccine D. Tetanus, diphtheria, & acellular pertussis vaccine (Tdap)

B. Measles, mumps, rubella vaccine (MMR) (The MMR should not be given to the pregnant or immunosuppressed patient as it is a live virus. And women should avoid getting pregnant until after 4 weeks after getting the MMR. The ideal time to vaccinate a pregnant woman is 27-36 weeks gestation.)

Which of the following is least likely to result in lead poisoning? A. A red wagon from the 1970s B. Mexican soft drinks C. Inner-city playgrounds near major highways D. Indian herbal remedies

B. Mexican soft drinks

Which of the following best defines a simple partial seizure? A. Staring for 25 seconds prior to onset of minor autonomic symptoms B. Minor motor symptoms w/no loss of consciousness C. Staring for 10 seconds w/brief onset & termination D. Partial LOC w/increased muscle tone

B. Minor motor symptoms w/no loss of consciousness

What are the last permanent teeth to fully erupt during development? A. Bicuspids B. Molars C. Incisors D. Canines

B. Molars

Which of the following best describes the state of the pancreatic islet cells @ symptom presentation in pediatric patients w/DM1? A. Pancreatic islet cells have shrunken in #s but grown in size B. Most pancreatic islet cells have been destroyed C. The # of pancreatic islet cells is near normal levels but shrinking rapidly D. Pancreatic cells produce insulin @ a lower rate but their # doesn't change

B. Most pancreatic islet cells have been destroyed

You're discussing treatment options with the family of a boy who has just been diagnosed with like Legg-Calvé-Perthes disease. Given that the boy has just turned 5, which of the following would you expect to be part of the treatment plan for this condition? A. Orthosis worn until early ossification is noted B. Observation only C. Referral for pin fixation with open epiphysiodesis using bone graft D. Referral for femoral osteotomy

B. Observation only (Since the child is < 6 years of age, observation without intervention is permissible depending on the extent of epiphyseal involvement, while aggressive treatment such as orthosis & femoral osteotomy are not recommended. The age of the child, whether full range of motion is preserved, & the involvement of the femoral head should all be considered in the management plan for this patient's condition. Pin fixation with open at epiphysiodesis using a bone graft is a treatment for slipped capital femoral epiphysis, rather than Legg-Calvé-Perthes disease.)

As a NP, you know that the eating disorder mortality rate is about 10% but that a complication from the eating disorder, & not the eating disorder itself, is frequently listed as the cause of death. Which cause of death is most likely to be listed as the cause of death of an individual who dies from an eating disorder? A. Asphyxia B. Organ failure C. Anemia D. Dehydration

B. Organ failure (Organ failure is the most common cause of death in eating disorders d/t severe malnutrition; cardiac complications, such as arrhythmias & loss of muscle are common, but they may also result in liver damage & multi-organ failure.)

You are conducting an assessment for a 12 yo whose feet point slightly inward, though he walks normally w/o a limp. His chief complaint is pain in the "front, bottom part" of his knee, pointing to the tibial tubercle. He indicates that he began playing soccer year-round 7 months prior w/o any significant incident or injury. The pain began 3 weeks ago & has been constant since. Which of the following conditions is the pt most likely experiencing? A. Genu valgum B. Osgood-Schlatter dz C. Grade I knee sprain D. Legg-Calve-Perthes Dz

B. Osgood-Schlatter dz

Which of these signs would indicate meningitis in a pt? A. Pain upon external rotation of the knee B. Pain upon extension of the leg when the hip is flexed 90 deg C. Sciatic pain when both legs are elevated off the exam table D. Pain w/internal rotation of the flexed right thigh

B. Pain upon extension of the leg when the hip is flexed 90 deg

How & where does the obturator sign present in pediatric patients w/appendicitis? A. Sharp pain in RLQ of abdomen B. Pain w/internal rotation of the right hip C. Pain w/extension of the right hip D. Tenderness 1/3 distance from the anterior superior iliac spine to the umbilicus

B. Pain w/internal rotation of the right hip

Mia brings her 4-year-old son, Mason, to the clinic because he has been having problems speaking. He had been speaking normally, but, as he tries to speak in longer sentences, he begins stuttering. Mia is concerned that her son will be made fun of by other children & asks the NP whether he should begin speech therapy. Which of the following is the NP's best advice? A. Begin speech therapy as quickly as possible B. Patiently observe for about 6 months then consider referral C. Begin speech therapy if the stuttering last > 3 months D. Postpone speech therapy until the child stop speaking altogether

B. Patiently observe for about 6 months then consider referral

Which of the following it's not true about Turner's syndrome? A. Most common sex chromosome anomaly of females B. Patients have tall stature w/irregular arm span C. Occurs in 1:2,000 live births D. 95% of embryos do not survive to term

B. Patients have tall stature w/irregular arm span (p. 23)

You are assessing a newborn who presents w/a reddish-purple vascular malformation of the skin consisting of dilated capillaries. You recognize this as which of the following? A. Café au lait spots B. Port-wine stain C. Strawberry mark D. Mongolian spots

B. Port-wine stain (p. 28)

A worried mother brings her 6yo son in for a check up. He presents with a 103F fever, subcutaneous nodules, & cries out frequently because of pain in his arms & legs. Which of the following labs would you most expect in this child? A. Right ventricular hypertrophy on the electrocardiogram B. Positive rapid strep assay C. Positive C-reactive protein D. Left ventricular hypertrophy on the electrocardiogram

B. Positive rapid strep assay (The child presents with 1 major finding, subcutaneous nodules, & 2 minor findings, 103F fever & arthralgia, included in the Jones' criteria for the diagnosis of rheumatic fever; therefore, a positive rapid strep essay is most expected, as this condition usually follows a group A strep infection of the upper respiratory tract. Furthermore, the patient is within the age group that has the highest incidence of this condition. The other findings are not expected with rheumatic fever.)

David is a 14-year-old with attention deficit hyperactivity disorder. He has been unresponsive to treatment with central nervous system stimulants. David has also been diagnosed with depression. Which of the following treatments is most appropriate for David's conditions? A. Referring the patient will behavior therapy B. Prescribing second-line medications C. Ceasing all medications D. Prescribing homeopathic treatment

B. Prescribing second-line medications

Which 2 lab findings best characterizes the findings of restrictive dz in pulmonary function test results? A. Reduced airflow rates & lung volumes w/in normal range or larger B. Primarily reduced volumes &, to some extent, reduced expiratory flow rates C. Normal volumes & reduced expiratory flow rates D. Primarily reduced airflow rates &, to some extent, increased lung volumes

B. Primarily reduced volumes &, to some extent, reduced expiratory flow rates (Restrictive dz in PFT results is primarily characterized by reduced volumes &, to some extent, expiratory flow rates. Primarily, reduced airflow rates & expiratory flow rates accompanied by lung volumes w/in the normal range or larger are, to some extent, more indicative of obstructive dz, not restrictive dz.)

You are treating Cynthia, age 9, who has recently been diagnosed w/hyperthyroidism, What 2 drugs should you recognize as typical 1st line treatment? A. Tapazole & synthroid B. Propranolol & Tapazole C. Levothyroxine & propranolol D. Insulin & propylthiouracil

B. Propranolol & Tapazole

The most common findings of impetigo include all of the following choices except: A. Pain B. Pruritic rash C. Honey-crusting lesions D. Regional lymphadenopathy

B. Pruritic rash

Which of the following is not a physical manifestation of asthma? A. Hypertrophy of mucus glands B. Purulent sputum C. Hypertrophy of smooth muscle D. Mucosal edema & hyperemia

B. Purulent sputum (p. 111)

You are weighing the benefits of performing invention on a patient & conclude that the quality of benefit the intervention will produce would be extremely poor. Which of the following is the correct definition of this type of intervention? A. Sentinel event B. Qualitative futility C. Quantitative futility D. Patient medical abandonment

B. Qualitative futility

Which of the following types of heart murmurs is not typically considered an innocent heart murmur? A. Continuous humming murmur B. Radiating murmur C. Murmur that may vary w/position changes D. Musical systolic murmur

B. Radiating murmur

Which f these clinical tests would be most useful in evaluating the degree of a pt's scoliosis? A. Genetic testing B. Radiograph C. Ultrasound D. Joint fluid aspiration

B. Radiograph

You are examining a newborn in the nursery. As part of your examination, you stimulate the Moro reflex. How would you do this, & what reaction would you look for in the patient? A. Turn his head to one side & watch for contralateral upper extremity to extend. B. Release your support of the newborn's head to observe nervous system response. C. Stroke the infant's lower foot to observe an upward curling of his big toe. D. Hold the baby aloft to see if he extends his arms towards the ground.

B. Release your support of the newborn's head to observe nervous system response. (The Moro reflex is best tested by releasing the newborn's head in order to observe the nervous system's response to surprise; most infants will respond by abducting or addicting their arms in an attempt to regain balance. an impaired or absent response may indicate motor or neurologic disorders. Turning the head to one side w/ipsilateral arm extension is the fencing reflex. Stroking the infant's lower foot tests the Babinski reflex, which, barring pathology, commonly vanishes by 2 years of age. Holding the infant aloft & rotating him or her forward to test the response to a simulated fall is part of the parachute test & should not typically stimulate the startle reflex.)

During a physical exam on a 17 yr old female, you note a sore throat, parotid gland enlargement, & dental erosion. You begin to suspect a possible eating disorder, Which of the following findings would most strongly indicate bulimia nervosa in this patient? A. Amenorrhea B. Russell's sign C. Lanugo D. Thinning hair

B. Russell's sign (Characterized by bruises on knuckles that result from self-induced vomiting, is often an indicator of bulimia. Amenorrhea can be an indicator for either bulimia or anorexia nervosa. Thinning hair is commonly a sign of anorexia nervosa. Lanugo is more typical of anorexia than bulimia.)

You are treating a 5-year-old boy who has been diagnosed w/celiac disease. You discuss w/his mother how to adjust the child's diet accordingly. Which of the following should be included in an appropriate diet for this child? A. Whole grain cereal B. Skim milk & fresh vegetables C. Rice & vegetables, w/soy substitute for protein D. Oatmeal w/soy milk instead of lactose milk

B. Skim milk & fresh vegetables (p. 77)

Danny, a 13yo African-American male, is 5'6" & has a BMI of 34. His temperature & BP are within the normal range. He has grown 2.5 inches in the last 12 months. He normally spends several hours each afternoon & evening in his room playing video games, but, in an effort to lose weight, he walks for an hour every evening. Today, he began to feel pain in his left thigh & knee, & now he has groin pain as well as pain in the right thigh & knee. He denies any recent bodily trauma. In your physical examination, you note that he is unable to properly flex either hip when the femur abducts. Which of the following is the most likely diagnosis? A. Osgood-Schlatter dz B. Slipped capital femoral epiphysis (SCFE) C. Legg-Calve-Perthes dz D. Septic arthritis

B. Slipped capital femoral epiphysis (SCFE)

A 14 yo patient presents to your clinic w/complaints of stiff joints & pain that starts in her shoulders before traveling down her arms. She reports that, following a recent nature hike, she found a tick on her leg. W/in a few days, a lesion that "looks like a bull's eye" had developed at the tick site. She also tells you that she has been experiencing severe HA, & that her heart occasionally "races like a drum". Which stage of the most likely dx is the patient experiencing? A. Stage 1 B. Stage 2 C. Stage 3 D. Stage 4

B. Stage 2 (HA, stiff joints, migratory pains, & cardiac irregularities all typically characterize Stage 2 of Lyme Dz, as would ascetic meningitis, Bell's palsy, & peripheral neuropathy. Although muscle & joint pain may present in Stage 1, this stage is more commonly characterized by flu-like symptoms, fatigue, & erythema migrants. Stage 3 is also often characterized by pain, stiffness, & swelling in the joints, but these symptoms would typically evolve into clear manifestations of arthritis, usually presenting in the knees. Furthermore, other symptoms such as acrodermatitis chronicum atrophicans, characterized by bluish-red discoloring of the distal extremity w/edema, or subacute encephalopathy may produce in Stage 3. There is not Stage 4 in Lyme Dz.)

Bulimia is harmful to overall health & destructive to teeth. Which of the following factors is most damaging to the teeth? A. Germs introduced into the mouth from the fingers B. Strong acids that erode tooth enamel C. Failure to brush twice a day D. Change to the oral pH from vomiting

B. Strong acids that erode tooth enamel

A 4-year-old boy comes to your office accompanied by his mother, who states that she has been concerned about frequent pauses & repetition in her son's speech. After conducting differential diagnosis for hearing & visual impairment, you diagnose the child w/stuttering. You will assure his mother that no immediate treatment is needed, & that stuttering frequently resolves on its own; however, her son may require a future referral. Which of the following would typically not require a referral? A. Stuttering lasting more than 7 months B. Stuttering lasting for 4 months C. Child totally avoids speaking D. 7-year-old child w/a stutter

B. Stuttering lasting for 4 months (p. 44)

You were taking the history of a 15yo patient who complains of being "tired all the time". She presents with dry, flaky skin, puffiness around the eyes, & reports being very depressed. Included with your examination & lab work should be which of the following tests? A. CBC, electrolytes, and BUN B. TSH and T4 C. Urine for T4 & T3 D. T4 & serum T3 test

B. TSH and T4

Seizures & dementia can manifest in infants & young pediatric patients as a typical result of which of the following disorders? A. Klinefelter's syndrom B. Tay-Sachs disease C. Turner's syndrome D. Marfan syndrome

B. Tay-Sachs disease (Progressive disorder that destroys nerve cells in the brain & spinal cord, manifesting as seizures, dementia and eventually leading to death.)

David & Sally want to make their home as safe as possible for their toddler. They come to your office asking for advice. When discussing safety issues with them, which of the following is the best advice to give? A. It is crucial to nail into place window guards. B. Test the temperature of hot water before giving a bath. C. Train the toddler to stay off the stairs. D. Talk to the toddler about staying away from poisons.

B. Test the temperature of hot water before giving a bath.

Tanner stages help providers understand the stage of sexual development in adolescents. Which of the following best describes the Tanner stage II in boys? A. Rugae appear B. Testes grow larger C. Penis elongates D. Glans develops

B. Testes grow larger (p. 58)

A 4-month-old baby presents with mild distress diagnosed as bronchiolitis. Immunofluorescence analysis is positive for respiratory syncytial virus, also known as bronchiolitis. Oral intake is adequate & urine output is good. Which statement is true regarding treatment options for this patient? A. A one time dose of palivizumab at 15 mg/kg should be given B. Infant may be treated at home w/hydration & saline nose drops C. The infant should be hospitalized for respiratory distress and receive respiratory treatments D. The NP should order home nebulizer treatment of albuterol PRN

B. The infant may be treated at home with hydration & saline nose drops

Two 2 year olds are playing in a sandbox. They are both scooping & pouring sand, occasionally glancing over at each other but not interacting. How would the NP best characterize this type of play? A. This behavior is abnormal, as by now they should want to play together B. This behavior demonstrates parallel play, which is common in this age group C. These children are participating in associative play, which is not expected in this age group D. This is active play & is typical in this age group

B. This behavior demonstrates parallel play, which is common in this age group

An infant presents w/a grade II systolic murmur. His chest x-ray reveals an "egg on a string" image, cardiomegaly, & increased pulmonary vascular markings. Which of the following is the most likely condition? A. Coarctation of the aorta B. Transposition of the great arteries C. Pulmonic stenosis D. Tetralogy of Fallot

B. Transposition of the great arteries. *Transposition of the great arteries presents w/a grade II systolic murmur & a chest x-ray findings of an "egg on a string, " cardiomegaly, & increased pulmonary vascular markings. Tetralogy of Fallot x-ray findings exhibit a boot-shaped heart & no cardiomegaly or increased pulmonary vascular findings. Coarctation of the aorta presents w/cardiomegaly & pulmonary vascular markings, but rib notching due to collateral circulation would also be noted. Pulmonic stenosis presents with normal x-ray findings.)

The mother of a 3yo boy inquires about her son's immunization schedule. The mother mentions that her son has had initial measles, mumps, and rubella (MMR) vaccination shortly after his 1st birthday, & she wants to know how many additional doses he should receive. How many doses of MMR are recommended for any child following the normal vaccine schedule? A. Four B. Two C. One D. Three

B. Two (p. 19)

Concerned parents bring their 2 yo son w/hypospadias to the clinic. They say they wish to learn more about the likelihood of other genitourinary anomalies that may occur alongside hypospadias. The NP should explain that all of the following disorders may potentially present in conjunction w/hypospadias except: A. Undescended testicles B. UTI C. Inguinal hernia D. Hydrocele

B. UTI (A UTI is not known to accompany hypospadias, although pts which have undergone hypospadias corrective sx are at an increased risk for UTIs, especially if hair-bearing skin was used int he operation. Undescended testicles, inguinal hernia, & hydrocele are GU anomalies that may potentially occur alongside hypospadias.)

A 3-year-old male patient presents for a WCE. While interviewing his parents, you learn that the patients family comes from a working-class neighborhood in downtown Los Angeles. All of the following laboratory exams are especially indicated at this age except: A. Hemoglobin & hematocrit B. Vitamin D level C. Cholesterol D. Lead level

B. Vitamin D level (Cholesterol only if warranted by weight > 95%, but there are not requirements for the vitamin D testing.)

Infants w/o any drastic health ailment would typically require supplementation of all of the following except: A. Vitamin D B. Vitamin K C. Iron D. Fluoride

B. Vitamin K (p. 36)

An elderly male presents with symptoms that include a burning sensation in the eye and red, scaly flakes around the eyelids, which are covered with thick crusts. Based on the patient's presentation, which of the following medications should be prescribed to treat the patient's condition?

Bacitracin or erythromycin

A chest radiograph shows an area of consolidation on the lower lobe of the lung. Which of the following conditions is most likely?

Bacterial pneumonia

Fever, tachypnea, or tachycardia, productive cough. CXR shows lobar consolidation. May have pleuritic chest pain with cough.

Bacterial pneumonia

Which of these would a practitioner use a test to assess a patient's cerebellar function?

Balance and coordination

Which test can be used to diagnose either Hirschsprung's disease or intussusception?

Barrium enema

Based on the image below, the patient has:

Basal cell carcinoma

Which of the following choices best describes the proper order of steps for performing the Lachman's test?

Bend the knee 20 degrees, apply pressure to proximal tibia and stabilize the thigh.

The practitioner knows that which of the following drugs should be increased to help alleviate Parkinson tremors?

Benztropine (anticholinergic)

Samantha, 15, has been dealing with asthma since she was 4 years old. She is now showing signs of hypertension. She asks for your advice concerning management of the condition and expresses specific concerns about any prescription that may cause her to wheeze. With this concern in mind, which of the following should you not advise her to prescribe for her new condition?

Beta blockers

Which of the following antihypertensive medications should the nurse practitioner avoid when treating patients with emphysema?

Beta-blockers

Which of the following classes of drugs is implicated with blunting the signs and symptoms of hypoglycemia in diabetics?

Beta-blockers

When should fetal heart tones be able to be first detected?

Between 10 and 12 weeks

Sandra, patient who is 34 weeks pregnant, arrives at your clinic with complaints of bleeding. She does not report any pain. Given your concern regarding possible placenta previa, what should not be performed at this time?

Bimanual exam

You notice on your patient's charts that he is dealing with a recurrent case of ulcerative colitis. Given your knowledge of ulcerative colitis, which of the following symptoms would you most expect your patient to report?

Bloody diarrhea

A physical examination of a patient reveals a heart murmur near the second intercostal space (ICS). You suspect the patient may have aortic stenosis and you auscultate again for more details. Which details of the murmur would most likely confirm your diagnosis?

Blowing, rough, radiates to neck

An elderly man presents with swollen lymph nodes and a painless lump in his neck. He also reports that he recently has been feeling generally weak and sometimes has a fever. During your exam, you note tiny, red spots on the skin of his hands. Which of the following is required to confirm the diagnosis of the most likely cause of his presentation?

Bone marrow aspiration

Which of the following laboratory tests would best confirm the most likely diagnosis of Leukemia?

Bone marrow aspiration

Which of these statements regarding folic acid deficiency and pernicious anemia is true?

Both folic acid deficiency and pernicious anemia are macrocytic anemias.

With regard to the culturally and linguistically Appropriate Services initiative, which of the following statements regarding health care organizations best defines Standard 5?

Both verbal and written notices must be provided to patients and consumers informing them of their rights to receive language assistance services.

Your post-menopausal patient complains of "hot flashes all the time" and insists that her quality of life is poor. You consider hormone therapy with which fact in mind?

Breast cancer has been linked to those taking HT

An infant who does not have a history of reactive airway disease and allergy has both inspiratory and expiratory wheezing accompanied by fever and profuse, clear nasal discharge. Which of the following is most likely?

Bronchiolitis (RSV)

Which of the following drugs is an opiate antagonist used to help opiate-dependent patients with withdrawal?

Buprenorphine (Buspirone)

The American Academy of Ophthalmology recommends that all patients recieve a complete screening for glaucoma by at least what age?

By age 40

A couple brings their 3yo boy to you for evaluation because he has been irritable & feverish. 3 days prior, they brought him in & were sent home w/instructions to given the child acetaminophen. However, his temp has remained @ or slightly above 102F. The parents are also worried about skin rashes found on the child's trunk & his reddened palms & soles, they point out how red the child's eyes are "from all the crying". Your exam reveals a whitish coat on the child's tongue & very chapped lips. You also see that the skin of the palms & soles is starting to peel. What of the following is the most appropriate treatment plan for this child's likely condition? A. Acyclovir; acetaminophen B. Corticosteroids; ibuprofen C. Admit to hospital for intravenous gamma globulin & high dose aspirin D. Oral penicillin; aspirin

C. Admit to hospital for intravenous gamma globulin & high dose aspirin

A Tanner sexual maturity rating of 3 in a male indicates which of the following? A. A penis that is adult in shape & appearance w/adult pattern pubic hair B. A rough, red scrotum with sparse, fine pubic hair C. An elongated penis with darker, curlier pubic hair D. Development of glans & rugae on the penis & curly, sparse pubic hair

C. An elongated penis with darker, curlier pubic hair

Which of the following best distinguishes epiglottitis from croup? A. Epiglottitis presents w/fever, whereas croup does not B. Epiglottitis targets children between 3 mo - 6 yo, whereas croup usually presents in children 6-10 yo C. An x-ray for epiglottitis will show a thumb sign, whereas an x-ray for croup will show a steeple sign D. Epiglottitis Is entirely bacterial in nature, whereas croup is entirely viral

C. An x-ray for epiglottitis will show a thumb sign, whereas an x-ray for croup will show a steeple sign

Which of the following serological test results indicate an active hepatitis A infection? A. Anti-HBc, Anti-HBsAg B. HBsAg, HBeAg, Anti-HBc, elevated IgM C. Anti-HAV, elevated IgM D. Anti-HAV, elevated IgG

C. Anti-HAV, elevated IgM (Anti-HAV, elevated IgM, serology indicates active hepatitis A. Anti-HAV, elevated IgG indicates recovered hepatitis A. HBsAg, HBeAg, Anti-HBc, elevated IgM indicates active hepatitis B. Lastly, serology results that show Anti-HBc, Anti-HBsAg indicate recovered hepatitis B.)

Autism in children is generally identified by 3 years of age. Alice is 30 months of age, & her mother mentions that she is not playing w/others at preschool, is unable to pretend play, & has a very limited vocabulary. Her only interest is stacking blocks, at which she is very good. When considering the disorders in the spectrum of pervasive development disorders, which of the following should be your initial consideration in your ddx? A. Down's Syndrome B. Fragile X C. Autism D. Asperger's Syndrome

C. Autism (Children w/autism have problems with social interaction, pretend play, & communication. They also have a limited range of activities & interests. Nearly 75% of children w/autism also have some degree of mental retardation. Children w/Asperger's syndrome display a relatively good grasp of language, unlike children w/suspected autism. Fragile X syndrome is not as prominent in female patients as male patients, as this condition severely affects those w/a single X chromosome. Thus, although this disorder causes similar symptoms, it is not an initial consideration for female patients. Children with Down syndrome present with a whole array of physical findings not seen in this patient; their findings usually include microcephaly, a flat nose, protruding tongue, among others.)

A pt has been referred to your practice by an optometrist, who found Lisch nodules in the pt's irises. Which finding would best help to confirm the suspected diagnosis? A. Photophobia B. FTT C. Axillary freckling D. Aphasia

C. Axillary freckling

Your patient, a 3yo male, hit his head after falling off his bike & now is having trouble w/his sense of smell. Which cranial nerve was most likely affected?

C. CNI (p. 123)

A 2 yo is brought to your clinic presenting w/red, irritated eyes. The mother says the symptoms manifested after they had visited a public swimming pool the day prior. Upon exam, you see no discharge. The child does not indicate any pain, only an itching sensation. You tell the mother that the condition is sel-limiting, & you proceed w/a normal saline flush on the patient's eyes. Which is the most likely diagnosis? A. Bacterial conjunctivitis B. Hordeolum C. Chemical conjunctivitis D. Viral conjunctivitis

C. Chemical conjunctivitis

You have diagnosed an infant w/hypospadias. Knowing that hypospadias is correctable, you advise the infant's parents of their available options. Which of the following must be true for the management & eradication of hypospadias to be successful? A. The infant must not have had any sort of flu vaccinations B. The newborn must still be breast-feeding C. Circumcision must not be performed D. The infant must not be toilet trained

C. Circumcision must not be performed (p. 149)

A mother brings her child, Ted, to your clinic. Ted has developmental delays, & the mother is concerned that he may not be able to be toilet trained. You know, however, that some children with developmental delays can be trained to use the toilet. Using the guidelines of the American Academy of Pediatrics, which of the following is the best advice to give her? A. Praise the child for using the toilet successfully, but scold, withhold affection from child if he was unsuccessful B. Schedule regular toilet times, rituals & make the child sit on the potty until the achieved action is done C. Confirm developmental readiness by ensuring that the child can stay dry for more than an hour, appears to be aware of when he needs to have a bowel movement, & has the motor skills to undress D. Make sure the child wears diapers in the first stage of training & later wears clothing that makes it easy for the child to manage on their own. Wait until child is chronologically 5 years of age to begin toilet training.

C. Confirm developmental readiness by ensuring that the child can stay dry for more than an hour, appears to be aware of when he needs to have a bowel movement, & has the motor skills to undress (According to the AAP, toilet training readiness can be determined in some children with developmental disorders or delays in the same manner as all children. If a child with delayed development can stay dry for more than an hour, appears to be aware of when he/she needs to have a BM, has the motor skills to undress self, & appears to be motivated, the process can begin. Most children begin to understand this concept by the cognitive ages of 1.5-2.5 yo. Depending on the individual child with delays, this process may not begin until the age of 5 or later. The chronological age of a child with developmental delays has no bearing on his/her toilet training readiness. After it is determined that the child is ready to be potty trained, then regular toilet times maybe scheduled. It is important to make it easy for the child to manage training on his/her own & to praise the child for good effort, rather than punish for failure.)

Which of the following is the most specific test for muscular dystrophy? A. Transaminase level B. Prolactin level C. Creatine kinase determination D. Polymerase chain reaction result

C. Creatine kinase determination

Sally is 16 years old & does not want to finish high school. Her mother brings her in for an exam to find out if there may be a condition causing Sally's "unusual" behavior. Sally was in excellent health, had an active social life, & received good grades. However, in recent few months, her behavior has become defiant, her grades have dropped, and she is uninterested in school or her friends. Her parents tell you that Sally's long time friend has moved out of state. Which of the following is the most likely reason for her behavior? A. Stress B. Overbearing parents C. Depression D. Puberty

C. Depression

A patient reports to your clinic w/intense fatigue & a fever. Suspecting infectious mononucleosis, you begin a PE. Which of the following signs is not consistent w/the common presentation of this condition? A. White exudate on tonsils B. Generalized lymphadenopathy C. Discolored nasal discharge D. Petechial rash

C. Discolored nasal discharge

You see 12yo Erin, who started menstruating earlier this year. Now she complains of pain & cramping, stating that she feels the pain both in her back & in her stomach. You prescribed ibuprofen. Based on Erin's presentation & your treatment, what is the most likely diagnosis? A. Gonorrhea B. Urinary tract infection C. Dysmenorrhea D. Chlamydia

C. Dysmenorrhea (p. 150-1)

A mother brings in her 15yo son because she is concerned about the effects of a concussion he had 10 days ago. She tells you that he passed out suddenly while playing video games this afternoon, and his body began "jerking around" for several minutes. Reviewing the child's medical history, you learned that he was referred for an immediate follow up at the hospital for a similar occurrence but was never seen. Which test is the most important in classifying this patient's seizure? A. Magnetic resonance imaging B. Echocardiogram C. Electroencephalogram D. Computed tomography scan

C. Electroencephalogram

You are teaching the parents of an 8 mo old infant w/GERD. Which of the following actions should you advise the parents to perform immediately after meals specifically to decrease reflux activity? A. Position the infant in a semi-supine position B. Assist the newborn to burp after meals C. Elevate the newborn's head D. Administer a histamine 2-receptor antagonist

C. Elevate the newborn's head

Which of these diagnostics is not routinely recommended for pts presenting w/classic symptoms of slipped capital femoral epiphysis? A. Radiography B. Bone scanes C. Endocrine studies D. CT

C. Endocrine studies

Which of the following is not associated w/some forms of anorexia nervosa? A. Lanugo B. Eating disturbances C. Episodic binging D. Amenorrhea

C. Episodic binging (p. 60)

Vitamin deficiency resulting from malabsorption is most likely to present with: A. Vomiting B. Chronic diarrhea C. Fatigue D. Thin stools

C. Fatigue (Fatigue is most likely to be associated w/vitamin deficiency d/t malabsorption b/c it may be a sign of anemia resulting from malabsorption of vitamin B12, folate, & iron. Vomiting & chronic diarrhea are also symptoms of malabsorption, but are not directly c/b vitamin deficiency. Thin stools usually are not indication on abnormal condition, but may result from IBS.)

The measles, mumps, and rubella (MMR) vaccine must be administered in two doses. When should the MMR vaccines typically be given? A. Both doses given between the ages of 13 - 18 years old B. First dose: 1 - 2 yo; Second dose: 15 yo C. First dose: 12 - 15 mo; second dose: 6yo D. First dose: 1 - 4 yo; second dose: 10 yo

C. First dose: 12 - 15 mo; second dose: 6yo

Which of these abdominal findings would be of most concern in newborns? A. Umbilical hernia B. Diastasis recti C. Flat abdomen D. Abdomen rising w/inspiration, falling w/expiration

C. Flat abdomen (A flat contour to the abdomen is abnormal in infants & may indicate malnutrition or IUGR. Reduction in nutrients to the fetus can cause a decrease in intra-abdominal fat. This can be caused by poor blood flow, decreased trans placental concentration of amino acids, or impairment of the ability of the placenta to transfer nutrients. Slight dissension is normal & the NP should see a slight protuberance in a health infant. A rounded, symmetrical contour to the abdomen that rises w/inspiration is normal in infants; an asymmetrical abdomen may indicate an abdominal mass. An umbilical hernia may present in the infant, but this condition is fairly harmless & often resolves itself by 1 year of age. Likewise, diastasis recti may present in newborns, bit this condition typically resolves on it is own accord.)

A 4 yo presents w/an awkward walk & run, w/o any pain & w/full ROM. The PE revels that the distance between his ankles exceeds 3 inches. Which of these conditions would be the most likely dx? A. Genu valrum B. Muscular dystrophy C. Genu valgum D. Slipped capital femoral epiphysis

C. Genu valgum

A 6 mo appears to have a shorter left leg, which turns outward slightly further than her right leg. Given this finding, which of the least expect to present during the diagnostic assessment? A. Galeazzi sign B. Barlow sign C. Gower's sign D. Allis' sign

C. Gower's sign

How would you best auscultate to confirm a venous hum? A. Have the patient lay supine; auscultate the LUSB B. Have the patient stand; auscultate the between the LLSB & the apex C. Have the patient sit up; auscultate the RUSB D. Have the patient turn his or her head; auscultate the apex

C. Have the patient sit up; auscultate the RUSB

Leonard, a 4 year old toddler, is brought to your office by this parents who have some concerns regarding his development. Which of the following factors should Leonard's parents be most concerned about? A. He has only eaten mashed potatoes for the previous 4 weeks B. He has had a stutter the previous 4 months C. He struggles w/nighttime bowel & bladder control D. He has never acquired the motor skill of skipping

C. He struggles w/nighttime bowel & bladder control (This should be acquired by the age of 3.)

You are conducting a WCE on a 5 month old boy, as per the Bright Futures guidelines. At this point in time, which of following would least qualify as a concerning finding? A. Undescended testes B. Galactorrhea C. Head lag when pulled to sitting position D. Failure to hold head up when lying on stomach

C. Head lag when pulled to sitting position

A parent has brought in her 14-month-old for a checkup. Which of the following should have already occurred in the development of the toddler? A. Eats with a fork B. Has started toilet training C. Pulls to stand D. Speaks in 2-3 word sentences

C. Pulls to stand (p. 11)

You are conducting a Well Child Check on a 5-month-old boy, as per the Bright Futures guidelines. At this point in time, which of the following would least qualify as a concerning finding? A. Undesended testes B. Galactorrhea C. Head lag when pulled to sitting position D. Failure to hold head up when lying on stomach

C. Head lag when pulled to sitting position (Although infants should mostly maintain head control by 4 months of age, it is still normal for an infant to reach this developmental milestone anywhere between 4 & 6 months of age. The inability to raise the head when lying on the stomach, galactorrhea, and undescended testes are all considered to be developmental warning signs starting at 3 on this of age.)

A 14 yo pt comes to your office showing signs of fever, painful tender joints, & strep throat. Laboratory findings show positive throat culture & a positive rapid strep assay. Based on the findings of this patient, which of the following is not typically included in the treatment regimen of the patients most likely condition? A. Bedrest acute carditis present B. Aggressive management of the strep infection C. High dose acetyl salicylic acid therapy D. Prophylactic antibiotics for invasive procedures

C. High dose acetyl salicylic acid therapy (p. 69)

A mother brings her 5 month old son in for a check up. She is concerned that her son's right leg appear shorter than his left. Furthermore, she states that his right leg looks as if it were turned out. She adds that it does not seem as if her child is in any pain though. Suspicious, the NP examined the child by adducting his right hip while applying pressure on the knee. For which the following conditions is the NP examining? A. Genu varum B. Slipped capital femoral epiphysis C. Hip dysplasia D. Genu valgum

C. Hip dysplasia

Christopher, 10 months old, is brought to your office by his parents, who state that he experiences bilious vomiting as well as infrequent BMs. Christopher's appearance is jaundiced, & his low weight & slight stature indicates failure to thrive. Upon examination you note progressive abdominal distention & a tight anal sphincter w/an empty rectum. Which of the following would most likely account for all of the patient's signs & symptoms? A. Neuroblastoma B. Intussusception C. Hirschsprung's disease D. Malabsorption

C. Hirschsprung's disease (p. 75)

When examining heart defects, what is a sign that would indicate his ventricular septal defect? A. Ejection click on the right upper sternal border B. Murmur at the left upper sternal border C. Holosystolic thrill felt at the lower left sternal border D. Ejection click on the left upper sternal border

C. Holosystolic thrill felt at the lower left sternal border (A holosystolic thrill felt at lower left sternal border is a finding of ventricular septal defect. Atrial septal defect is heard as a grade II - II/VI systolic ejection murmur at the left upper sternal border. An ejection click on the right upper sternal border is a finding of coarctation of the aorta. There is no heart defect that presents with an ejection click on the left upper sternal border.)

Max, the infant son of Mel & Anne, has a red rash on his pelvis due to the type of diaper he wears. A physical exam reveals raised bumps w/no discoloration. Which of the following would best treat the patient's bumps? A. Emollients B. Domeboro C. Hydrocortisone D. Topical antifungal

C. Hydrocortisone (p. 88 - eczema)

A 10yo female w/a loud aortic 2nd heart sound & an early systolic ejection click has a HR = 85, RR = 22, BP = 129/88. She skipped breakfast & is now c/o dizziness w/o fatigue or dyspnea. What is the most likely cause of her dizziness? A. Pulmonic stenosis B. Hypoglycemia C. Hypertension D. Congestive Heart Failure

C. Hypertension (The pts BP is elevated & the heart sounds are consistent w/HTN, which may cause dizziness. HTN in a 10yo pt is defined as 2 consecutive readings on 3 separate occasions of BP > 126/82 (note that hypertensive BP benchmarks differ by age, height, & sex for children). Hypoglycemia may make her feel light headed & unsteady but would not have associated heart murmur. Pulmonic stenosis is associated w/a systolic ejection murmur but also features cyanosis, fatigue, & dizziness, the cardiac exam was normal & did not indicate CHF.)

What is the preferred OTC analgesic for the tx of dysmenorrhea? A. Aspirin B. Acetaminophen C. Ibuprofen D. Indomethacin

C. Ibuprofen

A mother brings her 3-month-old male child to your office w/complaints of sudden difficulty feeding the child, as well as an inability to console his screaming. Although the child shows a response to her efforts, his crying & agitation have not decreased. The mother also notes that she noticed a bump on his head, which she assumed was from him hitting his head during one of his screaming fits. Which of the following would be the best initial course of action? A. Lumbar puncture B. Administration of ceftriaxone C. Immediate admission to hospital D. Blood count & culture

C. Immediate admission to hospital

A mother brings her 3-month-old male child to your office with complaints of sudden difficulty feeding the child, as well as an inability to console his screaming. Although the child shows a response to her efforts, his crying and agitation have not decreased. The mother also notes that she noticed a bump on his head, which she assumed was from him hitting his head during one of his screaming fits. Which of the following would be the best initial course of action? A. Lumbar puncture B. Administration of ceftriaxone C. Immediate admission to the hospital D. Blood count & culture

C. Immediate admission to the hospital (The signs that the child is displaying — persistent screaming w/no evidence of fever & a soft, bulging fontanelle — are indicative of intracranial pressure in the infant, which warrants immediate hospital admission. A bulging fontanelle typically occurs when fluid builds up in the brain, causing increased intracranial pressure. A lumbar puncture would be appropriate if the aim was to measure the intracranial pressure. Administering ceftriaxone would be warranted in cases of bacterial infection, which is often indicated by a temperature change in the infant. Performing a blood count test & obtaining cultures are often used to detect bacteremia in infants.)

An otherwise healthy 4 yo presents w/bruises that the parents can't explain, The parents 1st noticed the bruises a few days ago. The CBC results show the platelet count is low, hemoglobin is 12 g/dL, hematocrit is 36%, & her WBC is 7,000/mm3. Which of the following is the most likely diagnosis? A. Suspected child abuse B. Iron deficiency anemia C. Immune thrombocytopenic purpura D. Leukemia

C. Immune thrombocytopenic purpura

To ensure that your research methods are providing the intended data, you choose three parameters that you wish to test for & construct three questions for each one. After collecting the results, you look for a variance within each group of the three answers. Which of the following research principles have you appealed to him verifying your research? A. Sensitivity B. Reliability C. Internal consistency D. Validity

C. Internal consistency (Internal consistency measures the reliability of inquiries (or instruments) by grouping together various instruments that are aimed at achieving the same response. By asking the same three questions and three different ways, you have tested whether each instrument is providing the appropriate information. An instruments validity is tested by ensuring that the variable itself is tested accurately and yields the intended information. Sensitivity measures the proportion of actual positives to false positives (e.g. a pregnancy test that is 99% sensitive or accurate after a trial of 1000 uses indicates that about 990/1000 tests were accurate). Reliability gauges if an instrument will measure results the same way over time; test questions repeated over multiple administrations indicate high reliability when they elicit the same response from the same test takers.)

All of the following are school age fears the NP should expect when interviewing a child except: A. Pain B. Death C. Isolation D. Loss of control

C. Isolation

Which of these conditions is least likely to be inherited through genetics or other parental factors? A. Down syndrome B. Marfan syndrome C. Klinefelter's syndrome D. Tay-Sachs disease

C. Klinefelter's syndrome (Klinefelter's is not inherited a key clinical feature is infertility, it is a failure of chromosomal separation during meiosis.)

In which of the following age groups is primary dysmenorrhea most commonly seen? A. Early adolescence in individuals w/precocious puberty B. Women over the age of 30 C. Late adolescence D. Pre-menopausal women

C. Late adolescence

Which of the following is not a typical minor adverse reaction to a vaccine? A. Low-grade fever B. Local tenderness C. Local pallor D. Local muscle soreness

C. Local pallor

Michelle, a 6yo girl, arrives at your office presenting with a high fever. Immediately upon entering the waiting room, you see that the child has suddenly lost consciousness, has uncontrolled jerky movements, & urinary incontinence. How should the child symptoms be initially managed? A. Lumbar puncture B. Cardiopulmonary resuscitation C. Maintain airway & administer oxygen D. Lorazepam and diazepam

C. Maintain airway & administer oxygen

You are examining a toddler who is visiting from Mexico. The mother is very concerned about her child & is hoping you can treat him. You learn that the mother has been taking her son to a healer in Mexico, but his condition has not improved. The child appears fatigued & irritable. He's frequently constipated & does not have much of an appetite. The child's hemoglobin is 12.0 g/dl & the lead level is 18 mcg/dl. What is your next action? A. Order a blood transfusion B. Refer the child for possible chelation therapy. C. Measure blood levels in 1 month D. Hospitalize the child

C. Measure blood levels in 1 month (The child's lead level indicates that he has lead poisoning with a Class IIB lead level (level 15-19 mcg/dl), which should be treated completing the environmental assessment to identify potential sources of lead exposure & monitoring blood lead levels at recommended intervals. The hemoglobin level is normal for the child's age, so blood transfusion is not indicated. Chelation therapy is recommended for children with Class IV lead levels & greater (> 45 mcg/dl), & hospitalization is recommended for children w/Class V lead levels (> 70 mcg/dl).)

Tension & psychogenic HAs are commonly associated w/what mechanism of HA pain? A. Vascular dilation B. Traction C. Muscular contraction D. Inflammation

C. Muscular contraction

A newborn is born weighing 9lb. After 2 weeks, what would be the expected weight of the newborn? A. Eight lb B. Eight lb 5 oz C. Nine lb D. Nine lb 10 oz

C. Nine lb (The normal weight game progression of an infant indicates that, @ 1-2 weeks mark, the weight will be approx the same as birth. The infant will typically lose 10% of the BW in the days after birth, weighing a little over 8 lb, & then gain that weight back w/in 7-14 days. By 5 months, will double, & by 1 year triple, by 2 years quadruple.)

While examining a newborn infant, which of the following would be most indicative of a neuromuscular problem? A. Arms & legs flexed over abdomen B. Inability to track w/eyes C. No response to clapping of hands D. 3 cafe au lait spots on back

C. No response to clapping of hands (A newborn should respond to a startle w/a loud sound; failure to elicit this response could indicate hearing damage or other neuromuscular challenges. The normal posture of a newborn is flexion of extremities over trunk. Newborns may look at an object or face, but are not commonly expected to track. Cafe au lait spots are often counted & measured, & could be indicative of a problem if > 5-6 mm in size.)

During an annual physical exam, you notice that 12yo Lisa's areola & nipple project as a secondary mound. You tell her she can expect which of the following in 2 years: A. Sparse, pale, fine pubic hair B. Breast buds w/areolar enlargement C. Onset of menses D. Receding areolas & retracting nipples

C. Onset of menses

A 7-year-old patient presents w/a pruritic rash that is macular & has a crinkled appearance. 1 week later, at his follow-up exam, the appearance of the patient's rash has developed into a Christmas tree pattern, appearing on his trunk & proximal extremities. Which medication would be most likely to prescribe to treat the most likely condition? A. Calcipotriene B. Vancomycin C. Oral erythromycin D. Permethrin 5% rinse

C. Oral erythromycin (p. 91)

During the exam of a 3 mo old patient, you place the infant in the supine position w/the hips flexed 90 deg & abducted. You then grab the patient's leg while applying downward, lateral pressure. What is the name of the supplementary exam that is commonly performed in conjunction w/the exam you are performing on your patient? A. Klisic B. Barlow C. Ortolani's D. Gaeazzi

C. Ortolani's

You are evaluating a 4 mo old, whose mother says, "She's been vomiting & doesn't seem to be taking any milk or fluids. She's also been sleeping a lot more than usual." You note that the patient's fontanel is slightly sunken, & that her skin is cool to the touch & has poor turgor. These findings are least likely to be an initial indication of which condition? A. Intussusception B. Gastroenteritis C. Pyloric stenosis D. Hirschsprung's Dz

C. Pyloric stenosis

During the examination of a 3-month-old patient, you place the infant in the supine position w/the hips flexed 90 & abducted. You then grab the patient's legs by the thigh & gently adduct the leg while applying downward, lateral pressure. What is the name of the supplementary exam that is commonly performed in conjunction w/the exam you are performing on your patient? A. Klisic B. Barlow C. Ortolani's D. Galeazzi

C. Ortolani's (The scenario describes the Barlow test, which is used to assess for developmental dysplasia of the hip (DDH); the Ortolani test is typically performed w/the Barlow test & is also used to assess for DDH. These 2 tests evaluate the hip stability & are the favored diagnostic exams for patients 3 months of age & younger. The Klisic & Galeazzi signs are often performed on patients > 3 months to assess for decreased hip abduction & unequal thigh length.)

The NP is reading an interesting article that outlines a new protocol for the mgmt of COPD. She notices that the article was published in a well-known, peer-reviewed journal. Concerning the efficacy of the new protocol, which of the following is most important to consider before the NP adapts this new protocol to her practice? A. R-value B. T-test C. P-value D. Standard deviation

C. P-value (In this case, the NP should pay attention to the P-value. A high value such as 0.1, for example, would indicate the proposed protocol did not produce the desired results in 1/10 tests, whereas a p-value of 0.001 would indicate that the desired results did not occur in 1/1000 tests. The t-test compares the means of 2 groups, e.g., whether DBP differs between a control & treated group, between men & women, or any other 2 groups. While standard deviation can help determine if the results are indeed valid, this does not help in deciding whether a new protocol will produce significant results in practice. Lastly, R-value is not a measurement in statistical analysis.)

The first nurse practitioner program, in the early 1960s, was meant to address a shortage in which area of healthcare practice? A. Outpatient care B. Obstetrics C. Pediatrics D. Intensive care

C. Pediatrics

A 5 mo old infant's parent describes him as being recently irritable & constantly rubbing at his skin. Your exam reveals red-brown vesiculopapular lesions on his soles. Which drug would be the best choice to manage the most likely condition? A. Lindane lotion (Kwell) B. Calamine w/topical diphenhydramine (Ivarest or Caladryl) C. Permethrin (Nix) 5% rinse D. Griseofulvin (Grifulvin V)

C. Permethrin (Nix) 5% rinse (Permethrin 5% rinse is a common primary tx for scabies, which is indicated by the infant's irritability, itching, & red-brown vesiculopapular lesions on his soles. While Lindane is also a treatment for scabies it is not approved for < 6 months old d/t its neurotoxic side effects.)

Which of the following is an element essential for the diagnosis of attention deficit hyperactivity disorder? A. Symptoms occurring at age ≥ 7 B. Symptoms affect behavior at home C. Persistence of symptoms for at least 6 months D. Symptoms interfere w/school work

C. Persistence of symptoms for at least 6 months (p. 52)

A 7 yo presents w/symptoms indicative of asthma. His mother states that he typically experiences symptoms once per day. His mother adds that the symptoms also keep him out of some of the more active past times of his peers & cause him to wake 2-3 nights per week. How should you classify the pts asthma? A. Intermittent B. Persistent mild C. Persistent moderate D. Persistent severe

C. Persistent moderate (A pt between the ages of 5-11 yo who presents w/daily asthma symptoms, some limitation of activity, & nighttime awakening more than once per week s/be classified as experiencing persistent moderate asthma. Intermittent asthma typically presents w/no limitation of activity, symptoms < 2 d/wk, & nighttime awakenings < 2 nights/month. Mild persistent asthma commonly presents w/monitor activity limitation, as well as symptoms > 2 d/week, but not daily & nighttime awakenings > 3-4 times per month. Severe persistent asthma presents w/nighttime awakenings > once per week, but also presents w/asthma symptoms throughout the day & extreme activity limitation.)

Isaac age 6 months, has been brought to your practice by his parents for a routine check-up. As you enter the interview phase, you would know all of these methods would be well-suited for the interview except: A. Carefully phrasing potential health & safety concerns to respect the cultural practices of Isaac's parents B. Breaking from the assessment regularly to ensure the parents have accurately expressed their concerns C. Phrasing your questions in an open-ended fashion to ensure a nonjudgemental approach D. Using play to keep the patient engaged, regularly putting the assessment on hold to ensure a proper response

C. Phrasing your questions in an open-ended fashion to ensure a nonjudgemental approach (Although a non-judgmental approach s/be utilized @ all times, questions s/be directed & purposeful, not open-ended, to ensure key details of the patient's hx are not missed. Other proper techniques to utilize during an interview include ensuring cultural sensitivity, ensuring accurate perception of the parents' concerns, using lay to enhance the patient's comfort, & pausing to allow adequate time for a response.)

Which of the following pathogens responsible for pneumonia does not typically demonstrate lobar consolidation as a radiographic characteristic? A. Streptococcus pneumoniae B. Haemophilus influenzae C. Pneumocystis jiroveci D. Klebsiella pneumoniae

C. Pneumocystis jiroveci

Which of the following is typically used to screen for fungal infections? A. Erythrocyte sedimentation rate B. Serologic C. Potassium hydroxide D. Western blot

C. Potassium hydroxide (p. 85-6, 154)

Which of the following regimens would best constitute Step 3 in the stepwise mgmt of a pt w/asthma, according to national recommendations? A. Preferred low-dose inhaled corticosteroid (ICS) B. Cromolyn or Montelukast as required C. Preferred medium dose ICS w/considered oral systematic corticosteroid short course D. Preferred short-acting B2-agonist as required

C. Preferred medium dose ICS w/considered oral systematic corticosteroid short course

In male infants, cryptorchidism occurs in 20-30% of which specific population? A. Infants 6 months of age B. Brothers of males who experienced cryptorchidism C. Premature infants D. Sons of males who experienced cryptorchidism

C. Premature infants

A 5yo patient comes to your office w/a fever, shaking chills, and purulent sputum w/cough. Upon auscultation, you note that he also has lung consolidation. After a laboratory examination, you discover that the patient has a respiratory syncytial virus, Haemophilus Influenzae. Which of the following would be the best way to manage the patient's condition? A. Refer to a specialist B. Prescribe penicillin C. Prescribe amoxicillin D. Prescribe azithromycin

C. Prescribe amoxicillin (p. 114)

A mother brings her child in for a checkup. She is worried that the child is exhibiting mental development delays because she repeatedly has to keep saying things to him. You conduct a physical exam, including a Weber & Rinne test. The Weber test indicates sound lateralizes to the child's right ear, & the Rinne test indicates air conduction < bone conduction in the same ear. You also note â tympanic membrane mobility. Which of the following treatment options is most appropriate for this child at this time? A. Decongestants B. Oral antibiotics C. Reevaluation in 3 months D. Antihistamines

C. Reevaluation in 3 months (pts conductive hearing loss - evidenced by R/W test & decreased tympanic membrane mobility are suggestive of serous OM @ this time monitoring & re-eval in 3 mo is appropriate treatment)

A mother brings her 7 mo old infant daughter to your office b/c, over the past week, she's felt the daughter's lymph nodes in the neck "getting harder". Upon PE, you notice that the lymph nodes feel, small, round, & slightly hardened. Which of the following is the most likely cause? A. Pyelonephritis B. Lymphoma C. Respiratory infection D. Leukemia

C. Respiratory infection

A mother brings her 7-month-old infant daughter to your office b/c, over the past week, she's felt the daughter's lymph nodes in the neck "getting harder". Upon physical examination, you notice that the lymph noses feel small, round, & slightly hardened. Which of the following is the most likely cause? A. Pyelonephritis B. Lymphoma C. Respiratory infection D. Leukemia

C. Respiratory infection (The most likely cause is a respiratory infection b/c the infant is presenting w/shoddy lymph nodes, which often indicate a past infection. Although pyelonephritis is an infection, it is less likely than a respiratory infection to cause shoddy lymph nodes in the neck. Lymphoma would be more strongly indicated by enlarged lymph nodes, as opposed to shoddy lymph nodes. Leukemia would be characterized by generalized lymphadenopathy, not localized to one area.)

Tanya is 10 years old, 4'8" tall, & weighs 110 pounds. Tanya's mother is concerned about her weight & eating habits, as she has found junk food wrappers hidden around the house. She adds that Tanya it is very defiant when it comes to food restrictions. Which of the following is the most appropriate way of approaching the topic of diet with Tanya? A. Explain to Tanya that being overweight can cause type 2 diabetes, which can lead to major complications including amputations & early death B. Explain to Tanya that being overweight puts her at an increased risk for developing many types of cancer & heart disease C. Review the MyPlate standards with Tanya & help her understand why a balanced diet & maintaining a healthy weight are important D. Explain to Tanya that junk food, especially sweets, can lead to tooth decay

C. Review the MyPlate standards with Tanya & help her understand why a balanced diet & maintaining a healthy weight are important

You are conducting a WCE on a 9 mo old & discover that he has low hematocrit. His mother informs you that he drinks only goat's milk because he has trouble tolerating formula. A physical examination reveals tachycardia & pale skin. Based on your suspected diagnosis, which of the following laboratory test do you order initially? A. Peripheral smear B. Hemoglobin electrophoresis C. Serum iron tests D. ß-globin gene mapping

C. Serum iron tests (p. 133-4)

After which Tanner stage do males usually begin spermarche or nocturnal emissions? A. Stage 1 B. Stage 2 C. Stage 3 D. Stage 4

C. Stage 3

As a part of Priscilla's WCE, the NP is gathering information from Priscilla, age 13. The NP asks whether Priscilla has tried marijuana or engaged in sexual activity. The NP also asks whether or not Priscilla has any worries or stressors. Of the following types of data, which is the NP gathering? A. Objective data B. Management data C. Subjective data D. Clinical data

C. Subjective data

Which of these lab values would most likely be decreased in a patient w/hyperthyroidism? A. Free thyroxine index B. Serum antinuclear antibodies C. TSH D. Triiodothyronine

C. TSH

Of the following, which condition most often occurs in children between 10-20 years of age? A. Hypospadias B. Enuresis C. Testicular torsion D. Cryptorchidism

C. Testicular torsion (p. 150)

Which of the following types of congenital heart defects is most likely to be present w/right to left shunting? A. Transposition of the great arteries B. Coarctation of the aorta C. Tetralogy of Fallot D. Pulmonary stenosis

C. Tetralogy of Fallot

If a mother's HbsAg is unknown, the following steps would be taken w/her baby except: A. Administer hepatitis B vaccine within 12 hours of birth B. If a mother tests positive, hepatitis B immune globulin should be given no later than 1 week of age C. The 1st dose of the hepatitis B vaccine should be given at 6 months of age D. Check status immediately after birth

C. The 1st dose of the hepatitis B vaccine should be given at 6 months of age (p. 17)

The parents of a 6 mo old male infant come into your office. As of late, the baby has been fussy & drooling more than usual. The mother suspects that the child is teething & has questions regarding tooth eruption. Which of the following is most accurate regarding tooth eruption in children? A. Tooth eruption occurs in random order B. The lateral incisor is usually the 1st tooth to erupt C. The 1st tooth usually erupts @ 6 mo of age D. The primary teeth are usually in by 18 mo of age

C. The 1st tooth usually erupts @ 6 mo of age (p.7)

You were seeing an infant who presents with strabismus. In which situation would you be least likely to refer the patient to an ophthalmologist? A. The findings first presented at seven months of age B. The child presents with persistent hypertropia C. The child is four months of age D. The child presents with persistent hypotropia

C. The child is four months of age

Which of these factors is not a typical indication to recommend a prenatal evaluation for genetic disorders? A. The mother has experienced 2 stillbirths B. The father has a brother w/Down syndrome C. The father worked w/pesticides 3 years ago D. The mother is of African descent

C. The father worked w/pesticides 3 years ago (Exposure to toxic chemicals is a common indicator for genetic evaluation if the fetus was exposed but it was the father.)

Why must circumcision not be performed on pts w/hypospadias? A. It makes the pt susceptible to infection B. There is not foreskin to remove C. The foreskin is used in repair D. Complications often cause death

C. The foreskin is used in repair

Nolan, a 22-year-old, has lost the ability to control lateral eye movement. Which cranial nerve (CN) has most likely been affected?

CN VI (Abducens)

You recently diagnosed a 2yo female w/iron deficiency anemia. She has a Hgb level of 9.8 g/dL & the serum ferritin of 20 mcg/L. She presents w/increased labor of breathing, SOB & lethargy. Which of the following should be your next step in treating this patient? A. Treat w/2-3 mg/kg/d of elemental iron B. Order iron to be administered IV C. Treat w/3-6 mg/kg/d of elemental iron D. Order 1 unit packed red blood cells

C. Treat w/3-6 mg/kg/d of elemental iron (p. 134)

You are examining the newborn child of a recovering alcoholic. During the interview, she mentions she was still drinking early into her pregnancy & didn't know she was pregnant for some time. You examine the newborn's mouth. Which of the following findings would most strongly indicate fetal alcohol syndrome? A. Shrunken, contracted mouth B. Enlarged tongue C. Triangular upper lip D. Whitish-yellow cysts on gums

C. Triangular upper lip (A triangular upper lip with fish-like mouth, or the fish mouth deformity, is commonly observed in neonates & pediatric patients born w/fetal alcohol syndrome. Microstomia, on the other hand, presents as a shrunken, contracted mouth & is usually observed in trisomy 18 & 21. Macroglossia, or enlarged tongue, may be d/t hypothyroidism or mucopolysaccharidoses. Epstein pearls present as whitish-yellow cysts on the gums & roof of the mouth; these reals are common in newborns & are not considered an abnormal sign b/c they typically disappear w/in 1-2 weeks of birth.)

Jack, a school-aged child, has a 4-year-old brother who has just been diagnosed with acute lymphocytic leukemia (ALL). Jack should receive which of the following flu vaccine preparations? A. Inhaled influenza B. Quadrivalent inactivated influenza C. Trivalent inactivated influenza D. The child should not receive any influenza vaccine at risk of shedding the virus to the sibling

C. Trivalent inactivated influenza (The inhaled version is a live virus & should be avoided in families w/immunocompromised members. The trivalent inactivated influenza is a dead virus & can't shed the virus, this is the recommended version for patients under 18.)

According to Erik Erickson, the industry vs. inferiority stage occurs during school age. Which of the following would Piaget expect for appropriate cognitive development during this stage? A. Being active, energetic, & curious B. Magical thinking C. Understanding the concept of space D. Seeking rewards & approval

C. Understanding the concept of space (Concrete operational thinking - concepts of space, marked cognitive development in the school age years.)

Which of the following is a developmental warning sign in older school-age children? A. Height gain of only 2 inches a year B. Sleeps 10 hours a night C. Uses excuse of illness to get out of chores D. Weight gain of only 5 pounds a year

C. Uses excuse of illness to get out of chores (p. 49)

Regarding congenital heart defects, what would the NP recognize as not true? A. Left to right shunting occurs w/acyanotic defects B. Transposition of the great arteries, Tetralogy of Fallot, & tricuspid atresia are cyanotic defects C. Ventricular septal defect, pulmonary atresia, & patent ductus arteriosus are examples of acyanotic defects D. Right to left shunting occurs w/cyanotic defects

C. Ventricular septal defect, pulmonary atresia, & patent ductus arteriosus are examples of acyanotic defects (p. 64-5)

The NP would expect which of the following lab tests to be least useful in a work up for cystic fibrosis? A. Pilocarpine lontophoresis sweat test B. Pulmonary function tests C. WBC D. Sputum culture

C. WBC

All of the following accurately reflect the typical WCE except: A. After age 4 years, a child should have a WCE yearly B. Children on ADHD medication should see a physician or NP q6m C. WCE are arranged around immunizations schedules, which are the key purpose of the visit D. If the parents are experienced, their newborn does not commonly need to have a check-up until 1-2 weeks after birth

C. WCE are arranged around immunizations schedules, which are the key purpose of the visit (WCE commonly include PE to assess the physical well-being of a child; preventive care (immunizations).. communication w/the parents, developmental tracking, & personal family issues are all equally important aspects of the WCE. If no complications they should have a WCE yearly after age 4, if taking ADHD meds then they s/be seen q6m, after birth, it is recommended that the newborn be seen in 2-4 days unless the parents are experienced.)

Which of the following is not a developmental warning sign in younger school age children? A. Lack of social interaction B. Not paying attention in class C. Worrying about the opinions of peers D. Dropping in test score performance

C. Worrying about the opinions of peers

A 7 year old child arrives at your practice w/a radial fracture & bruising around the lower arm. When asked how he received the fracture, the child says he fell while riding his bike earlier that day, Suspecting abuse, you decide to undertake a more thorough examination. All of the following signs would increase your concern of child abuse except: A. The child appears shy w/a limited vocabulary B. Closer examination of the lower arm reveals thin, finger-like bruises C. You learn that the child exhibits poor academic performance D. You not that the child has "fallen off his bike" before & presented w/a slight concussion

C. You learn that the child exhibits poor academic performance

Acute onset. High fever and chills. Productive cough and large amount of green to rust-colored sputum. Pleuritic chest pain with cough. Crackles; decreased breath sounds, dull. CBC: leukocytosis; elevated neutrophils. Band forms may be seen.

CAP No. 1: Streptococcus pneumoniae

As a nurse practitioner, you know that which two cardiac enzymes are entirely cardio selective and only elevate when myocardial damage is present?

CK-MB and troponin I

Which cranial nerves does not control eye movements

CN II

Which of the following cranial nerves does not control eye movement? CN III CN II CN VI CN IV

CN II

On OLD Olympus Towering Tops, A Finn And German Viewed Some Hops

CN1: Olfactory: Smell CN 11: Optic: Vision CN III Oculomotor: Most EOMs, opening eyelids, pupillary constriction CN IV Trochlear: Down and inward eye movement CN V Trigeminal: muscle of mastication , sensation of face, cornea, scalp. Think cotton wisp to test corneal reflex CN VI Abducens: lateral eye movement CN VII Facial: Move face, close mouth and eyes, taste ( anterior 2/3), saliva and tear recreation . Test think puff checks CN VIIIAcoustic: hearing and equilibrium CN IX Glossopharyngeal: gag reflex, carotid reflex, swallowing, taste, phonation (one-third) CN X Vagus: Talking, swallowing, carotid reflex CN XI Spinal Accessory: shrug shoulders CN XII Hypoglossal: Move lounge

A 65-year-old noticed persistent, sharp, and painful spasms in her left cheek. She describes the pain as feeling like "an electric shock." Which would be the least appropriate treatment for this condition? Tricyclic antidepressants Calcium channel blockers Muscle relaxants Anti-seizure drugs

Calcium channel blockers

An older adult patient with congestive heart failure (CHF) presents for type 2 diabetes management. The patient has current glycosylated hemoglobin (HgbA1C) of 8.5% and takes metformin (Glucophage). The patient drives a truck for work and must avoid hypoglycemic episodes. Which medication would be an appropriate adjunct to manage diabetes in this patient?

Canagliflozin (Invokana)

A 23-year old male presents with inflammation and itching of the penile head. He reports being in a long-term, monogamous sexual relationship and states that he knows that he and his partner have been faithful for their entire relationship. Which of the following is the most likely diagnosis and treatment?

Candida balanitis; topical antifungal

A 27-year-old female comes to your office and explains that she has recently felt an itching and inflammation in her vagina, despite being sexually inactive for several months. Upon physical exam, you note vulvovaginal erythema. In addition, you discover a thick, white, curd-like discharge. Which of the following is she most likely experiencing?

Candidiasis

Patients are at increased risk for what fungal infection when using the sponge?

Candidiasis

A 62-year-old patient has recently been diagnosed with heart failure. With respect to the standard of care, the best treatment regimen for this patient would include nonpharmacologic steps, such as sodium restriction and weight reduction, alongside pharmacologic measures, such as which of these drugs?

Captopril (ace mainstay for HF)

During auscultation of a newborn, a grade IV murmur is heard with a holosystolic thrill at the left lower sternal border. You suspect a congenital heart defect and order an x-ray. What findings would be most consistent with a suspected diagnosis of ventricular septal defect?

Cardiomegaly with increased pulmonary vascular markings

Which of the following conditions is least associated with causing spontaneous abortions in the second trimester of pregnancy? a. Chromosomal abnormalities b. Cervical incompetence c. Infection d. Uterine abnormalities

Chromosomal abnormalities

All of the following factors are associated with an increased incidence of placenta previa except: a. Chronic hypertension b. Previous uterine surgery c. Multiparity d. Malpresentation

Chronic hypertension

Most common leukemia in adults

Chronic lymphocytic leukemia CLL

The results of a patient's chromosome analysis show the presence of the Philadelphia chromosome. What type of leukemia does this finding most often indicate?

Chronic myelogenous leukemia

A 66-year old female has a history of varicose veins and presents to your office complaining of aching and water retention in her legs during the last few months. She is also experiencing night cramps in her lower extremities. Which of the following disorders best characterizes her symptoms?

Chronic venous insufficiency

Which cardiovascular disorder is best defined as impaired venous return due to either destruction of valves, leg trauma, or sustained elevation of venous pressure?

Chronic venous insufficiency

A patient comes to you with complaints of a "bad headache." He is not able to clearly articulate any other signs or symptoms. Which of the following features is most important for you to assess to determine the type of headache?

Chronology

A patient recently returned from a trip to Africa and is experiencing 10 to 12 loose stools/day. The patient takes metformin every morning and otherwise is in good health. Which medication will the nurse practitioner prescribe to treat the diarrhea?

Ciprofloxacin (Cipro) in a single dose

A 45 year old patient presents to the clinic complaining of weak arms, pins and needles, and fingers that can't keep a pen steady. A blood panel shows that his hemoglobin, hematocrit and red blood cell count are decreased, whereas his mean corpuscular volume is increased. Which follow-up diagnostic would be least helpful in confirming the most likely condition? A. Citrate agar gel electrophoresis B. Anti-intrinsic factor test C. Schilling test D. Antiparietal cell antibody test.

Citrate agar gel electrophoresis

H. pylori triple therapy

Clarithromycin + Amoxicillin + PPI *(CAP)*

Pioglitazone (Actos) is contraindicated in patients with which of the following conditions?

Class III or IV heart disease

According to the New York Heart Association Functional Classification system, if a heart failure patient has marked limitations of physical activity but is comfortable at rest, which classification of the disease best applies to the patient?

Class Ill

Susan, age 67, has been experiencing difficulty planning and organizing her weekly errands. She frequently has trouble articulating her thoughts and putting sentences together, and struggles with remembering how to cook her favorite meals. Which of the following medications would be least helpful in treating Susan? a. Tacrine hydrochloride b. Clonazepam c. Donepezil d. Galantamine

Clonazepam

Your patient comes to the clinic with a blood pressure reading of 184/112 mmHg, but is otherwise asymptomatic. Your clinic does not have the appropriate medication to treat his condition, so you refer him to the local ER. Which medication would you most expect your patient to receive at the ER?

Clonidine

Considering a likely diagnosis of peptic ulcer disease, which of the following findings would be most indicative of a bleeding duodenal ulcer?

Coffee-ground emesis

Which statement is true regarding collaborative practice agreements for nurse practitioners?

Collaborating physicians define their relationship with a nurse practitioner through a collaborative practice agreement.

During his check-up, Gary, a 66-year-old male, reports thin stools and recurring bouts of both constipation and diarrhea. He has also experienced rapid, unexplained weight loss in the last month (around 5% of his body weight). Gary is concerned because he has a family history of cancer. Of the following, which procedure would you order to best rule out colon cancer?

Colonoscopy

Which of these headaches occurs as a result of vasodilation and excessive pulsation of the external carotid artery branches but does not typically present with an aura?

Common migraine

Describe a cohort study

Compares a particular outcome in groups of individuals who are alike in many ways but differ by certain characteristics.

A patient complains of feeling anxious for no reason. He states he does not think he has panic attacks, but there will be moments of excessive worry about minute things seemingly triggered by nothing. He complains of not being able to breathe at moments and has a "tight feeling" in his chest. After these brief moments, the patient states that feels fatigued. Which of the following should be your lowest priority to order? CBC EKG FBG TSH

Complete blood count

A patient with a seizure disorder as he suddenly stops talking, sits down and stares straight ahead. During the seizure, which lasts approximately two minutes, he makes chewing movements and repeatedly pulls at his shirt. After the seizure, the patient cannot recall the onset of the seizure or describe what happened to him. What type of seizure should the practitioner document?

Complex partial

Jamie, a 4-year-old male, presents to the clinic in a sleepy, afebrile state. His mother mentions that, after his afternoon nap, he started screaming in their brightly-lit den and began vomiting. When she tried to talk to him, he spoke nonsense. When you talk to Jamie, he rouses, exhibits normal speech, and says he feels fine. What is the most likely source of Jamie's symptoms?

Confusional migraine

Possible cause of dementia

Cortical atrophy

A hockey player presents with pain around the sternum that worsens whenever he takes a deep breath. He says his symptoms began nearly a week ago after receiving a hard blow to the chest during a game, and that the pain has been getting progressively worse. for which condition should the patient most likely be first evaluated?

Costocondritis

Which of the following tasks is most pertinent in assessing a patient's attention in a mini-mental status exam? a. Repeating 3 objects b. Counting backwards from 100 in serial 7s c. Recalling 3 objects 5 minutes later d. Copying a design

Counting backwards from 100 in serial 7s

A 27-year-old woman presents to the emergency room with vaginal bleeding. She states that she is 14 weeks pregnant and has not had any prior bleeding or spotting. Of the following signs, which would be the most serious in regard to the patient's pregnancy?

Cramping and pelvic pressure

What effect does Depo-Provera have specifically on the endometrium?

Creates a thin, atrophic lining

What is the name of the condition in which the foreskin is not retractable over the tip of the penis? A. Epispadias B. Chordee C. Hypospadias D. Phimosis

D Phimosis (Phimosis presents w/foreskin that is not retractable over the tip of the penis. This condition is considered common in infant patients & is often resolved by 3 years of age. Hypospadias results in the opening of the urethra manifesting on the underside, rather than the tip of the penis; epispadias similarly affects the urethra, causing it to open on the top or side of the penis. Chordee refers to curvature of the penis at the junction of head & shaft.)

Your 68-year-old patient has a blood pressure reading of 145/82 mmHg. As a nurse practitioner, you know that these readings can be caused by certain conditions and tests should be ordered to rule out other causes. Which of the following tests would you be likely to order at this time?

D-dimer

In considering the need to refer a child to a specialist for cystic fibrosis, all of the following statements from the mother would typically support the decision except: A. "My child's stool is often bulky & foul smelling w/an oily residue" B. "My child doesn't seem to be growing. Am I not breastfeeding her enough?" C. "My child always seems to coughing & has a runny nose that won't quit." D. "My child wets his diaper 6 times a day & nearly soaks it through each time."

D. "My child wets his diaper 6 times a day & nearly soaks it through each time."

What physical exam findings would lead you to diagnose a toddler w/allergic rhinitis? A. Nasal quality voice B. Orbital edema C. Fissures @ lip corners D. Mouth breathing

D. Mouth breathing (p. 40)

Hector, an infant born to a working-class couple, has been diagnosed with phenylketonuria (PKU), a known inborn error of metabolism. While PKU is treatable, the diagnosis of this condition has nonetheless caused his parents to believe that they are cursed. They ask you what the occurrence of PKU in the general population is. Which of these most clearly elucidates the designated occurrence of PKU? A. 1:500-1,000 B. 1:1,000-3,000 C. 1:5,000-9,000 D. 1:10,000-25,000

D. 1:10,000-25,000 (p. 32)

Dan & Kara bring their toddler son, Ian, to your office b/c they are worried about his feeding habits They explain that they still have to feed him b/c he is unable to use utensils — even a spoon — on his own. As the NP, you know that at what earliest age would Ian have to be for this issue to be a developmental concern? A. 12 months B. 15 months C. 17 months D. 20 months

D. 20 months (This control s/be gained by 18 months)

When should the rotavirus not be administered to an infant? A. 24 weeks of age B. 16 weeks of age C. 8 weeks of age D. 32 weeks of age

D. 32 weeks of age

You are examining a prepubescent child w/cafe au lait macules. You suspect that she may have neurofibromatosis-1. Which of the following physical findings would lead you to suspect this? A. 4 cafe au lait macules measuring 5 mm, & 2 cutaneous neurofibromas B. 6 cafe au lait macules measuring 4 mm, & 2 cutaneous neurofibromas C. 3 cafe au lait macules measuring 6 mm, & 3 cutaneous neurofibromas D. 6 cafe au lait macules measuring 6 mm, & 2 cutaneous neurofibromas

D. 6 cafe au lait macules measuring 6 mm, & 2 cutaneous neurofibromas

Which of the following physical findings would you least expect in individual w/trisomy 21? A. Hypotonia B. A single palmar crease C. The appearance of hypertelorism D. A "shield" shaped chest

D. A "shield" shaped chest (p. 22)

There are several reflexes that initiate in the newborn period. Which of the following is most indicative of a baby is exhibiting normal development? A. A 24 mo old whose big toe points up, w/other toe fanning out B. A 6 mo old who has just been able to watch a person's face intently C. An 8 mo old who started holding his head steady when pulled to sit last month D. A 4 mo old infant who lifts his head & chest when on his stomach

D. A 4 mo old infant who lifts his head & chest when on his stomach (p. 11)

When examining the vision of an infant, which finding would be most indicative of Tay-Sachs disease? A. Brushfield spots B. Ectopia lentis C. Iridodonesis D. A cherry red macula

D. A cherry red macula (p. 23)

You are performing an eye exam on a newborn child. When you perform the red reflex test, a black spot presents instead, Which of these conditions would this finding typically indicate? A. Optic nerve hypoplasia B. Retinoblastoma C. Heterochromia D. Absence of a clear retinal pathway

D. Absence of a clear retinal pathway (The red reflex replaced by a black spot commonly indicates that there is no clear pathway from the lens to the retina & is a crucial finding for congenital contra acts, corneal scar, or ocular hemorrhage. Optic nerve hypoplasia, on the other hand, is a midline defect of the CNS & may present s/colobomas. Heterochromia simply refers to a difference in coloration of the iris & usually does not indicate an underlying condition. Further, retinoblastoma & congenital cataracts may ne indicated by the red reflex being replaced by a whitish color, not a black spot.)

The parents of a 16yo patient are concerned that their child may be depressed. The parents state that their child has increasingly isolated himself & has an overall sad demeanor. While interviewing the child, which of the following guidelines for effective communication is necessary? A. Asking the parent to repeat to you what the school counselor has told them B. Taking a lead in the conversation & stay on topic C. Telling the parent the exact details goal resolution D. Active listening skills & therapeutic response

D. Active listening skills & therapeutic response

Which of the following is not an indication for genetic evaluation? A. Fetal exposures to medication B. History of miscarriages & stillbirths C. Family history of birth defects D. Advanced paternal age 30

D. Advanced paternal age 30 (p. 21)

You are taking over the case of a 2yo girl w/a 5 day history of fever; erythema on her palms & soles; & erythematous, cracked lips. In addition, a physical examination indicates a strawberry tongue & cervical lymph nodes that are palpable & 2 cm in diameter. Based on the most likely diagnosis, which of the following findings would you most expect? A. A shortened PR interval B. A decreased erythrocyte sedimentation rate C. Bradycardia D. An increased C-reactive protein

D. An increased C-reactive protein (p. 69-70)

Pyloric stenosis is characterized by which type of abdominal mass? A. A lump in the lower abdomen B. Multiple sausage shaped masses around the abdomen C. A sausage shaped mass in the RUQ D. An olive shaped mass in the RUQ

D. An olive shaped mass in the RUQ (p. 74)

Your 9yo patient is brought to the clinic with a 3 day history of sneezing, runny nose, & coughing. His temperature is 103F & his cough produces yellow-tinged mucous. His father reports that the child has been very tired & has been shaking because of overwhelming chills. You hear crackles during auscultation & order an x-ray. The x-ray indicates patchy infiltrates & pleural effusion. Which of the following would be the most likely diagnosis & pathogenic cause? A. Bronchiolitis; respiratory syncytial virus B. Bacterial pneumonia; Streptococcus pneumoniae C. Bronchiolitis; mycoplasma D. Bacterial pneumonia; Escherichia coli

D. Bacterial pneumonia; Escherichia coli (The patient's findings & chest x-ray indicate bacterial pneumonia caused by Escherichia coli. Coughing in bronchiolitis is generally non-productive, & patients typically have a mild fever. Additionally, an x-ray showing bronchiolitis will have areas of scattered consolidation rather than patchy infiltrates in this case. Infections with S. pneumonia presents with lobar consolidation, not patchy infiltrates, on an x-ray.)

Jeanette, age 14, comes to your office for her annual checkup accompanied by her mother. You observe that Jeanette has stained teeth, calloused knuckles, & hoarseness in her voice. Upon physical examination, you also note that she has lost weight & presents w/decreased vital signs & anemia. Which of the following is the most likely diagnosis? A. Substance abuse B. Personality disorder C. Anorexia nervosa D. Bulimia nervosa

D. Bulimia nervosa (p. 60-1)

Which cranial nerve controls hearing & equilibrium? A. CN II B. CN IV C. CN VI D. CN VIII

D. CN VIII

Kussmaul's breathing is a finding most commonly associated with which condition?

Diabetic ketoacidosis

During auscultation of a newborn, a grade IV murmur is heard w/a holosystolic thrill @ the LLSB. You suspect a congenital heart defect & order an x-ray. What findings would be most consistent w/a suspected diagnosis of ventricular septal defect? A. Normal chest x-ray B. Cardiomegaly w/rib notching C. Boot-shaped heart D. Cardiomegaly w/increased pulmonary vascular markings

D. Cardiomegaly w/increased pulmonary vascular markings

A 10 yo male w/a recent history of group A Beta-hemolytic streptococcus infection currently has a fever of 103. Which of the following findings is most consistent w/a diagnosis of rheumatic fever? A. Elevated acute phase reactants B. Arthralgia without objective inflammation C. Prolonged interval between the P-wave to the beginning of the QRS complex D. Carditis

D. Carditis

You are examining a newborn's head after a full term vaginal delivery, without complications, from a mother who does not have a significant medical history. The newborn is alert and crying, the anterior fontanelle is full, and the scalp feels loose & slightly edematous. The blood does not seem to increase with crying & does not cross the midline, which is palpable. What is the most probable diagnosis? A. Microcephaly B. Macrocephaly C. Caput succedaneum D. Cephalohematoma

D. Cephalohematoma (A diagnosis of cephalohematoma, the collection of blood between the surface of the cranial bones & the periosteal membrane, is most appropriate because the blood does not cross the patient's midline. Caput succedaneum is often the result of a long & difficult labor or vacuum extraction, &, in this case, blood would be seen crossing the suture lines. Microcephaly & macrocephaly refer to cranial circumference, of less or greater than two standard deviation's, respectively. These are not reported in this patient.)

Kevin, 3yo male, is brought to your clinic. Upon evaluation, you determine that he has poor motor, language, & social skills. His parents state, however, that until very recently, Kevin's communication skills & social relationships were much better & age-appropriate. Which of the following disorders does he most likely have? A. Asperger's syndrome B. Rett syndrome C. Down syndrome D. Childhood disintegrative disorder

D. Childhood disintegrative disorder

Johnny, age 3, is brought into the clinic by his parents, who state that he has been unable to perform certain skills that he used to be able to perform. They also say that his ability to use language has started to deteriorate, & he doesn't want to play w/the other childrean @ school. Which of the following conditions would be the most likely diagnosis? A. Autistic disorder B. Asperger's syndrome C. Rett's disorder D. Childhood disintegrative disorder

D. Childhood disintegrative disorder (Based on the age of the patient & the recent decline in multiple areas of functioning.)

A 7 yo patient is brought to you complaining of difficulty swallowing & drooling. During the examination you note that the epiglottis is inflamed. Which of the following pathogens is usually not associated with this type of inflammation? A. Streptococci B. Haemophilus influenzae C. Pneumococci D. Chlamydia trachomatis

D. Chlamydia trachomatis (Chlamydia trachomatis may cause pharyngitis, not epiglottitis. Pneumococci, streptococci, and Haemophilus influenzae are all common causes of epiglottitis.)

An infant presents with cyanosis in the lower extremities. Upon for the exam, you know that the patient has a higher pulse rate and blood pressure in the upper extremities in comparison to his lower extremities. What is the most probable cause of these findings? A. Cutis marmorata B. The patent ductus arteriosus did not close properly C. Harlequin color change D. Coarctation of the aorta

D. Coarctation of the aorta

Jamie, a 4 yo male, presents to the clinic in a sleepy, afebrile state. His mother mentions that, after his afternoon nap, he started screaming in their brightly-lit den & began vomiting. When she tried to talk to him, he spoke nonsense. When you talk to Jamie, he rouses, exhibits normal speech, & says he feels find. What is the most likely source of Jamie's symptoms? A. Viral meningitis B. Brain tumor C. Neurofibromatosis D. Confusional migraine

D. Confusional migraine

Which of the following is not a finding of DiGeorge syndrome? A. Short palpebral fissures B. Lateral displacement of the inner canthi C. Hypoparathyroidism with hypocalcemia D. Decreased muscle tone

D. Decreased muscle tone (p. 24)

There is concern at your clinic that some children with developmental delays are not being identified soon enough. You are examining a 6yo male. Of the following, which is the best tool to identify risk for developmental delay in this child? A. Cognitive adaptive test B. M-CHAT C. Ages and stages questionnaire D. Denver II

D. Denver II

Michael, a 5 yo pt, is experiencing involuntary urination during waking hours. His parents point out that Michael historically exhibited bladder control until 2 weeks ago. As he does not demonstrate any signs of psychological distress, you suspect Michael's condition may be neurogenic in origin. How would you best classify Michael's enuresis? A. Primary enuresis B. Nocturnal enuresis C. Functional enuresis D. Diurnal enuresis

D. Diurnal enuresis

A 1 month old baby girl presents with a swollen eyelid with purulent discharge. The NP notes red inflammation on the eyelid. Of the following, which is the best treatment for this patient? A. Oral antihistamines B. Allergist referral C. Ophthalmologist referral D. Erythromycin ointment

D. Erythromycin ointment

What finding is not a classic characteristic of a 3 month old w/congenital heart disease & signs of heart failure? A. Failure to thrive B. Irritability C. Rales upon lung auscultation D. Frequent vomiting w/feeding

D. Frequent vomiting w/feeding

Whic of these treatments is primarily recommended for mgmt of tinea capitis? A. Oral acyclovir B. Aluminum subacetate solution C. Need shampoo D. Griseofulvin

D. Griseofulvin

Which of the following pathogens is most commonly responsible for epiglottitis? A. Neisseria gonorrhoeae B. Streptococcus pneumniae C. Moraxella catarrhalis D. Haemophilus influenzae

D. Haemophilus influenzae

A 15yo male presents w/ c/o general malaise & fatigue for the past few wks w/o resolution. He has also had a fever & sore throat. All of the following exam findings would support the diagnosis of infectious mononucleosis except: A. Diffuse cervical lymphadenopathy B. White exudate on tonsils C. Maculopapular rash D. Hepatomegaly

D. Hepatomegaly (Splenomegaly, not hepatomegaly, is found in infectious mononucleosis. All the other listed exam findings are indicative of infectious mononucleosis, along w/an early rise in immunoglobulin M, & a permanent rise in immunoglobulin G.)

A mother brings in her 2-year-old child for a routine follow-up as well as immunizations. The child weighed 3.45 kg @ birth & was 20" long. Which of the following is the most accurate statement regarding her physical development? A. Her current height should be about 1/3 her adult height B. Her birth weight was appropriate for her gestational age C. Her birth weight would be categorized as below average D. Her current weight should be about 14 kg today

D. Her current weight should be about 14 kg today (p. 6)

A 4-year-old girl c/o localized, painful swelling of her left eyelid. Her parents say that an abscess on her eyelid seems to be causing the swelling & redness. Your examination confirms the parents' suspicions concerning the abscess. Which of the following would be the most likely cause? A. Chalazion B. Blepharitis C. Conjunctivitis D. Hordeolum

D. Hordeolum

A 1 mo male is brought to the clinic or a regular check-up. He exhibits irritability & fever of 100.2, & shows signs of dehydration & documented weight loss. A urinalysis is positive for leukocytes & nitrites. Given the most likely diagnosis, what is the 1st step that s/b taken in treating his underlying condition? A. Home course of oral cephalosporins B. Surgical intervention C. Oral desmopressin D. Hospitalization

D. Hospitalization

When using the Weber & Rinne tests to screen for hearing loss, which of the following statements is true? A. In sensorineural hearing loss, sound lateralizes to the unaffected ear with the Rinne test B. Normal findings w/the Weber test include an air conduction rate > the bone conduction rate C. In conductive hearing loss, air conduction > bone conduction is expected w/the Rinne test D. In conductive hearing loss, sound lateralizes to the affected ear during the Weber test

D. In conductive hearing loss, sound lateralizes to the affected ear during the Weber test (p. 105)

A 5 yo male presents w/weakness, muscle fatigue, & arthralgias. A PE reveals dry skin, diminished heart sounds, & diminished DTRs. Suspecting hypothyroidism, you order a series of lab studies. Which of following findings would confirm a dx of hypothyroidism? A. Low liver enzymes B. Low serum cholesterol C. Elevated T3 D. Increased TSH

D. Increased TSH

The Denver II assessment test commonly measures a child for all of the following except: A. Personal-social development B. Language C. Fine motor development D. Intelligence

D. Intelligence (Although the Denver II assessment test measures several aspects of child development, it is NOT an intelligence test. It measures language, personal-social dev, fine motor, & gross motor dev.)

A child's serology results come back showing Anti-HCV & HCV RNA. Which of the following may be included in his treatment plan? A. Oxycodone B. Trimethoprim-sulfamethoxazole C. Erythromycin D. Interferon & ribavirin

D. Interferon & ribavirin

A 6 mo old male is brought to your clinic w/complaints of vomiting. His mother says that her some was fine, but then "he just got really irritable & started crying, & then just started throwing up green barf". The mother says that she noticed her son has not been as energetic as usual, & that there was "jelly-like poop" when changing his diaper. Based on these findings, which GI disorder would you most likely suspect? A. Gastroenteritis B. Hirschsprung's Dz C. Pyloric stenosis D. Intussusception

D. Intussusception

The parents of 9mo Kent comes to your office, hurried & during the middle of the night. The parents state that their child is lethargic & constantly cries out despite all attempts to comfort him. Upon exam, the NP notes palpable sausage-shaped mass in the RUQ & abdominal distention. When asked about the child's stool, the parents stated that his diapers were jelly like. Of the following, which is the most likely diagnosis? A. Appendicitis B. Pyloric stenosis C. Malabsorption D. Intussusception

D. Intussusception

The acronym SAFETEENS provides a checklist of preventive care topics for NPs. Going by this checklist, which of the following is not a significant concern in preventive care? A. Sexual abstinence B. Suicide C. School performance D. Involvement in hobbies

D. Involvement in hobbies (SAFETEENS - S - sexuality, A - accidents, F - firearms/violence, E - emotions (suicide), T - toxins, E - environment (school perfo, E - exercise, N - nutrition, S - shots

A 24 month old Caucasian baby brought in by her mother so says the child is "unusually tired". She has no history of chronic illness. After interviewing the mother, the pediatric NP learns that she was switched to whole milk at 12 months of age. Currently she is drinking 60 ounces per day & is not interested in solid foods. Upon physical exam, you notice a pale conjunctiva, and lab results reveal microcytic hypochromic anemia with normal blood white blood cell, platelet, & reticulocyte counts. What is the most likely diagnosis? A. Vitamin B12 deficiency B. Alpha-thalassemia minor C. Pernicious anemia D. Iron deficiency anemia

D. Iron deficiency anemia

Shortly after a routine, healthy delivery of a male newborn, you are determining the necessary tests & exams to be administered. Mandatory screening for this child would include all of the following except: A. Galactosemia B. Hypothyroidism C. Phenylketonuria screening D. Iron deficiency anemia

D. Iron deficiency anemia (p. 31)

All of the following are findings of pre-icteric hepatitis except: A. Fatigue B. Anorexia C. Malaise D. Jaundice

D. Jaundice (p. 78)

A concerned mother brings her 18 month old child to you with his feet conspicuously turned inward. Which of the following is not included among the differential diagnoses? A. Metatarsus adductus B. Adducted great toe C. Femoral anteversion D. Legg-Calve-Perthes disease

D. Legg-Calve-Perthes disease (Legg-Calve-Perthes disease (LCPD) is an aseptic or avascular necrosis of the femoral head. Physical findings include limited passive internal rotation & abduction of the hip joint & do not include inward pointing feet. Inward turned feet may be attributed to femoral anteversion, metatarsus adductus, and an adducted great toe but not LCPD.)

Which of these genetic disorders is most likely to lead to blindness in later life? A. Turner's syndrome B. DiGeorge syndrome C. Kleinfelter's syndrome D. Marfan syndrome

D. Marfan syndrome (This is b/c of ectopia lentis & the increased chance of retinal detachment & glaucoma which can lead to blindness.)

Which of the following physical features is not characteristic of a newborn with Down's syndrome? A. Unruly hair B. Brushfield spots C. Wide fontanelle D. Micrognathia

D. Micrognathia (p. 29)

You are assessing a toddler's fractured right radius. There is minimal swelling @ the fracture site & no bruising. Other than the fracture, a physical exam did not reveal anything other than the obvious pain the child felt @ the site of the fracture. The mother is perplexed as to how the fracture happened. You suspect abuse but also consider a differential diagnosis that could account for this toddler's symptoms. Which of the following could most explain the child's fracture? A. Avascular necrosis B. Osteoporosis C. Sickle cell anemia D. Osteogenesis imperfecta

D. Osteogenesis imperfecta

The 1st NP programs were initiated, primarily, because of what issues in healthcare at that time? A. Hospital restructuring B. Managed-care C. Indigent patient needs D. Pediatric physician shortages

D. Pediatric physician shortages (p. 186)

Which of the following newborn screening tests are required by law in all 50 states? A. Galactosemia, HIV, and phenylketonuria B. Galactosemia, hemoglobin disorders, & hepatitis C. Sickle cell disorders, phenylketonuria & vision scan D. Phenylketonuria, hypothyroidism, sickle cell, & galactosemia

D. Phenylketonuria, hypothyroidism, sickle cell, & galactosemia

What is the name of the condition in which the foreskin is not retractible over the tip of the penis? A. Epispadias B. Chordee C. Hypospadias D. Phimosis

D. Phimosis

Which of the following skin variations that occurs in newborns would be of most concern? A. Milia B. Hemangioma C. Erythema toxicum D. Port wine stain

D. Port wine stains (Port wine stains are vascular birthmarks consisting of superficial & deep dilated capillaries that cause permanent reddish to purplish discoloration of the skin & may signify neurologic concerns. Milia are Pinpoint white papule on the face that spontaneously disappear w/in 3-4 weeks of like. A hemangioma is characterized by raised, soft, red lumps on the skin & is not usually a concern, Erythema toxicum is a benign newborn rash.)

You determine that a 12 yo male presents w/testicular torsion. Your tx priorities should include: A. Recommend cilostazol to improve blood flow B. Write release from physical education for the next 5 days C. Advise to make appt w/urology D. Refer to the ED for immediate surgical evaluation

D. Refer to the ED for immediate surgical evaluation

Metformin is not recommended as a 1st line therapy for DM2 patients presenting w/which of the following? A. Children < 10 yo B. Hypoglycemia C. Polydipsia D. Renal failure

D. Renal failure (Patients w/hepatic or renal failure s/not be prescribed metformin, which is commonly used to control high blood glucose & reduce gluconeogenesis in pts w/DM2. Metformin is known to upset the GI tract & exacerbate lactic acidosis. Metformin doesn't generally produce or exacerbate hypoglycemia in DM pts. Polydipsia may present as a symptoms in DM1 or DM1 pts, but is not a contraindication for metformin.)

In a pt w/tic disorders, all of the following presentations would typically be recognized as a simple tic except: A. Blinking for 30 seconds w/o interruption B. Extensive clearing of the throat C. Jerking the head back & forth D. Repeating a word another person said

D. Repeating a word another person said

What the following heart sounds is known as the "Kentucky" gallop? A. S2 B. S4 C. S1 D. S3

D. S3 (p. 62)

Which of these heart sounds would most strongly indicate a cardiac disorder in a pediatric patient? A. S1 B. S2 C. S3 D. S4

D. S4

The process of developmental surveillance includes eliciting & attending to parental concerns, obtaining a relevant developmental history, & making accurate & informative observations of children. Of the following, which best accomplishes this task? A. Using standardized growth charts B. Using standardized questionnaires C. Using clinical impressions primarily D. Screening process instruments

D. Screening process instruments (The screening process is best done by using developmental screening instruments; these are brief, standardized tests used to identify children who require further in-depth examination. When physicians use on my clinical impressions, such as an interview, estimates of children's developmental status are often inaccurate. Standardized questions & growth charts are part of developmental screening instruments that are utilized, but these should be used along with other screening instruments for a more comprehensive gathering of information.)

A child presents with symptoms of nausea, vomiting, & diarrhea which you determine to be the result of an acute inflammation of the gastric mucosa. After a diagnostic culture, you determine that the cause is not viral, so you decide to test for bacterial agents. Of the possible bacterial pathogens, you most respect to find which of the following bacteria? A. Salmonella, E. coli, & beta Streptococci B. Clostridium, Giardia, & E. coli C. Shigella, botulism, & H. pylori D. Shigella, Clostridium, & E. coli

D. Shigella, Clostridium, & E. coli

A febrile 5 yo patient w/sickle cell anemia has continued bed wetting. You order a urinalysis. Which of the following findings would you expect? A. Proteinuria B. Red blood cells C. Leukocyte esterase D. Specific gravity of 1.008

D. Specific gravity of 1.008 ( The urinalysis for a child w/ sickle cell anemia is usually hypoconcentrated, as indicated by a urine specific gravity of 1.008. Presence of WBCs, leukocyte esterase, or RBCs would not be expected w/sickle cell anemia. Proteinuria may be found in adult patients w/sickle cell anemia b/c of sickle cell neuropathy, but in pediatric sickle cell dz patients typically have normal renal function.)

Which of the following authorizes the Board of Nursing of the respective state to establish statutory authority? A. Standards of Advanced Practice B. Drug Enforcement Agency C. Department of Health and Human Services D. State practice acts

D. State practice acts

During a routine pediatric physical exam, you hear a murmur in a 12-year-old boy. You hear it best between the left lower sternal border & the apex of the heart. The murmur is "musical" or "vibratory" as opposed to harsh. Which of the following is the most likely cause of this murmur? A. Pulmonic stenosis B. Venous hum C. Aortic stenosis D. Still's murmur

D. Still's murmur

Which of the following components of Continuous Quality Improvement monitors the numbers of qualified staff, equipment, & resources? A. Establishments B. Appropriateness C. Processes D. Structures

D. Structures

You expect your patient has chancroid but know that a definitive dx is difficult. Which statement of morphological dx is true?

Diagnosis requires ID of chancroid causing bacteria but sensitivity is less than 80%

Which test would provide the most definitive diagnosis of neuroblastoma? A. Physical exam B. Low levels of urine catecholamines C. Abdominal CT scan D. Surgical biopsy

D. Surgical biopsy (A sx biopsy is the most definitive dx for neuroblastoma. Elevated urinary catecholamine levels, rather than low levels, may be used to dx neuroblastoma. Imaging studies, such as CT & physical exams, may strongly suggest the presence of this condition, but are unable to confirm a dx.)

Which of these genetic disorders is not associated w/cardiac defects? A. DiGeorge syndrome B. Marfan syndrome C. Turner's syndrome D. Tay-Sachs disease

D. Tay-Sachs disease (DiGeorge: aortic arch anomalies; Marfan: mitral valve prolapse, aortic regurgitation, & aortic dissection; Turner's: coarctation of the aorta.)

Which of the following conditions is least likely to present as asymptomatic in males? A. Gonorrhea B. Cryptorchidism C. Chlamydia D. Testicular torsion

D. Testicular torsion (Testicular torsion is not likely to present as asymptomatic, & is often heralded by sudden, severe pain in one testicle. Gonorrhea, cryptorchidism, & chlamydia are all often asymptomatic in males.)

Jason, age 8 years, is going to South America on vacation w/his parents. His parents want to know which vaccinations he needs to keep from getting sick. Assuming he is caught up on all of his other vaccinations, which of the following vaccines would be least necessary for Jason? A. Hepatitis A vaccine B. Typhoid vaccine C. Malaria vaccine D. Tetanus, diphtheria, acellular pertussis vaccine

D. Tetanus, diphtheria, acellular pertussis vaccine (Since this vaccine will normally protect the patient for 10 years, since they are up to date, this vaccine would not be needed.)

Regarding cardiac assessment, which of the following is true? A. The S1 heart sound is c/b opening of the mitral & tricuspid valves B. The S2 heart sound is c/b closure of the mitral & tricuspid valves C. The S1 heart sound is c/b opening of the aortic & pulmonic valves D. The S2 heart sound is c/b closure of the aortic & pulmonic valves

D. The S2 heart sound is c/b closure of the aortic & pulmonic valves (p. 62)

Mary brings her 2m old son, Parker, for A WCE. Mary has been breastfeeding Parker since birth, & Parker has been growing @ a rate of about 31 g/day. Mary wonders whether she is producing enough milk for her baby & would like info about switching to formula. Which of the following is the most appropriate advice to give Mary? A. If Mary feels that she is not producing enough milk, she should include vitamin D & B12 supplements for the baby, but she does not need to switch to formula. B. Mary should follow the AAP guidelines & continue breastfeeding. C. Parker could be suffering from malnutrition, but further tests are necessary to confirm this. D. The best way to confirm that Parker is receiving adequate nutrition is by his weight gain. According to his growth chart, he is in an appropriate rage for his age.

D. The best way to confirm that Parker is receiving adequate nutrition is by his weight gain. According to his growth chart, he is in an appropriate rage for his age.

All of these statements are true of slipped capital femoral epiphysis except: A. The condition typically occurs in girls prior to menarche B. The condition usually occurs w/o sudden severe force or trauma C. There is an increased incidence of the condition amongst obese adolescents D. The condition is more common in males & Caucasian adolescents

D. The condition is more common in males & Caucasian adolescents (SCFE is most common in male & AA, not Caucasian, adolescents. Female adolescents are also likely to experience the condition during growth spurts & prior to menarche. SCFE also shows a higher incidence amongst children who are obese or who have sedentary lifestyles & often presents w/o sudden force or trauma.)

Which of the following statements is most true regarding sinusitis in young children? A. Sinusitis occurs in children beginning @ 6 years of age or older B. Sinusitis is distinguished from acute otitis media by its typical pathogens C. Radiological studies are the most common method of confirming a diagnosis D. The paranasal sinuses are the most commonly affected sinuses

D. The paranasal sinuses are the most commonly affected sinuses (In young children, the 2 sinuses most commonly affected by sinusitis are 2 of the paranasal sinuses: the maxillary & ethmoid sinuses. Sinusitis can occur in children < 6 yo b/c the ethmoid & maxillary sinuses are present @ birth. Sinusitis can't be distinguished from AOM by its typical pathogens, as sinusitis shares the same typical pathogens w/AOM, such as Streptococcus pneumoniae, Haemophilus influenzae, & Moraxella catarrhalis.

Sharon, age 16, has just received her 1st meningococcal conjugate vaccine. How long should she ideally wait before receiving the booster dose? A. 8 weeks B. 6 months C. 1 year D. The patient does not require a booster dose.

D. The patient does not require a booster dose. (If given prior to the age of 16, as recommended from 11-12 yr, then a booster would be needed at the age of 16 or at least 8 weeks after the 1st dose. Since the dose was given @ 16, she does not require a 2nd dose.)

Which of the following statements is not true about the varicella vaccine? A. It is preferable to separate administration by 1 month from the administration of the MMR vaccine B. The vaccine is administered in a series of 2 shots C. The vaccine must be administered between 12 months & 12 years of age D. The series of shots must be administered a minimum of 4 months apart

D. The series of shots must be administered a minimum of 4 months apart (p. 19)

Jack, age 12, is diagnosed with ADHD. Which of the following is most strongly recommended regarding Jack's medication regimen? A. There should be a drug holiday @ nighttime during the week B. There should be a drug holiday on the weekends C. There should be a drug holiday during holiday breaks from school D. There should be no drug holiday in the course of treatment.

D. There should be no drug holiday in the course of treatment.

A 12 yo boy comes to the clinic w/a small, circular, erythematous rash w/a central clearing. His mother reports that his sister has the same rash. A physical exam indicates normal vitals. Which of the following is the most likely diagnosis? A. Tinea versicolor B. Lyme disease C. Molluscum contagiosum D. Tinea corporis

D. Tinea corporis (p. 86)

The measles, mumps, & rubella vaccine (MMR) may be given simultaneously w/tuberculosis testing & the purified protein derivative (PPD) skin test, but it is commonly recommended to postpone the PPD for 4-6 weeks for which reason? A. To avoid unwanted protein binding to the MMR vaccine B To avoid adverts effects C. To allow the MMR to form antibodies D. To avoid possible false negative on the PPD test

D. To avoid possible false negative on the PPD test (The combination of the MMR, PPD can yield an unwanted false negative response.)

According to the normal stage of primitive reflex development, which of the following reflexes is expected to not disappear by about 12 months of age? A. The head turning towards anything that strokes the cheek or mouth B. The arms spreading or contracting in response to loss of balance C. Sucking in response to objects touching the roof of the mouth D. Toes fanning out in response to stroking of the sole

D. Toes fanning out in response to stroking of the sole (The Babinski reflex, or fanning of the toes upon stroking of the sole, is expected to disappear after 12 months or when the infant starts walking, but may last for up to 2 years; if it persists past this point, it may indicate a nervous system disorder. The turning of the head towards anything that strokes the cheek is a sign of the rooting reflex, whereas sucking on anything that touches the roof of the mouth demonstrates the sucking reflex; both these reflexes are expected to disappear at 3-4 months of age. The Moro reflex, indicated by stretching or contracting the arms upon perceiving a loss of balance, is also expected to disappear during this window.)

A 7yo male presents to the clinic w/a limp from pain on his right side. You ask how long he has been limping but the boy's mother is not sure. She tells you that she noticed him limping a few days ago, but the boy did not c/o pain until very recently. He is unable to walk long distances. His temperature is 99F, which the mother explains is d/t a recent cold. Upon physical exam, you see no obvious signs, but an internal rotation of the hip causes spasm. The x-ray appears normal. Which of the following conditions do you suspect is the most likely cause? A. Septic arthritis B. Legg-Calve-Perthes dz C. Slipped capital femoral epiphysis D. Toxic synovitis

D. Toxic synovitis

Karen & Alan bring their 9-year old son, Pascal, because they are worried that he has gained an excessive amount of weight in the past 6 months. You are about to interview them and Pascal himself about his nutritional history. As you talk to them, you keep in mind the likely causes of obesity. Which of the following is not because of obesity? A. Hypothyroidism B. Antipsychotic C. Prader-Willi syndrome D. Type I diabetes

D. Type I diabetes (p. 50, 139, 144)

A 9 mo old infant presents to the clinic fussy, w/decreased appetite & fever of 102.3 F. Upon examining the patient, you find it difficult to find a localized site of infection. Which of the following is the most likely condition? A. Otitis media B. Group B streptococcal infection C. Occult bacteremia D. Urinary tract infection

D. Urinary tract infection

A 9-month-old infant presents to the clinic fussy, w/decreased appetite & a fever of 102.3. Upon examining the patient, you find it difficult to find a localized site of infection. Which of the following is the most likely condition? A. Otitis media B. Group B streptococcal infection C. Occult bacteremia D. Urinary tract infection

D. Urinary tract infection (A dx of UTI s/be considered if a local site of infection can't be found in a febrile infant. Typically, an unexplained fever is the most characteristic symptoms in an infant w/a UTI. Pneumonia & OM are also associated w/causing fever in infants, but both conditions would likely present w/other findings as well, such as tachypnea or respiratory distress in pneumonia & irritability & vertigo in OM. Gastroenteritis in infants would also cause other symptoms & wouldn't be the most likely dx.)

Maude, the 8-year old daughter of Judd, complains of an increased frequency in urination, having to go to the bathroom 3-4 times a night. This has already caused difficulty in school due to tiredness & now she is missing school due to a fever. Which of the following diagnoses do you suspect the most? A. Enuresis B. Hypospadias C. Type I DM D. Urinary tract infection

D. Urinary tract infection (p.148)

By the age of 3, a child should have received the 3rd dose of all of the following immunizations except: A. Hepatitis B B. Diphtheria, tetanus, & acellular pertussis (DTaP) C. Haemophilus influenzae D. Varicella

D. Varicella (Only 1 dose of varicella by age 3, they will have the 2nd and final dose between 4-6 years of age.)

When performing a physical exam on a 14-year-old patient, which of the following me be subject to change because of puberty? A. Teeth corruption B. Basic fine motor skills C. Hearing D. Visual acuity

D. Visual acuity (p. 58)

Incidence of viral gastroenteritis in pediatric patients usually peaks during which season? A. Spring B. Summer C. Fall D. Winter

D. Winter

A patient has a negative D-dimer test. Which of the following conditions would this lab result most strongly rule out?

Deep vein thrombosis

Richard, age 55, presents with pain and tenderness in his left ankle while walking, as well as a low-grade temperature. A physical examination finds edema around Richard's ankle. Which of the following conditions should you most suspect?

Deep vein thrombosis

After developing a quality management plan, which of the following is the next component of quality assurance?

Delineating the scope of care

Rachel, age 33, comes to the clinic with complaints of frequent distraction, lack of focus, and disorientation. While in your office, Rachel's face goes blank and she forgets where she is, although she remembers a few minutes later. Which is the following is the most likely diagnosis?

Delirium

There is concern at your clinic that some children with developmental delays are not being identified soon enough. You are examining a 6-year-old male. Of the following, which is the best tool to identify risk for developmental delay in this child?

Denver II

What purpose does Cronbach's alpha typically serve in a statistical research analysis?

Determines correlation of values in a survey instrument to assess its reliability

Melanie, an 8-month-old baby, comes to the clinic with bluish skin and a cleft palate. The pediatric nurse practitioner (NP) notes that the baby has twitching around her mouth with spasms on her hands and arms. Melanie's parents also report seizures. Upon further observation and interview, the NP learns that the baby is also developmentally delayed in many areas. Which of the following should be a part of your differential diagnosis?

DiGeorge syndrome

Cataracts are most commonly associated with which condition?

Diabetes

A 25-year-old male, who just finished a four-day camping trip with his parents and siblings, complains of having been abnormally hungry the past few months. The 6'1" man is especially concerned because his weight has dropped to 140 pounds as well. Lab results reveal two significant findings: hyperglycemia and ketonuria. What is the most likely diagnosis?

Diabetes mellitus type 1

Doris, 77 years old, reports abdominal pain in the lower left side of her abdomen. For the last several weeks, Doris experienced frequent constipation and has taken laxatives for relief. However, her abdominal pain has become increasingly severe. Her lab work shows an erythrocyte sedimentation rate of 45 millimeters per hour and leukocytosis. Additionally, a sigmoidoscopy reveals inflamed mucosa. Based on her findings and lab results, which of the following is the most likely diagnosis?

Diverticulitis

What 2 gastrointestinal disorders are more common in women that in men?

Diverticulitis and irritable bowel syndrome

Susana, age 60, exhibits slow tremors that are exacerbated when she experiences high levels of stress. She also complains of difficulty swallowing and increased muscle rigidity that has developed over the last 5 years. Upon tapping over the bridge of the patient's nose, you note repeated blinking of the eyes, which you recognize as Myerson's sign. which deficiencies does Susana most likely have?

Dopamine

A patient presents with sudden onset of wheezing, coughing, and headache, but no fever. A physical exam reveals lungs with clear auscultation that are resonant to percussion. Given these findings, you conclude the patient has bronchitis. With these particular findings in mind, which of the following drugs would be least suited for treating the patient's condition? a. Ibuprofen b. Guaifenesin/dextromethorphan c. Albuterol d. Doxycycline

Doxycycline (Viral bronchitis)

Which of the following antibiotic regimens is recommended as first-line treatment for community-acquired pneumonia in patients with no comorbidity?

Doxycycline 100 mg PO BID

Which of the following does NOT accurately describe the usual mucosal findings of a patient with a typical case of sinusitis? a. Dry b. Reddened c. Foul-smelling d. Discolored

Dry

When starting an elderly patient on a new prescription of levothyroxine (Synthroid), the NP should keep in mind that the rationale for starting an elderly patient on a lower dose is which of the following?

Due to its cardiac effects

A 52 year old male presents with complaints of heartburn and a persistent "stomahc ache" that has increased in intensity over the past 2 weeks. He points to his upper abdomen to indicate the location of the pain, and he says that he usually "feels better" after he eats. These findings most strongly suggest:

Duodenal ulcer

A patient arrives for a regular injection of Depro-Provera 14 weeks since her last injection. When you ask when her cycle begins, she says, "I don't know." After administering the injection, you should caution her to use backup contraception for how long?

During the first 2 weeks after injection

palpation of fetal movement by the mother is considered a

presumptive sign of pregnancy

An otherwise healthy, 28-year-old male presents with fever, chills, and joint pain. The patient says he went on a hiking trip in a wooded area two weeks prior. Upon examination, you notice a circular pattern resembling a bull's eye on his back. Which test would you initially order to narrow the differential diagnosis of the patient's most likely condition?

ELISA

When discussing dietary recommendations for a patient with hypertension, the nurse practitioner recommends which diet?

Eat four to five servings of vegetables per day, and limit sodium intake to 1,500 mg daily.

A patient with a history of heart failure presents to your practice complaining of classic symptoms of his disease. On examination, numerous murmurs are noted throughout the chest. In addition to a 12-lead electrocardiogram, what is the next diagnostic study to order that would be most helpful to identify the severity of the patient's problem?

Echocardiogram

A patient in the early 2nd trimester of pregnancy is experiencing seizures. She says that she experiences a severe, unrelenting headache and blurred vision prior to each episode. The blood pressure readings indicate that her BP is consistently elevated above 160/100 mmHg. Based on these findings, for what condition should she be treated?

Eclampsia

A 36-year-old female comes to you after getting a positive result on a pregnancy test. She says that she has some dull pain in her lower abdomen, adding that the day before she experienced some "spotting" in her underwear. During the physical exam, the patient winces when you palpate the left side of her pelvis. Which of the following is the most likely cause of her complaints?

Ectopic pregnancy

A patient's complaints include pelvic cramps and dark heavy vaginal bleeding. She missed her period the month prior, and has not seen any bleeding in 8 weeks until now. She also has pain in her shoulder. The exam indicates an adnexal mass. Considering that the patient is likely pregnant, these findings most indicate which of these?

Ectopic pregnancy

Which major pregnancy-related complication is known to occur in no more than 2% of the general population?

Ectopic pregnancy

A 19-year-old female presents with the following rash and describes the itchy sensation as intense. Additionally, she says that she has been experiencing occasional itchy flare up since childhood, yet her symptoms last no more than a few days. Which condition do her complaint most likely indicate?

Eczema

Your patient starts to have difficulty moving his extremities. He is coughing, wheezing, and having difficulty breathing. You look at his fingers and notice they are starting to turn blue. Which test would you order to distinguish between a myasthenia crisis from a cholinergic crisis?

Edrophonium test

Using the Dubowitz/Ballard exam to estimate the gestational age of a Caucasian newborn, you determine him to be post-term. Which of the following is a finding not related to a criterion included in the Dubowitz/Ballard exam? Thickness and size of breast tissue Elasticity of cartilage in nose Plantar creases over sole of feet Hypertonic flexion of knees

Elasticity of cartilage in nose

Patients with medial epicondylitis (golfer's elbow) typically present with:

Elbow pain that worsens with pulling activity Medial elbow pain with tenderness at the medial epicondyle Decreased grip strength.

Sarah, a 26-year-old female, experiences recurring episodes of headaches that are characterized by unilateral and lateralized throbbing. She says that changes in weather and emotional stress tend to increase the incidence. Which of the following diagnostic tests would you least likely order, based on the patient's suspected condition? a. Erythrocyte sedimentation rate b. Computed tomography scan c. Electroencephalography d. Venereal Disease Research Laboratory test

Electroencephalography

Which action should should you advise the parents to perform immediately after meals specifically to decrease reflux activity?

Elevate the newborn's head

You are taking the medical history of 15-year-old Penelope, who complains of regularly being tired. You note that she has dry skin, thinning hair, and intolerance to cold. Which of the following should be tested for in order to verify the most likely diagnosis?

Elevated TSH and decreased T4

As a nurse practitioner, you know that a diagnosis of diverticulitis usually involves a physical exam, imaging studies, and which of the following lab findings?

Elevated WBC count, elevated ESR

The nurse practitioner who suspects that one of her hypertensive patients has Cushing's syndrome would expect to find which of the following laboratory results? a. Hyponatremia b. Hypoglycemia c. Elevated serum cortisol levels d. Decreased urine 17-ketosteroids

Elevated serum cortisol levels

You are evaluating a 45 year old male who presented to your clinic with a low-grade fever, itching, and pain in the right upper area of his abdomen. He mentions during the evaluation that he has noticed his stool is "really pale, like kind of grey". Based o the patient's signs and symptoms, which of the following lab results is least likely to help in confirming a diagnosis?

Elevated white blood cell count

If a patient with a sore throat has been advised to avoid contact sports, his condition most likely stems from which of the following viruses?

Epstein-Barr virus

Which of the following skin conditions is usually NOT caused by Staphylococci? Erysipelas Impetigo Paronychia Hidradenitis suppurativa

Erysipelas (Strep)

Which acne treatment would NOT be recommended to a patient who is going to be on a 3-month backpacking trip in the desert and will not have access to electricity? Benzoyl peroxide Clindamycin Erythomycin-benzoly peroxide Clindamycin-benzolyl peroxide gel

Erythomycin-benzoly peroxide

To rule out Kawasaki disease, which test would best help to isolate the child's most likely condition?

Erythrocyte sedimentation rate

A 74-year-old patient presents with fatigue, as well as pain, stiffness in neck, and some diminished motion in her shoulders and hip. The patient is given an initial diagnosis of polymyalgia rheumatic. Which tests would best assist in initially confirming this diagnosis?

Erythrocyte sedimentation rate.

Which pathogen would be the most likely cause of prostatitis in a 64-year old male ?

Escherichia coli

Which method would yield the most sensitive but not specific lab result for the patient's suspected condition?

Esterase detection by dipstick

A patient with tubercolosis is being treated with a regimen of isoniazid, rifampin, pyrazinamid, and ethambutol. The patient's condition is improving significantly and there is evidence indicating that the isolate being treated is fully susceptible to the current regimen. At this time, which change would be recommended to incorporate into the patient's regimen?

Ethambutol may be dropped

Percy, a 38 year old male, comes to your practice with fever, cough, headache, sore throat, and excessive sweating. After ordering a series of labs and diagnostics, you find that he has an elevated white blood cell count and chest infiltrates. Given the many ways the microorganism responsible for the most likely condition might gain access to the host, which route for infection is least likely? a. Aspiration b. Inhalation c. Exogenous penetration d. Hematogenous dissemination

Exogenous penetration

In a patient with tic disorders, all of the following presentations would typically be recognized as a simple tic except:

Extensive clearing of the throat Jerking the head back and forth Blinking for 30 seconds without interruption

What finding are classic characteristic of a 3-month-old with congenital heart disease and signs of heart failure?

Failure to thrive, Irritability, Rales upon lung auscultation

Approximately 95% of ectopic pregnancies occur in which location?

Fallopian tubes

What are hyperopia and myopia also known as, respectively?

Farsightedness and nearsightedness

Vitamin deficiency resulting from malabsorption is most likely to present with:

Fatigue

You have just diagnosed Harold, age 19, with depression. His concerned parents want to know what fast-acting, safe medications are available for his condition. Which of the following is the best first-line chemotherapeutic choice for Harold in this case? a. Diazepam b. Amitriptyline c. Phenelzine d. Fluoxetine

Fluoxetine

A patient presents to the clinic with complaints of a constant headache, fatigue, and a "racing heart." A blood panel reveals elevated mean corpustular volume, normal mean corpuscular hemoglobin concntration and a red blood cell folate of 83 mg/mL. Which of the following supplements would be best for the patient?

Folate (less than 100)

A 37 year old male presents with recurring headaches, fatigue, and a decreased desire to eat. An examination and lab panel reveals that the patient has a resting heartbeat of 122 bpm, yet a normal mean corpuscular hemoglobin concentration. Which condition is the patient most likely experiencing?

Folic acid deficiency

A patient's lab results show a mean corpuscular volume of 120 fL. Which of the following anemias would be most strongly indicated by this finding? A. Iron deficiency anemia B. Sickle cell anemia C. Folic acid deficiency D. Thalassemia

Folic acid deficiency

Which of these anemias is properly paired with a dietary staple well-suited for treating the condition?

Folic acid deficiency: peanut butter

Jackie, a 25 year old female, comes to the clinic experiencing respiratory distress and difficulty speaking. Her lungs are hyperresonant and show hyperinflation on the x-ray. Which result would most strongly indicate that Jackie should be admitted to a hospital?

Forced expiratory volume is below 30

You have been treating Doug for asthma. You first prescribed a short-acting B2 adrenergic agonist. After a month or so, he reported that there had been little improvement, so you added an inhaled corticosteroid to the regimen. Now, after another month, you note that Doug's asthma does not present with secretions, but he states that his symptoms still have not markedly improved. Of the following, which should you prescribe next?

Formoterolong-acting B2 adrenergic agonist

A pregnant woman in her first trimester presents with dysuria and urinary frequency. Her urinalysis is positive for leukocytes and nitrites. The patient has no allergies. Which is the best medication to prescribe? Doxycycline Hyclate Fosfomycin Nitrofurantoin Trimethoprim-sulfamethoxazole

Fosfomycin

When caring for a patient with DM, HTN, and persistent proteinuria, the NP prioritizes the choice of antihypertension and prescribes:

Fosinopril

What law sets national standards for the security of electronic protected health information?

Health insurance portability and accountability

Dori, 64, has experienced recurring bouts of shingles. Although you note during today's appointment that her skin eruptions have disappeared, she complains of pain and burning sensations that remain in the areas of outbreak. These sensations, she adds, have been severe enough to interfere with her sleep. Which of the following do you prescribe to alleviate the patient's complaints?

Gabapentin

Casey, a 28 year old male, is admitted to the emergency room after complaining of a sharp pain in his right side and of a fever. He says that he noticed he woke up feeling nauseous the day priod, and that the pain slowly started around his "belly button" and shifted to his right side slowly throughout the day. What complication is Casey most at risk of developing if the patient's condition were to be left untreated any longer?

Gangrene

You are seeing a 62 year old female patient for a health assessment. Her medical history shows that she is a longtime smoker, and that she is currently taking NSAIDs for rheumatoid arthritis. Considering her age and history, you would know that these risk factors most likely further increase the risk of developing what condition?

Gastric ulcers

Which of the following characteristics is not typically associated with febrile seizures? a. Fever with rectal temperature of 102 °F or above b. Generally manifests in early infancy by 3 months of age c. There is usually a family history of febrile seizures d. There is typically a loss of consciousness with this type of seizure

Generally manifests in early infancy by 3 months of age. Febrile seizures do not generally manifest before 6 months of age

Which of the following patients has the least normal hemoglobin count? A. Daisy, a 29 year old mother with 15grams/100ml B. George, a 42 year old father with 13 grams/100mg C. LInda, a 50 year old grandmother with 12 grams/100ml D. Dan, a 21 year old student with 18 grams/100ml

George, a 42 year old father with 13 grams/100mg

During an eye exam, a patient's optic cup-to-disc ratio indicates that the size of the optic cup is larger than one half of the diameter of the optic disc. Which optic disease does this finding most strongly indicate?

Glaucoma

A 34-year-old real estate agent, Marco, presents with what he initially assumed was a simple canker sore. Several more sores have appeared, however. You determine that these sores are actually brown spots. The same type of spots also appears on other parts of the patient's body such as the knuckles and nail beds. After testing to confirm a diagnosis of Addison's disease, which of the following would you use to treat this patient?

Glucocorticoids and mineralocorticoids

Which of the following tests would usually be performed during a prenatal exam for a patient who is 28 weeks pregnant a. Human choronic gonadotropic test b. Utlrasound c. Glucose tolerance test d. Triple/quad screening (16-20)

Glucose tolerance test

Which of the following medications should be used with caution in a person with severe sulfa allergy?

Glyburide

A 29 year old dental assistant returns to your practice to have her annual purified protein derivative test read. The nurse in your office documents an 8 mm induation that is reddened. Which of the following is your best plan of action? a. Order an acid-fast bacilli test b. Send the patient for a chest x-ray c. Order a complete blood count d. Have her return for testing in 1 year

Have her return for testing in 1 year

How would you best auscultate to confirm a venous hum?

Have the patient sit up; auscultate the right upper sternal border.

Steven, a 48-year-old father and husband, has been brought to you by his family for alcoholism treatment. Steven, however, denies being an alcoholic. At this point, which of the following questions is the most effective one to ask this patient?

Have you ever felt the need to cut down on your drinking?

Shawn, a 23 year old male, has been experiencing nasal muscosal discharge, headache, cough, and malaise for approximately 5 days. You suspect that he has a cold. Which findings would best suggest an alternative diagnosis?

He has occasional muscle aches

Somatic symptoms of premenstrual syndrome

Headache, breast tenderness, poor coordination

Which of the following signs or symptoms would you not expect to see in a patient who is in the icteric phase of hepatitis C? a. Low-grade fever b. Dark urine c. Clay-colored stool d. Headaches

Headaches (preicteric)

An exam reveals a painful lesion on her cervix. She also has a fever, fatigue. What is the most likely dx?

Herpes

Yolanda, a 52-year-old female, states that her face seems "pulled to one side." However, no other neurological deficits are noted. Given the most likely diagnosis, what is most likely to be the underlying etiology of Yolanda's condition?

Herpes simplex

A 22 year old male comes into clinic complaining of pain in the goin area, Upon examination of his groin area, you identify abscess formations. Based on the patient's presentation, which is the best diagnosis for his skin condition?

Hidradenitis suppurativa

A 55-year-old male patient with a body mass index (BMI) of 30 has a history of angina and type 2 diabetes. His lipid profile results are total cholesterol of 280 mg/dL, low-density lipoprotein (LDL) of 195 mg/dL, and high-density lipoprotein (HDL) of 25 mg/dL. The nurse practitioner diagnoses him with hyperlipidemia and wants to start him on statin therapy. What intensity of treatment is recommended for this patient?

High-intensity statin

A mother brings her 5-month-old son for a checkup. She is concerned that her son's right leg appears shorter than his left. Furthermore, she states that his right leg looks as if it were turned outward. She adds that it does not seem as if her child is in any pain though. Suspicious, the nurse examined the child by adducting his right hip while applying pressure on the knee. For which of the following conditions is the nurse practitioner examining?

Hip dysplasia

The practitioner knows that all the following types of bone tissue are typically measured by a DEXA including

Hip, wrist, Spine, Ankle

A 4-year-Old girl complains of localized, painful swelling on her left eyelid. Her parents say that an abcess on her eyelid seems to be causing the swelling and redness. Which of the following would be the most likely cause?

Hordeloum

A patient presents with a white. pimple-like small lesion protruding on his eyelid. It is tender, firm, and discrete. Which of the following should you most likely chart?

Hordeolum

A patient presents with the following abscess and reports that it appeared suddenly, presenting with pain in the surrounding area. Of the following, which condition is the patient most likely experiencing?

Hordeolum

To qualify as a Medicare provider, a nurse practitioner is usually required to possess all of the following qualifications except; a. Master of Science in Nursing or high degree b. Hospital privileges for Advanced Practice Nurses c. Certification as a nurse practitioner by recognized national certifying body d. License in the state in which he or she intends to practice

Hospital privileges for Advanced Practice Nurses

Which of the following setting does not typically allow incident-to billing by the nurse practitioner?

Hospitals

Which of the following medications would be the best choice for anxiety with COPD patients? a. Buspirone b. Tranylcypromine c. Lorazepam d. Hydroxyzine

Hydroxyzine

What complication usually occurs as a result of type 2 DM, causing patients to produce insufficient amounts of insulin to prevent severe hyperglycemia, osmotic diuresis, and extracellular fluid depletion

Hyperosmolar hyperglycemic non ketosis

A diabetic patient reports that he has felt extreme thirst over the last 2 days, and that he has been drinking more water and urinating more frequently. You observe that he appears lethargic, and that his skin is warm to the touch and displays poor tugor. His breathing pattern appears to be normal, and he does not report nocturia. Vitals include a resting heartbeat of 112 , a blood pressure of 88/52, and a serum glucose of 649. Which condition should the patient be further evaluated?

Hyperosmolar hyperglycemic non-ketosis

A 43-year-old patient visits your office complaining of severe suboccipital pulsating headaches occurring in the morning and fading throughout the day. He is somewhat overweight and smokes. Which of the following should you most likely suspect as the primary diagnosis?

Hypertension

Which of the folding regimens would best constitute step 3 moderate persistent in the stepwise management of a patient with asthma

ICS-LABA such as budesonide-formoterol

Of the following, when is an antinuclear antibody examination of a knee injury most often indicated?

If rheumatoid arthritis is suspected.

Which of the following serology markers indicates a previous exposure to hepatitis A?

IgG

The nurse practitioner sees an adult patient who reports frequent headaches over the past few months. The patient has tried acetaminophen and ibuprofen without relief. The patient has a history of a myocardial infarction, bradycardia, and kidney stones. Which medication is most appropriate for this patient? Topiramate (Topamax) Sumatriptan (Imitrex) Imipramine (Elavil) Atenolol (Tenormin)

Imipramine (Elavil)

which of the following is attributed to decreased renal function in the elderly with regard to prescription medication

Impaired drug elimination

You are examining a new patient, Stacy, who has a history of focal seizures that do not cause loss of consciousness. Given this information, you know that if she were to have a seizure, it would likely include all of the following except: a. Tingling b. Vocalizations c. Flashing lights d. Impaired swallowing

Impaired swallowing

A young adult presents with the following lesion on his face. The rash is most characteristic of which condition?

Impetigo

During a routine physical exam, 35-year-old Sarah says that she and her partner are in a committed relationship and are looking for a long-lasting, effortless birth control method. She wants to explore her birth control options but mentions that she has an allergy to estrogen-based birth control. Under the circumstances, which of these choices should you recommend?

Implanon

In adult with newly confirmed hypertension (HTN) presents for initial treatment. BP is averaging 164/92 mm Hg. The patient's labs are normal, and the calculated atherosclerotic cardiovascular disease (ASCVD) 10-year risk is less than 10%. According to the American Heart Association and American College of Cardiology guidelines, which is the most appropriate plan?

Implement lifestyle modifications, prescribe a beta-blocker, and follow up in one month.

Rosario just gave birth to her first child born in the United States. She and her family, which includes two other children, recently immigrated from Mexico. You compliment her baby for doing well and being adorable. The next day, your supervisor gets an angry call from Rosario's husband regarding your behavior. Which of the following is the most likely complaint?

In some Mexican cultures, you must always touch the baby when complimenting him or her.

The nurse practitioner should tell her that female incidence of myasthenia gravis peaks approximately:

In the third decade of age

Which of the following would most likely result from a breakdown in the body's ability to either produce or utilize insulin in type 1 diabetic patients?

Inappropriate hyperglycemia

Patient reports her glucose in slightly elevated at 3:00am.

Increase the patient's at-bedtime dose of insulin (dawn)

Which of the following is the least likely undesirable effect to expect in a female patient using spermicides? Incomplete dissolution of suppositories Increased risk for candidiasis vaginal skin irritation unpleasant taste

Increased risk for candidiasis

Which of the following disadvantages is most commonly associated with IUDs?

Increased risk of pelvic inflammatory disease after insertion

What is the mechanism of action of probenecid (Probalan)?

Increases uric acid excretion

Which of the follow may likely result in lead poisoning?

Indian herbal remedies, inner city playground near major highway, a red wagon from 1970

Which of the following conditions would not be a contraindication to hormone therapy? a. Breast cancer b. Infertility c. Myocardial infarction d. Uterine cancer

Infertility

All of the following assessment findings are associated with Crohn's disease, except: Mass palpated in abdomen Weight loss and dehydration Fistula formation Inflammation limited to mucosal layer of colon

Inflammation limited to mucosal layer of colon

In what area of the brain do childhood tumors predominately occur?

Infratentorial region

Eleanor, 21, seeks counseling on contraceptive methods. She plans on having children within the year but would like a convenient, long-acting contraceptive method. Which of the following contraceptive methods is relatively contraindicated for this patient?

Injected contraception

Which of the following is least true regarding the physiologic changes and findings in the immune systems of the elderly?

Innate immunity functions decline while adaptive immune responses remain constant

Which of the following is least likely to present as a differential diagnosis in the assessment of dementia? a. Drug reactions and interactions b. Nutritional problems c. Emotional disorders d. Insomnia

Insomnia

What is the most common methods for diagnosing molluscum contagiosum?

Inspection and microscopic exam

Type 2 diabetes mellitus is associated with insulin resistance. Which of the following statements about insulin resistance is true?

Insulin resistance may improve with weight loss

A child's serology results come back showing Anti-HCV and HCV RNA. Which of the following may be included in his treatment plan?

Interferon and ribavirin

A patient presents to the office with complaints of asthma 1 or 2 days a week and sleeps routinely 8 to 9 hours a night with rare occurrences of nighttime asthma symptoms. Forced expired volume in one second (FEV1) is >80%, and respirations are 20 breaths/minute. What asthma classification protocol should the nurse practitioner follow?

Intermittent Step 1

A man who recently attempted suicide is brought to your office by his wife. Having conducted a significant amount of research on her own about suicide prevention, she has many questions. Which of following choices is the least effective treatment for her husband? Antidepressants if the situation is chronic Intervention if the patient's risk is escalating Antipsychotics if the patient is psychotic Hospitalization if the situation is acute

Intervention if the patient's risk is escalating

Which contraceptives has two product commonly names paragard and mirena

Intrauterine device

Which type of contraceptives may be used to prevent the recurrence of Asherman's syndrome?

Intrauterine devices

Bilious vomiting, progressive lethargy, and acute colicky pain and jelly like stools

Intussusception

The parents of 9-month-old Kent comes to your office, hurried and during the middle of the night. The parent state that their child is lethargic and constantly cries out despite all attempts to comfort him. Upon physical exam, the nurse practitioner notes palpable sausage-shaped mass in the right upper quadrant and abdominal distention. When asked about the child's stool, the parents stated that his diapers were jelly like. Of the following, which is the most likely diagnosis?

Intussusception

Your clinically depressed patient has attempted suicide twice in the last five years. He recently told you that he has been having overwhelming suicidal thoughts over the last week. Which of the following is the best way to proceed?

Involuntary commitment

Appropriate thyroid hormone biosynthesis is dependent on the dietary intake of:

Iodine

Which of the following medications is considered to be the mainstay of treatment for chronic obstructive pulmonary disease?

Ipratropium bromide (SAMA)

A patient with fatigue and anorexia. A blood panel shows that he has a normocytic and normochromic anemia, but has low total iron binding capacity and a serum ferriting level of 120 ng/mL. Given the most likely diagnosis, which of these drugs would be best suited for resolving the condition?

Isoniazid

Which of the following choices is not a standard advantage of using a diaphragm or cervical cap?

It remains in place during intercourse

You are treating a 52 year old female diagnosed with chronic glaucoma. The patient is most likely to state which of the following?

It's harder to see objects that are on the side of me

Peter comes to you after having stood for four straights days working his company's booth at a convention. He complains of intermittent numbness and tingling on his right foot up into his calf, as well as pain in the calf. During examination, you have Peter walk on the heels of his feet and you note pain and weakness of the dorsiflexion mechanism of the great toe and foot. Which of the following is your most likely suspicion?

L4-L5 disk pathology

Swollen lympnodes in the inguinal region is common in what conditions?

LVG, chancroid, syphilis

For swollen, painful knees, what is the easiest most sensitive test to assess for anterior and posterior cruciate ligament tears?

Lachman's test

Benjamin, a 55 year old male, presents with complaints of fatigue, a heartbeat that "races" even when sitting down, and difficulty breathing when lifting heavy objects. What finding would most strongly indicate a vitamin B12 deficiency rather than a folic acid deficiency?

Lack of intrinsic factor

A left-hand-dominant high-school tennis player presents with a concern of left elbow pain. Which signs and symptoms suggest that the patient is suffering from tennis elbow? (Select all that apply.)

Lateral elbow pain with tenderness at the lateral epicondyle Elbow pain with grasping movement

ST-segment elevation in leads I and aVL indicates which type of infarction?

Lateral wall

For which of the following types of condoms is use of oil-based lubricants most strongly discouraged?

Latex condoms

Owen, a 45 year old male, presents to the clinic complaining of watery diarrhea, stomach cramps, and vomiting. He states that he has been under tremendous pressure at work and is so stressed out that the often feels nauseous. Based on these findings for the most likely diagnosis, which of the following indications would you least expect to find during a physical examination?

Left lower quadrant pain

Ethel, a 90 year old female, presents to the clinic with fever, headache, excessive sweating, and soreness in the throat and chest. Results from a complete blood count reveal a low white blood cell count an a chest x-ray shows infiltrates. Based on the most likely diagnosis, which antibiotic should Ethel be treated with?

Levofloxacin

Your patient, Franklin, age 64, has the following lab values: elevated TSH, low T4, and decreased T3. Which of the following drugs should be prescribed to treat his condition?

Levothyroxine

Which medication will the nurse practitioner prescribe to a 28-year-old female newly diagnosed with hypothyroidism?

Levothyroxine T4

A nurse practitioner treats a patient who has a body mass index (BMI) of 35, type 2 diabetes, and cardiovascular disease. In addition to developing a structured lifestyle intervention program, which medication is the best choice for weight loss?

Liraglutide (Saxenda)

In an older adult with type 2 diabetes with gastroparesis, the use of which of the following medications should be avoided?

Liraglutide (Victoza)

A cardiologist has prescribed a pharmacologic treatment for your 54-year-old Caucasian patient diagnosed with hypertension. When the patient comes back to your practice for a check-up, he says that the medication causes him to cough and sometimes have trouble breathing. Of the following, which medication was he most likely prescribed?

Lisinopril

Ingrid, you 26 year old schizophrenic patient, has attempted suicide multiple times. You want to prescribe her an anti-psychotic; however, extreme caution should be exerted when specifically prescribing which of the following medications of psychosis? a. Buspirone b. Lithium c. Donezepil d. Memantine

Lithium

A newly pregnant 23-year-old woman, Jamie, comes to your office. Jamie has been experiencing food cravings, including a desire to consume ice. You suspect a specific type of anemia. Which of the following findings would you most expect as result?

Low hemoglobin/high total iron binding capacity

Which of the following lab findings is most likely to be indicative of iron deficiency anemia?

Low mean corpustular volume

An examination and series of lab panels for an elderly patient with pneumonia is most likely to show which finding?

Low white blood cell count

What is the preferred reliever medication for asthmatics according to the Global Initiative for Asthma (GINA, 2020) treatment guideline?

Low-dose ICS with formoterol

A patient diagnosed with irritable bowel syndrome (IBS) tells the nurse practitioner that over the past few months, she has been experiencing frequent bouts of constipation. Which prescription will the nurse practitioner add to the treatment plan?

Lubiprostone (Amitiza) 8 mcg PO BID

A 25-year-old male presents with a bubo in the right groin, as well as stiffness and aching near his genitals. You note that there is swelling in his right inguinal area. You aspirate the bubo and prescribe doxycycline. For what condition would this be the proper treatment? Herpes simplex virus type 2 Genital warts Molluscum contagiosum Lymphogranuloma venereum

Lymphogranuloma venereum

When using intrauterine devices, which of the following mechanisms of action is typically caused by local foreign body inflammatory responses?

Lysis of the blastocyst due to foreign body inflammatory responses.

Which class of antibiotics is first-line treatment for an unvaccinated infant diagnosed with pertussis?

Macrolides

A patient who is 32 weeks pregnant is hospitalized for severe preeclampsia. Which type of therapy to stabilize her condition?

Magnesium sulfate

In following your patient who is 32 weeks pregnant who is admitted to the hospital for convulsions. you note that her medical record shows that she has preeclampsia. Which of the following drugs is most likely to be ordered in the plan of care to stabilize the patient at this time?

Magnesium sulfate

Of the following, which method is commonly considered in addition to a computerized tomography scan in detecting ischemic infarcts?

Magnetic resonance imaging

What is the preferred diagnostic test for visualizing Grave's ophthalmopathy?

Magnetic resonance imaging

Michelle, a 6-year-old girl, arrives at your office presenting with a high fever. Immediately upon entering the waiting room, you see that the child has suddenly lost consciousness, has uncontrolled jerky movements, and urinary incontinence. How should the child's symptoms be initially managed?

Maintain airway and administer oxygen

Sharon, age 45, comes to your office with asymmetrical skin lesions that are elevated and are larger than 6 millimeters. The color and lesions varies from brown to purple, and the borders are irregular. Which of the following is the most likely diagnosis?

Malignant Melanoma

Which skin skin cancer has the highest mortality rate?

Malignant Melanoma

You are performing a fundoscopic exam on George, age 55, who presents with changes that include flame-shaped retinal hemorrhages, superficial white areas with feathered edges, and a swollen optic disc. Which of the following conditions is he most likely experiencing?

Malignant hypertension

A 48-year-old female complains of a severe headache with throbbing pain. Her children are entering college, and she admits that she has been drinking alcohol at a higher rate than usual. Furthermore, she states, "I haven't gotten a good night of sleep in weeks." Of the following types of headaches, which is she most likely experiencing?

Migraine headache

Which test will the nurse practitioner order for a patient with symptoms of chronic prostatitis?

Meares-Stamey 2-glass test

Among patients experiencing meniscus knee injuries, which type of tear is typically the most common ?

Medial meniscus. Think pop and lock with grapefruit swelling

A patient complains of "this feeling like my head's in a vise" and "this tight aching sensation in my neck," symptoms that have never presented before. Suspecting a tension headache, you would inspect the patient's neck to rule out which condition?

Meningitis

Purple to dark-red painful skin lesions all over body, acute-onset high fever, headache, level of consciousness changes, rifampin prophylaxis for close contact

Meningococcemia

"Locking" of the knee usually indicates loose bodies in the affected area. What other type of knee injury would this presentation most likely suggest? A. Ligament tear B. Knee sprain C. Soft tissue injury D. Meniscal tear

Meniscal tear

Which of the following conditions accounts for the highest number of suicides? a. Alcoholism b. Drug Abuse c. Terminal diseases d. Mental disorders

Mental disorders

You are performing a health exam on James, a 54 year old patient with a long history of alcohol addiction. You notice that his abdomen is swollen and his skin is a slight shade of yellow. When you ask him about his diet, he tells you that he does not normally have an appetite because he usually feels nauseous and his stomach "has been bothering [him] lately". Suspecting hepatitis resulting from his history of alcohol abuse, you know that all of the following options would be usually used in the patient's treatment plan except: a. Mesalamine b. Vitamin K c. Oxazepam d. Folic Acid

Mesalamine

Lactic acidosis is a potential side effect of what treatment for type 2 diabetes?

Metformin

Which agent helps decrease hepatic glucose production and intestinal absorption of glucose, while improving sensitivity by increasing peripheral glucose utilization and uptake

Metformin

Which medication would warrant the most caution for adverse interactions with niacin

Methyldopa (bp med)

A patient with anxiety complains of heart palpitations and frequent shortness of breath. Her blood pressure is 155/100. Which of the following drugs would be the best choice for her condition at this time? a. Amitriptyline b. Buspirone c. Metoprolol d. Diazepam

Metoprolol

Epidemiologic studies show that Hashimoto's disease occurs most commonly in:

Middle-age to older women

Frank, age 56, has just been diagnosed with hypertension. After looking at his medical history, you decide that propranolol would be the most effective form of treatment, as it would treat his hypertension while also managing other conditions. Given the decided treatment, Frank most likely has which of the following co-occurring conditions?

Migraines

Dave,a 72 year old patient, receives a score of 22 on a mini mental status exam. What does his score indicate?

Mild cognitive impairment

Which of the following indications of bowel obstruction is properly paired with the source of obstruction?

Minimal abdominal distention; proximal obstruction

Which of the following best defines a simple partial seizure?

Minor motor symptoms with no loss of consciousness

You are prescribing medication for Helicobacter pylori eradication to a 44 year old female with NSAID-induced peptic ulcer disease. Considering the fact that the patient must still remain on a chronic, daily NSAID regimen, which of the following drugs should you strongly consider including as part of the patient's prophylactic therapy?

Misoprostol

You hear a loud pansystolic murmur that is "blowing" at the apex of the heart. You refer the patient to a cardiology service to rule out which of these?

Mitral regurgitation

You hear a low-pitched diastolic rumble at the fifth intercostal space with the patient in the left lateral position. The murmur does not radiate. You refer the patient to cardiology to "rule out"

Mitral stenosis

A FNP is trying to determine whether a group of elderly patients are at increased risk for falls and wants to design a research study to examine the concerns of both patient and family member relative to the effect that falls in the elderly population have on physical and emotional lifestyle changes. What type of research design should be used?

Mixed methods design looking at physical variables related to falls, hospital admissions, and focus group discussions to gather information about lifestyle changes experienced by patients and families after a fall.

Which of these conditions is most closely associates with CIN2?

Moderate dysplasia

Although a chest X-ray is not necessary during an asthma attack, the nurse practitioner would expect to see which of the following on the film if one were taken? =

Moderate hyperinflation

A 7 year old with asthma symptoms once per day with 2-3 nights per week awakening. How would you classify asthma?

Moderate persistent

A 23 year old with pearly white papules around anogenital region. Aprox 2 mm wide, round, smooth and firm. No other signs or symptoms? What do they have?

Molluscum Contagiosum

Adam, age 65, presents with intense chest pain that has persisted for the last 30 minutes, In addition to what he describes as "fading in and out." His wife elaborates that on the drive here, Adam passed out a couple times. Your examination reveals a low-grade fever, dysrhythmia, and an S4 heart sound. Which of the following cardiovascular conditions would most likely present with these signs and symptoms?

Myocardial infarction

Which of the following is associated with thyroid hypofunction?

Myxedema

A nurse practitioner is slowly and cautiously rewarming a patient with blankets to avoid circulatory collapse. Which condition is the nurse practitioner most likely trying to prevent?

Myxedema coma

What method of management is recommended for plantar fasciitis?

NSAIDS Corticosteroids orthotics Night splints Physical therapy

John, age 46, comes to your office with a history of headaches. He states that the headaches last for several hours and feel like a vise is gripping his head, yet he experiences no changes in movement or sensation. Given the most likely type of headache, where would you expect the headaches to be the most intense?

Neck

A patient with advanced case of LGV reveal buboes in the vulvovaginal area. Which treatment is recommended to prevent ulceration?

Needle aspiration

Lauren, a 29 year old female, presents to the clinic with a sudden onset of productive cough that is accompanied by wheezing and a high-grade fever. During the physical exam, you find no evidence of lung consolidation, as the lungs are clear to auscultation and resonant to percussion. Given the most probable condition, which of the following pathogens would be the least likely cause? a. Streptococcus pneumoniae b. Neisseria gonorrhoeae c. Mycoplasma pneumoniae d. Haemophilus influenzae

Neisseria gonorrhoeae

According to the 2007 National Heart, Lung, and Blood Institute Guidelines for the Diagnosis and Treatment of Asthma Expert Panel Report 3, which of the following findings is not characteristic of severe asthma in an adult?

Nighttime awakenings 3-4 times per month ( mild persistent )

Sandra, age 65, presents to the clinic with chest discomfort. She describes her pain by clenching a fist to her chest. Upon examination you detect transient S4 sound. Which of the following would be in the treatment plan for Sandra?

Nitroglycerin, Atenolol, Diltiazem

The term "pervasive development disorders" refers to a group of conditions that involve delays in the development of many basic skills. Children with these conditions often are confused in their thinking and generally have problems understanding the world around them. Which of the following is a factor leading to this condition?

No known factor

Which of the following is the most approprate choice of management for moderate thalassemia? A. Red blood cell transfusion or splenectomy B. Chelation therapy C. Oral ferrous sulfate D. No treatment

No treatment

If a patient presents with xanthoma, how would best describe the skin configuration?

Nodule

Which of the following best delineates the degree to which a nurse practitioner may prescribe medication to patients?

Nurse practice acts

All of the following patients have an increased risk of developing adverse effects from Metformin (Glucophage) except: a. Patients with renal disease b. Patients with hypoxia c. Obese patients d. Patients who are alcoholics

Obese patients

A professional claim incident occurred during an active policy period. The claim was filed after the policy period expired. Which type of professional liability insurance provides coverage in this circumstance?

Occurrence

A patient arrives at your clinic to discuss long-term options for contraception. In evaluating her circumstances, you decide that Implanon might work best for her. You might tell her all of the following regarding the implant except:

Odds are you will be able to maintain a regular cycle"

Which agent is used specifically for the treatment of peptic ulcer disease and may also use as an alternative drug for H2 receptor blockers to manage gastroesophagal reflux disease?

Omeprazole

Following a regimen for Helicobacter pylori eradication therapy, a patient is continued on for antiulcer therapy to ensure symptomatic relief and promote ulcer healing. Which of the following regimens would be best suited for treating duodenal ulcers as a part of this therapy?

Omeprazole 40 mg daily or lansoprazole 30 mg daily for 7 weeks

The results of a prenatal exam for a patient who is 16 weeks pregnant show normal findings. The nurse practitioner should schedule the patient's next prenatal visit in

One month

During an annual physical exam, you notice that 12-year-old Lisa has breast enlargement without seperate nipple contour. You tell her she can expect which of the following in the next stage of her development:

Onset of menses

A 26-year-old male was recently diagnosed with plantar fasciitis. He says he experiences sharp pain and stiffness in the affected foot each morning; however this pain usually fades by the time he's ready for work. Which treatments would have the lowest priority at this time? A. Orthotics B. Physical therapy C. Night splints D. Opioids

Opioids

Joshua, a 45 year old male, comes to your practice with complaints of fatigue, weakness, and weight loss. After further investigation, you discover that he has generalized lymphadenopathy. You order a complete blood count, which reveals subnormal red blood cells and neutrophils. Given the most likely diagnosis of leukemia, you might recommend any of the following methods of management except: A. Symptomatic control B. Oral ferrous sulfate C. Bone marrow transplantation D. Chemotherapy

Oral ferrous sulfate

A patient comes to your practice with a severely swollen knee. He says that he was training for a marathon and attempted to run at a faster pace than he usually runs. Almost as soon as he sat down to rest, he noticed swelling, inflammation, and pain upon knee movement. Which would not be a standard form of treatment for the patient's symptoms at this time? A. Splinting B. Asprin C. Oral steroids D. Heat application

Oral steroids

A 62-year-old postal worker enters the emergency room and tells you that he thinks he has been exposed to anthrax during the course of his workday. He believes that the exposure took place the previous day at work, though he felt nothing at the time. He went home and awoke this morning feeling feverish with muscle pain and breathlessness. Which of the following is the most appropriate response?

Order a Gram stain and alert local law enforcement, if needed

Your colleague's patient experiences a sudden onset of pain in her lower leg. The area where the patient identifies the pain is erythematous and warm to the touch but not swollen. The patient also has a consistent temperature. Your colleague suggests elevating the leg and applying warm compresses at night. He also prescribes a nonsteroidal anti-inflammatory drug and suggests bed rest for at least a week to help the pain subside. Based on these findings, which of these measures is not a correct form of treatment for the patient's condition? a. Elevating the leg b. Applying warm compresses c. Prescribing non-steroidal anti-inflammatory drugs d. Ordering bed rest

Ordering bed rest

The following methods of management are recommended for Morton's neuroma?

Orthotics and Corticosteroids Referral for cryogenic neuroablation or neurectomy

Positive radiographic findings are most instrumental in diagnosing which of these conditions? A. Plantar fasciitis B. Bursitis C. Carpal tunnel syndrome D. Osteoarthritis

Osteoarthritis

A healthy, fit, 41 year old male says the the has been experiencing several problems that manifested about 2 days ago after his head was slammed onto the mat while wrestling at a mixed martial arts school. His initial complaints were dizziness and ringing in the ears. Since then, he has been experiencing short periods of dizziness and disorientation, ringing in the ears, and blurred vision. An ear exam shows no signs of inflammation or exudate. The patient may require a further diagnostic assessment for all of the following conditions except: a. Retinal detachment b. Vertigo c. Otitis externa d. Hearing loss

Otitis externa

Kenny is 24-years-old and works for a package delivery service. During the course of his workday, he repetitively lifts heavy items. He reports a history of muscle strain and comes to your office with a particularly severe strain that has kept him out of work for the past two days. You note slight edema and ecchymosis in his right upper back as well. However, an x-ray reveals no rib fractures or other bone injuries. Of the following, all are advisable in managing this patient's injury at this time except: Prevention education Non-steroidal anti-inflammatory drugs Oxycodone Immobilization and ice

Oxycodone

STDs matched with pathogens

PID- Neisseria Gonorrhoeae & Chlamydia Lymophogranuloma venereum- chlamydia trachomatis Chancroid- Haemophilus ducreyi Condyloma acuminata (genital warts)-human papillomavirus (HPV). Syphilis- Treponema Pallidum

Which of the following indications would the nurse practitioner most likely expect to see during a physical exam that would suggest a diagnosis of cholecystitis?

Pain upon inspiration while fingers are placed under the right rib cage

Three signs of positive pregnancy

Palpation of fetus by health care provider ultrasound and visualization of the fetus Fetal heart tones auscultated by health provider

Although tricyclics and monoamine oxidase inhibitors are less efficacious for generalized anxiety, these medications are most likely to be useful in treating which issues associated with anxiety? a. Tachycardia b. Palpitations c. Panic attacks d. Breathlessness

Panic attacks

Which of the following anxiety disorders is most commonly tied to the development of agoraphobia?

Panic disorder

During a routine pap smear, 37-year-old Angelica requests a birth control prescription and adamantly states that she does not want to conceive at her age. However, during your exam, you note that her blood pressure is at a prehypertensive level. Based on her medical status, which of the following is the safest and most effective birth control option for her?

ParaGard

Nancy, a 24 year old female, was engaging in intercourse with her boyfriend 3 days ago when the condoms broke. She seeks emergency contraception; however, in the past, she has experienced severe nausea and vomiting after using levonorgestrel products. Which other product would be best suited for Nancy at this time?

ParaGard (copper IUD can be alternative form of emergency contraception within 5-6 days of intercourse)

During a routine pap smear, 37-year-dd Angelica requests a birth control prescription and adamantly states that she does not want to conceive at her age. However, during your exam, you note that her blood pressure is at a prehypertensive level. Based on her medical status, which of the following is the safest and most effective birth control option for her?

Paragaurd

A patient experiences tremors, impaired swallowing, and drooling, in addition to testing positive for Myerson's sign. Based on the above symptoms and lab results, you conclude that the patient is exhibiting the initial onset of

Parkinson's disease.

A nurse practitioner listens to the heart of an infant patient during a well-child care visit and hears a continuous and machine-like heart murmur. This exam finding could represent which congenital heart defect?

Patent ductus arteriosus

A 32-year-old male marathon runner, Freddy, reports to the urgent care center with severe pain in his right lower quadrant (RLQ) that began shortly after a 15 mile training run. While examining the patient, you find that Freddy clutches his RLQ upon contact and, when asked to extend his right leg, explicitly describes pain. Given this information, which of the following findings would best assist you in differentiating the most likely diagnoses of appendicitis, colon cancer, gastroenteritis, or irritable bowel syndrome?

Patient's bowel habits are normal

Susana, age 45, presents to urgent care with severe chest pain, accompanied by cold sweats and nausea. An examination reveals pulmonary crackles. You administer nitroglycerin; however, it proves ineffective. Which of the following lab results would be consistent with the most likely diagnosis?

Peaked T waves, Elevated ST segment, Leukocytosis, development of Q waves (not all)

You are consulting Amber, a 41 year old female who has gastroesophageal reflux disease. In discussing her diet, you should advise her to avoid consumption of all off the following food products except: a. Caffeine b. Spices c. Peanut butter d. Alcohol

Peanut butter

As a practitioner, you know that the approximate theoretical and actual failure rates for oral contraceptives are:

Perfect use: 0.1%, typical use 3%-5%

You know that the following factors may usually contribute to sensorineural hearing loss except: a. Close range to an explosion b. Acoustic neuroma c. Perforated tympanic membrane d. Long-term exposure to loud music

Perforated tympanic membrane

A 68-year-old male presents to your office complaining of sporadic numbness in his big toe that occasionally affects his entire foot. His history is positive for smoking, hypercholesterolemia, as well as type 2 diabetes. On the physical exam, you note that the skin on his feet and lower legs is shiny and significantly less hairy; however, neither his feet nor legs are remarkably swollen or red. Which of the following is the most likely diagnosis?

Peripheral arterial disease

A 47 year old male patient has signs of leukemia. Which of the following diagnostic tests would best distinguish acute leukemia from chronic leukemia?

Peripheral blood smear

Which of the following is NOT a common component of the pathology for type 1 DM? a. Human leukocyte antigens b. Islet cell antibodies c. Peripheral insulin resistance d. Ketone development

Peripheral insulin resistance

Dependent rubor is a physical finding associated with which of these diseases or conditions?

Peripheral vascular disease

The Schilling test is most associated with screening for which of the following conditions?

Pernicious anemia

Sarah, a 35 year old female, presents to your practice with anorexia, glossitis, and dizzziness. She tests positive for both the Romberg and Babinski tests. Which of these is the most likely condition?

Pernicious anemia (vitamin B12 anemia)

Intermittent cough that becomes more severe with inspiratory whoop; may be followed by posttussive vomiting. Cough worse at night; persists for 2-6 weeks or longer. Infants do not "whoop"; minimal cough followed by vomiting; apnea more common.

Pertussis

Which of these tests would provide the best assessment for carpal tunnel syndrome?

Phalen's test

Which of the following is least likely to be used to treat patients wit depression due to its side effects and high overdose potential? Alprazolam Memantine Citaloprm Phenelzine

Phenelzine (Monoamine oxidase)

Which of the following results is considered the hallmark ofChronicMyelogenousLeukemia(CML)

Philadelphia chromosome in leukemic cells

You are ordering tests to confirm a diagnosis of bowel obstruction for a 36 year old male who presents with complaints of committing, inability to pass stool, bloating, and severe abdominal pain around his "belly button." Which of the following indications would provide the earliest suggestion that the patient's condition may be due to colon cancer?

Positive guaiac test

A "heavy-set" dock worker in this late 30s presents with pain in his back. The patient says that the pain has recently increased and has begun spreading to his legs. His posture is slightly twisted to the right, and he appears to be stooped forward, indicating that he may have radiating or sciatic pain. Which of these test results would best confirm the initial diagnosis:

Positive straight leg raise test.

Which of the following areas is not a common site of bursitis? A. Postpatellar (prepatellar) B. Olecranon (knee) C. Subdeltoid (shoulder) D. Ischial

Postpatellar One of the most common sites for bursitis is the prepatellar, not post patellar, area. Other common sites for bursitis include the olecranon, subdeltoid, and ischial areas.

Mr. Thomas is a 74-year-old who was diagnosed with Parkinson's disease at the age of 57 said "I'm really having trouble getting up from my recliner, and sometimes I can't even pull the chair up to the dinner table." Considering common drug regimens for Parkinson's patients, you realize that Mr. Thomas should be referred to his neurologist for an increase in which drug?

Pramipexole (Mirapex) (dopamine agonist)

According to the latest guidelines regarding statin therapy, which of these should you prescribe to specifically reduce low-density lipid cholesterol by approximately 30% to less than 50%?

Pravastatin, 40 mg

A patient presents with an inability to move the left side of his face. The nurse practitioner identifies the cause as an inflammatory reaction involving the facial nerve. Which medications would be best suited for treating Bell's palsy

Prednisone

which of these management strategies is commonly ordered to treat severe cases of allergic contact dermatitis?

Prednisone Taper

Which treatment is best employed to treat severely enlarged tonsils in cases of mononucleuosis?

Prednisone taper

Which three medications are appropriate for the treatment of chronic pain associated with fibromyalgia?

Pregabalin (Lyrica) Duloxetine (Cymbalta) Milnacipran (Savella)

Which of these most strongly reflects the theory behind natural family planning?

Pregnancy is less likely when the female is lactating

Both cocaine use and poor nutrition are known risk factors for which pregnancy-related complication?

Premature labor

Considering that age is a risk factor for some pregnancy related complications, a 16 year old and a 45 year old who are pregnant are both at risk of developing which complication due to their ages?

Premature labor

You recommend to a 20-year old patient that she significantly reduce her caffein and salt intake. You also recommend that she eat more foods containing vitamins E and B6 to help reduce or manage symptoms associated with which condition?

Premenstrual syndrom

A male patient in his early 20s who was diagnosed with a stye in his left eye has been applying a warm compress to the affected area for more than a week as part of a home remedy. Despite this treatment, the patient's stye is still present. What further treatment would be best to recommend at this time?

Prescribe erythromycin topical ointment

A nurse practitioner evaluates an older adult patient during a chronic disease management visit. The patient has a history of type 2 diabetes, essential hypertension, and coronary artery disease, sustaining a myocardial infarction one year ago. The patient is taking rosuvastatin (Crestor) 40 mg daily for cholesterol management, with a recent low-density lipoprotein (LDL) of 90 mg/dL. Which should be considered for this patient's current LDL level?

Prescribe ezetimibe (Zetia) 10 mg daily.

Identify the finding that best indictes pernicious anemia

Presence of antiparietal cell antibodies

A patient comes to your practice with symptoms of fever and pain in the throat. Suspecting strep pharyngitis, you assess the patient under the guidelines of the Center criteria. Under these standards, which of these findings would not be confirmatory of strep pharyngitis?

Presence of persistent cough

A patient diagnosed with abnormal uterine bleeding is undergoing further evaluation to determine the etiology. Which condition is least likely to be the cause of the patient's abnormal uterine bleeding? a. Polycystic ovarian disease b. Primary dysmenorrhea c. Perimenopause d. Immature hypothalamic-pituary-ovarian axis

Primary dysmenorrhea

You order a referral to further evaluate your 19-year-old patient who reported symptoms of cramping, pain in her upper thighs, nausea, and fatigue during the first few days of her menstrual cycle. For which do you make a referral?

Primary dysmenorrhea

Nick, a 39-year-old patient diagnosed with hyperthyroidism, complains of frequent hand tremors. Lab results indicate that the patient has subacute thyroiditis. Which of the following medications would be the preferred symptomatic treatment in this case?

Propranolol

Sandra, a 44-year-old, experiences a unilateral, lateralized throbbing headache with accompanying visual disturbances at least three times a month. She says that these headaches are usually preceded by auras. Of the following medications, which would be the best option to order for prophylactic therapy for the most likely type of headache?

Propranolol

Which medication would you be least likely to prescribe to a patient diagnosed with hyperthyroidism and asthma, unless absolutely necessary?

Propranolol

A 19 year old patient tells you that she normally experiences pain during the first few days of the menstrual cycle. She describes the pain as a cramp that radiates to her back and upper thighs, usually accompanied by nausea and fatigue. Which high levels of which hormone are most likely responsible for her symptoms?

Prostaglandin

What is the aim of Title I of the health Insurance Portability and Accountability Act?

Protection of health insurance coverage for workers when they change or lose their jobs

What is indicative of an initial herpes outbreak?

Pruritic ulcers lasting 12 days.

Your patient lives in Palm Desert and presents to your practice with this finding. He describes it as painless, though sometimes it "feels like there's something sticking to my eye." What is the diagnosis?

Pterygium

A 25-year-old female patient presents with increased sweating, fatigue, tremors, and weight loss. Upon examination, you note she has moist velvety skin and fine hair. Based on the most likely condition, which of the following findings would you LEAST likely expect? a. Puffy eyes b. Heat intolerance c. Anxiety d. Increased appetite

Puffy eyes

The nurse practitioner orders a pulmonary function test for a patient exhibiting signs of an obstructive lung disease. All of the following are differential diagnoses, except: Asthma Bronchiectasis Pulmonary fibrosis Emphysema

Pulmonary fibrosis

which of these conditions are restrictive disorders?

Pulmonary fibrosis Tuberculosis Heart failure Kyphoscoliosis

A patient presents to you with a productive cough and headache. The patient displays no other signs or symptoms and is afebrile. Upon physical examination, you note that his lungs are clear upon auscultation and resonant upon percussion. You suspect acute bronchitis; as the indications are far from definitive, however, you wish to rule out other conditions. Which diagnostics would be least helpful in confirming or excluding probable conditions at this time? a. Sputum culture b. Chest x-ray c. Sputum sensitivity d. Pulmonary function test

Pulmonary function test

All of the following would be consistent with a typical manifestation of severe asthma except: a. Respiratory rate of 35 bpm b. Pulse of 125 bpm c. Pulsus paradoxus of 15 mmHg d. White blood cell count of 1800 eosinophils/mcl

Pulsus paradoxus of 15 mmHg (moderate is 10-20)

You are treating a patient who is unable to move the right side of her face. As a nurse practitioner, you know that while this condition may resolve without treatment, some patients may benefit from treatment. Which of the following treatment options is least likely to be beneficial a.Prednisone b. Artificial tears c. Pyridostigmine(Used for MG) d. Acyclovir

Pyridostigmine(Used for MG)

An anteroseptal wall infarction would present with

Q waves in leads V1-V4.

You are writing a manuscript about an unsolved medical case in which the patient was being treated for heartburn and unexpected died. Which of the following would best describe this type of research?

Qualitative

Which of the following is not typically preformed either during the first trimester of pregnancy or upon a new visit? Quantitative human chorionic gonadotropin blood titer STD screeding HBV surface antigen screening dating ultrasound

Quantitative human chorionic gonadotropin blood titer

Which of the following types of heart murmurs is not typically considered an innocent heart murmur?

Radiating murmur

Which of these is not a typical finding of malignant melanoma? Asymmetry Border irregularity Radius larger than 6mm Elevation

Radius larger than 6mm. (Should be diameter>6)

Which treatment is recommended for an infant with noncommunicating hydrocele?

Reassess in 1 year

Iron deficiency anemia results in a decrease in the amount of iron available for which process?

Red blood cell formation

All of these treatments may be necessary for a patient with severe thalassemia?

Red blood cell transfusion chelation therapy splenectomy

For patients with inadequate iron intake, most of what composes a patient's blood will be lower than normal in laboratory findings. What aspect of blood, however, may actually be found to increase?

Red cell distribution width

What would be your most likely reason for prescribing Lugol's solution to a patient?

Reduce vascularity of the gland

A mother brings her 1-month-old infant to the clinic for evaluation, stating the infant has poor weight gain despite constant hunger and projectile vomits after eating. The nurse practitioner notes that the infant has a distended abdomen. Upon palpation, the nurse practitioner detects an olive-shaped mass in the epigastrium. The nurse practitioner will:

Refer the child for ultrasound testing because the infant presents with signs and symptoms of pyloric stenosis: abdominal distention, dehydration, projectile vomiting, and failure to thrive.

A patient presents with hearing loss and painless otorrhea. Upon physical exam, you note the canal is filled with mucopus and granulation tissue. Tympanic perforation and ossicular damage are also noted. What is the best way to treat this patient's condition?

Refer the patient for surgery

A family nurse practitioner assesses an asymptomatic adolescent patient in the clinic with a maternal history of celiac disease. The nurse practitioner orders initial labs to assess for celiac disease; tissue transglutaminase (tTG) IgA is elevated at 50, and serum IgA is normal. What is the next appropriate step?

Refer the patient to a gastroenterologist.

An adult comes to your practice with complaints of occasional weakness, numbness, and double vision but no reported breathing difficulties. You order lab tests and the results indicate lymphocytosis, an elevated protein level in the cerebrospinal fluid, and an elevated igG index level in the cerebrospinal fluid. Which would be most appropriate for managing the patient's most likely condition at this time?

Refer the patient to neurology for suspected multiple sclerosis

Which of the following lab results is typically present in diabetic ketoacidosis but NOT in hyperosmolar hyperglycemic non-ketosis? a. Hyperosmolarity b. Decreased sodium c. Relatively low pH d. Elevated BUN and Cr

Relatively low pH

Depo-Provera (progestin only) and NuvaRing share all of the following mechanisms of action except: a. Release of synthetic estrogen and progestin b. Thickening of cervical mucus c. Suppression of follicle-stimulating hormone d. Suppression of luteinizing hormone

Release of synthetic estrogen and progestin is only NuvaRing

Sarah brings her 2-year-old son in for a routine checkup. During the exam, she expresses concern about her son's behavior during the last several months. She says that her son frequently touches his genitals and obsessively pokes at the genitals of his dolls. Additionally, since the last month, he has been crying and resisting his mother's attempts to wipe his bottom after defecation. Sarah works full-time and leaves her son at a home-run daycare during the week. She is concerned that someone at the daycare might be abusing her son. Which of the following indicates the most appropriate initial course of action the nurse practitioner must take?

Report suspected abuse immediately.

The following are common known causes of primary amenorrhea except for: a. Turner syndrome b. Vaginal agenesis c.Resistant follicular stimulation d. Endocrine imbalance

Resistant follicular stimulation

In cases of asthma, the trachea and bronchi typically become more:

Responsive

During an eye exam, a 65 year old man complains of blurred vision and flashes of light in his periphery, which began to manifest within the past day. The patient was diagnosed with type 2 diabetes mellitus at age 55. Given these findings, which of these is the most likely eye condition, and how would the nurse practitioner best manage this condition?

Retinal detachment and should be referred to a surgeon

Tanya is 10 years old, 4'8" tall, and weighs 110 pounds. Tanya's mother is concerned about her weight and eating habits, as she has found junk food wrappers hidden around the house. She adds that Tanya is very defiant when it comes to food restrictions. Which of the following is the most appropriate way of approaching the topic of diet with Tanya?

Review the new MyPlate standards with Tanya and help her understand why a balanced diet and maintaining a healthy weight are important.

Which of the following is least useful as an initial test for providing a diagnosis of suspected placenta previa? a. Complete blood count b. Ultrasound c. Rh blood test d. Electronic fetal monitoring

Rh blood test

All of the following situations constitute a high risk situation in pregnancy except: a. Multiple gestations b. Thrombocytopenia c. Premature rupture of membranes d. Rh desensitization

Rh desensitization (Rh sensitization)

Of the following, which musculoskeletal disorder would most likely present with symmetrical inflammation?

Rheumatoid arthritis

Which findings typically distinguishes a viral form of pharyngitis from a bacterial form of the condition? a. Fever b. Erythematous pharynx c. Painful throat d. Rhinorrhea

Rhinorrhea

Sharon, a 24 year old female says that her Nuvaring, which had been in place for 2 weeks, fell out about 4 hours prior. What instructions would be most effective in helping Sharon continue on her contraceptive schedule?

Rinse the ring with cool water, reinsert, and use a spermicide or barrier for 1 week

A patient arrives at your clinic after camping in the woods of Connecticut over a week ago. He says he had a fever and nausea last week but didn't think it was serious enough to go to the doctor. This morning, he notice purple spots on his legs. Upon an examination you identify petechial rash around his ankles. You immediately prescribe antibiotics. Which is the most likely diagnosis?

Rocky Mountain spotted fever.

Which type of heart sound is caused by aortic and pulmonic valve closures?

S2

Which of these heart sounds would most strongly indicate a cardiac disorder in a pediatric patient?

S4

What pathogens responsible for pneumonia typically demonstrate lobar consolidation as a radiographic characteristic

STREPTOCCOUS PNEUMONIAE Hamemophilus influezae Klebsiella pneumonia

In cases of abruptio placentae, separation of the placenta from the uterus wall usually occurs at which point in pregnancy?

Second or third trimester

Which of the following processes is most commonly associated with type 2 diabetes?

Secretory defect causing a resistance to insulin

Katie, a 17 years old female, has a history of PMS. She complaining of PMS symptoms have gotten worse over the past few months. You consider pharmacological management for her. Which of the following would be effective in managing her concerns at this time?

Selective serotonin reuptake inhibitors, Hormonal birth control, Vitamin E, low salt intake

what is the typical window of time in which an infected person most commonly converts from HIV neg to positive?

Seroconversion, takes between 3 weeks and 6 months after initial infection

An older adult presents with back pain, weight loss, and fatigue. Physical exam reveals pallor and height loss of 2 inches. Labs show hemoglobin 10 g/dL, creatinine 2.5 mg/dL [0.6-1.3 mg/dL], serum protein 9.2 g/dL [ 6.4-8.3 g/dL] and a calcium 11.8 mg/dL [9-10.5 mg/dL]. Which is the best test for the nurse practitioner to order?

Serum protein electrophoresis

You are conducting a physical exam on a patient with severe chest pain that occurred while he was at home watching television. He also complains of sudden sweating in a room temperature environment, weakness, difficulty breathing, and nausea. He comments that he had an uneasy feeling of impending doom during the event. When he experiences somewhat similar angina pain, he takes nitroglycerin, but this time, the nitroglycerin did not help with the pain. Based on the likely diagnosis, which of the following would you least expect to find? a. S4 heart sound b. Wheezing c. Shiny, hairless skin d. Pulmonary crackles

Shiny, hairless skin

A 16-year-old male, with no abnormalities in his medical records, presents with unusual body proportions and undeveloped sexual characteristics. A blood test indicates Klinefelter syndrome. Which of the following is not a typical feature of Klinefelter syndrome? Learning disability Hypogonadism Gynecomastia Short stature

Short stature (Tall)

The SCOFF screening questionnaire is a series of five questions that can help a provider determine whether a patient may have a diagnosis of an eating disorder:

Sick: Do you make yourself sick because you feel uncomfortably full? Control: Do you worry you have lost control over how much you eat? One stone: Have you recently lost more than one stone (14 lbs or 6.35 kg) in a three-month period? Fat: Do you believe yourself to be fat when others say you are too thin? Food: Would you say that food dominates your life?

Carrie, 74, comes to you for her checkup. During the interview, she mentions that her eye sight has deteriorated drastically. She states, "Although I am old and have less than 20/20 vision, my eye sight has become cloudy, and I sometimes see halos around lights." Carrie denies pain when asked. Accounting for Carrie's age and stated symptoms, which of the following would best serve to confirm the most likely diagnosis? Halos around lights Constriction of visual fields Single-eye diplopia Dilated pupils

Single-eye diplopia

A 29-year-old male visits your clinic complaining of external burning and itching on his penis. He reports that his condom recently broke during sex, and he has since switched to a different brand. There are rumors at his school that the partner he was with when the condom broke has human papilloma virus. He is worried that he may have been infected with the virus as well. Which of the following options is the best initial course of action?

Skin test to check for latex and/or spermicidal allergy

A 29-year-old male visits your clinic complaining of external burning and itching on his penis. He reports that his condom recently broke during sex, and he has since switched to a different brand. Which of the following options 1s the best initial course of action?

Skin test to check for latex and/or spermicidal allergy

Danny, a 13-year-old African-American male, is 5'6" and has a body mass index of 34. His temperature and blood pressure are within the normal range. He has grown 2.5 inches in the last 12 months. He normally spends several hours each afternoon and evening in his room playing video games, but, in an effort to lose weight, he walks for an hour every evening. Today, he began to feel pain in his left thigh and knee, and he now has groin pain as well as pain in the right thigh and knee. He denies any recent bodily trauma. In your physical examination, you note that he is unable to properly flex either hip when the femur abducts. Which of the following is the most likely diagnosis?

Slipped capital femoral epiphysis

Hannah, a 23-year-old female, comes to the clinic after having multiple episodes of headaches. She complains that the headaches throb and usually manifest on one side of her head, build "to a slow crescendo," and last for several hours. She also experiences visual disturbances and nausea. Which of the following foods would most likely be contributing toher complaints?

Smoked salmon

Of the following, which type of cancer usually develops from a actinic keratosis?

Squamous cell carcinoma

A patient who has recently started on the patch mentions that she is also pursuing an herbal regimen for various health issues. Which of the following herbs would be most likely to affect her treatment with the patch?

St. John's Wort

According to the American College of Cardiology and the American Heart Association guidelines, which average confirms stage I HTN?

Stage I HTN is classified as systolic BP 130-139 mm Hg or diastolic BP 80-89 mm Hg.

A 35-year-old male presents with complaints of a "strange swelling" in his throat. He also mentions that he has lost about 12 pounds in the past several months. A physical examination reveals that he has swollen lymph nodes, and there is involvement in the right axilla and right side of the neck. Based on the suspicion of lymphoma, which stage of disease does this presentation most likely represent?

Stage II

A 56-year-old marketing executive has various past blood pressure (BP) readings of between 140 to 150 mmHg (systolic) and 80 to 100 mmHg (diastolic) over the last few months. During an office visit, you find his current BP is 142/88 mmHg. This collective data confirms a diagnosis of hypertension as per the ACC/AHA guidelines. Based on these guidelines and the patient's current blood pressure, how would you classify his hypertension today?

Stage II

After your patient undergoes diagnostic studies for lymphocytic malignancy, you determine that there are two lymph node groups involved on both sides of the patient's diaphragm. In addition to bone marrow involvement. Which stage of lymphocytic malignancy is best illustrated in the patient?

Stage IV (bone marrow or liver involvement)

Which of the following pathogens is least likely to cause acute sinusitis in the adult-gerontology patient?

Staphylococcus aureus

A middle-aged female who is complaining of gradual weight gain, lack of energy, dry hair, and an irregular period over an 8-month period. The routine annual laboratory testing showed a TSH result of 10 mU/L. The NP decides to order a thyroid panel. The TSH is 8.5 mU/L and the serum T4 is decreased. The patients BMI is 28. The heart and lungs exams are normal. Which of the following is the best treatment plan?

Start the patient on levothyroxine (Synthroid 0.25 mcg daily)

A patient presents to the office with complaints of asthma 1 or 2 days a week and sleeps routinely 8 to 9 hours a night with rare occurrences of nighttime asthma symptoms. Forced expired volume in one second (FEV1) is >80%, and respirations are 20 breaths/minute. What asthma classification protocol should the nurse practitioner follow

Step 1

A patient presents to the office with complaints of asthma symptoms daily with occurrences of nighttime asthma symptoms twice weekly. Forced expired volume in one second (FEV1) is >79%, and respirations are 20 breaths/minute. What asthma classification protocol should the nurse practitioner follow?

Step 3 moderate persistent

A female patent presents to the clinic with the following lesion on her eye. Which treatment would you be least likely to recommend? Chemical cauterization Carbon dioxide laser ablation Cryotherapy Steroid injection

Steroid injection

A 56 year old patient presents with complaints of nausea, vomiting, and watery diarrhea, all of which have lasted for 4 days. Upon examination, you note hyperactive bowel sounds, abdominal distension, and tachycardia. Based on these findings, which of the following lab tests would best help to diagnose the patient's condition?

Stool test

Nonviral otitis media is most commonly caused by what pathogenic bacteria?

Streptococcus pneumoniae

A patient using the calendar method for contraception has determined the shortest cycle lasts 28 days, her longest cycle lasts 32 days, and that her cycle starts on day 5 of the month. Which range of days of the month should she abstain from intercourse to best avoid pregnancy? Day 15 to day 28

Subtract 18 days from shortest cycle and 11 days from longest cycle. Add both totals to day of month her cycle begins to determine window

Which of the following findings would you expect to see in a patient with cholelithiasis caused by biliary obstruction?

Sudden intensifying pain in the upper right abdomen, fever, dark urine, pale colored stools

Your 57-year-old male patient has awakened nightly for the last three weeks to severe throbbing pain around his left eye, particularly after drinking alcohol. The pain usually subsides within an hour. A physical exam reveals nothing too unusual, other than eye redness and rhinorrhea. Of the following, which would be the best way to manage the patient's condition? Clopidogrel Sumatriptan Over-the-counter analgesics Ticlopidine

Sumatriptan

Which test would provide the most definitive diagnosis of neuroblastoma?

Surgical biopsy

You are testing a patient who you suspect has Parkinson's disease. You repeatedly tap the bridge of her nose. Which of the following responses or results would indicate a neurological disorder?

Sustained blinking (Myerson's sign)

A Caucasian older adult patient presents for follow-up after hospitalization for New York Heart Association (NYHA) class III heart failure (HF). Physical exam reveals blood pressure 114/78 mm Hg and heart rate 55 beats/min. Current therapy includes enalapril (Vasotec) 20 mg twice daily, metoprolol succinate (Toprol-XL)100 mg daily, spironolactone 50 mg daily, and furosemide (Lasix) 40 mg daily. Which adjustment to the patient's therapy is the best option to reduce morbidity and mortality?

Switch the angiotensin-converting enzyme (ACE) inhibitor to a combination angiotensin receptor blocker (ARB) and neprilysin inhibitor for example Sacubitril/valsartan (Entresto)

Which of these methods of natural family planning uses both the basal body temperature graph and cervical mucus test?

Sympto-thermal method

What is the 3rd most commonly reported STD in the US?

Syphilis Chlamydia is most common, 2nd=gonnorhea

After reviewing Betsy's electrocardiography and echocardiogram results, it is clear that her heart is unable to contract sufficiently, resulting in decreased cardiovascular output. Which heart failure does this best describe?

Systolic

Benjamin, age 62, presents with wheezing, a frothy cough, and shortness of breath at rest. The cardiologist's report reveals that his condition derives from the inability of the heart to contract, which results in decreased cardiac output. Which of these is Benjamin most likely experiencing?

Systolic acute

You are working on a study comparing the differences between patients with glaucoma and those with cataracts. Which of the following tests would be the best to employ to test differences in this case?

T-test

Margaret, a 29 year old, complains of feeling nervous, moody, and anxious. In addition, she experiences excessive tension and a "rapid heartbeat". To narrow your differential diagnosis, which of the following tests should be ordered first?

TSH

The most sensitive laboratory indicator of overall thyroid function is to evaluate the level of circulating:

TSH

You are treating Henry, a 21-year-old male diagnose with anxiety disorder. The patient complains of experiencing panic attacks while attending his college classes and admits to dreading the approaching date of graduation, saying that he is worried about finding a job. In an effort to promote effective therapeutic communication, which of these statements would be best suited for this patient?

Tell me more about the things that trigger your panic attacks?

hen Maria was 49 years old, she "had a scare" when she discovered a lump in her breast. It was eventually determined to be benign, but, since then, she has been diligent about monitoring herself for other lumps. She has also made sure to receive mammograms annually. Now, at 65, she is asking when these examinations will end. What should you tell her?

Ten years from now

A 28 year old male had a nonindurated ulcer on his prepuce that he claims appeared 3 days ago. He thinks that he thinks he has Chancroid or LGV. While you examine him you explain that which finding would best distinguish chancroid from LGV?

Tender inguinal lymphadenopathy is generally indicative of LGV

A 30-year-old female presents to your office with frequent and debilitating headaches that are particularly intense about her neck. She says that her neck and shoulders feel extremely tight and tense during these headaches. However, she denies any neurologic deficits. Based upon her complaints, what type of headaches is this patient most likely experiencing?

Tension headaches

Which of these represents the standard of care for men with a diagnosis of symptomatic begin prostatic hypertrophy?

Terazosin (Hytrin)

Which of these is not a common indoor trigger for asthma? a. Cockroaches b. Dust mites c. Exercise d. Termites

Termites

Pt has been hospitalized for meningitis, what STD would you most likely suspect the patient to have?

Tertiary Syphilis

David and Sally want to make their home as safe as possible for their toddler. They come to your office asking for advice. When discussing safety issues with them, which of the following is the best advice to give?

Test the temperature of hot water before giving a bath.

An adult patient from Guatemala presents to the clinic with a raccoon bite and an unknown tetanus vaccination history. Which orders should be considered for post-exposure prophylaxis?

Tetanus toxoid, reduced diphtheria toxoid, and acellular pertussis (Tdap), rabies immune globulin (RIG), tetanus immune globulin (TIG), and human diploid cell rabies vaccine (HDCV)

Which of the following types of congenital heart defects is most likely to present with right to-left shunting?

Tetralogy of Fallot

Julie, a 44 year old female of Middle Eastern descent, comes in for routine bloodwork. You note that she has a low hemoglobin level and a mean corpuscular volume of 75 fL/red cell. Her mean corpuscular hemoglobin concentration is 30%. She denies experiencing any changes in geneal well-being and assures you that she has been feeling normal. Which condition does Julie most likely have?

Thalassemia

The practitioner knows that conditions associated with an MCV 124 u3 include all of the following except: A. Alcoholism B. B12 deficiency anemia C. Folate deficiency anemia D. Thalassemia

Thalassemia

For home NP visits billable for Medicare B services, an NP usually does not need a physician's order to bill under the NPs own provider number unless

The NP is proving nursing services exclusively

Who manages healthy people 2020?

The United States Department of Health and Human Services

Damage to cranial nerve IX would most likely result in impairment of:

The gag reflex

Which of these statements is true regarding hyperthyroidism?

The incidence of hyperthyroidism is higher in women than in men

The FNP is planning to conduct a study to examine the efficacy of a high-protein, high-fiber diet in weight reduction of obese patients. What are the independent variable and the dependent variable in the research question?

The independent variable is the high-protein, high-fiber diet and the dependent variable is the patients' weight.

Pheobe, a 23 year old female, has gastroenteritis. While reviewing her medical history and discussing management options, she inquires about treatment for her severe diarrhea. Which of the following findings would contraindicate an anti-motility agent regimen to help manage the patient's diarrhea? a. The patient has a history of alcohol abuse and is a regular tobacco user. b. The patient is prescribed antidepressants to treat an unrelated condition c. The patient is obese and has a lifelong history of diabetes d. The initial workup shows that the patient has a temperature of 38.5 C

The initial workup shows that the patient has a temperature of 38.5 C

Your female sickle cell patient reveals to you that she plans to go on vacation in the next few months. Which of the following environments should you most strongly advise the patient against visiting?

The mountains

What agency enforces the protections safeguarded by the health insurance portability and accountability act

The office for civil rights

Which of the following contraceptive methods should not typically be suggested to woman who weighs more than 90 kg?

The patch

A patient's vision is recorded as 20/40 using the Snellen eye chart. What does this mean?

The patient can read at 20 feet what a person with normal vision can read at 40 feet

A patient presents with a new onset of jaundiced sclera, right upper quadrant tenderness, and anorexia. The nurse practitioner draws labs with the following results: AST = 24 mg/dL; ALT = 13 mg/dL; HbsAg = negative; anti-HBc = negative; anti-HBs = positive. How will the nurse practitioner interpret these data?

The patient has immunity from hepatitis B vaccinations

A patient has been taking albuterol via a metered-dose inhaler for a month and remains in the yellow zone. Which of the following statements is true?

The patient is at 50% to 80% of their personal best

A patient states that she is unable to hear well with her left ear. The Weber test shows lateralization to the right ear. The Rinne test shows Ac > BC with a ratio of 2:1 in both ears., left AC 4 sec. What would be the best interpretation of these results?

The patient may have sensorinuel hearing loss

The TNM classification for a patient reads as follows: T2, N1, M0. All of the following inferences are supported by the patient's TNM classification except: A. The patient's bladder tumor has spread to the muscle of the bladder wall B. The patient's breast tumor has spread to axillary lymph nodes C. The patient's colon tumor has spread to nearby lymph nodes D. The patient's brain tumor has not spread to distant tissue

The patient's brain tumor has not spread to distant tissue. Brain tumors do not have TNM classifications

You have been treating Jason, a 35-year-old male, for HIV infection, and the course of his treatment requires that he be hospitalized for a few days. His medical bill is being paid by his aunt, Sylvia, who wants to know why her nephew was in the hospital, but Jason is not willing to share that information. Of the following, which is the most appropriate course of action for the nurse practitioner to take at this time?

The practitioner should not release the details of Jason's health status to his aunt.

How should a spermicide be applied in conjunction with use of a diaphagrm fro maximum efficacy?

The spermicide should be placed inside the diaphragm, which is the removed at least 6 hours after intercourse

The following statements are true regarding herpes zoster, except: It occurs secondary to reactivation of the varicella-zoster virus The typical lesions are bullae It is usually more severe in immunocompromised individuals Infection of the trigeminal nerve ophthalmic branch can cause corneal blindness

The typical lesions are bullae

Which of the following is the most accurate statement regarding the biological effects during menopause?

The vagina experiences decreased stimulation as the epithelium atrophies

James, a 52 year old male, has just fallen into a diabetic coma. Robert, his domestic partner arrives at the hospital with a living will from James regarding theses particular circumstances. Which component of this will would grant it the strongest standing?

The will names Robert as proxy and grants him authority to do as he sees fit in maintaining James's care.

6-year-old patient who was recently started on methylxanthine for asthma treatment returns to the clinic for a medication follow-up. Which of the following findings is abnormal? Heart rate = 102 beats/minute Respiratory rate = 18 beats/minute Theophylline level = 16 mcg/mL Temperature = 99.9°F

Theophylline level = 16 mcg/mL. A therapeutic level of theophylline for a child is 5 to 15 mcg/ mL. A level of 16 mcg/mL would be concerning because the level could reach toxicity.

The primary function of all of the following pharmacologic agents is to treat inflammation in the lungs, except: Nedocromil sodium inhaler (Tilade) Cromolyn sodium inhaler (Intal) Theophylline oral (Theo-24) Fluticasone inhaler (Flovent)

Theophylline oral (Theo-24 bronchodilator)

Which of these statements is true regarding the use of condoms?

There is a higher reported failure rate of female condoms than male condoms

A nurse learns that a female patient's injuries stem from a case of domestic violence. What legal steps should the NP take in this case?

There is no legal obligation to do anything

Common concerns regarding musculoskeletal changes in the gerontologic population often include all of the following except: A. Thickened skin B. Unstable gait and muscle strength C. Increased percentage of body fat D. Height reduction

Thickened skin (usually thinning)

Management of symptoms in infants with GERD disease is more likely to include which of these in non-server cases?

Thickening of feeding

Which of these characteristics would lead you to believe she has emphysema and not chronic bronchitis?

Thin and wasted habitus

Burtonian lines can best be described as which of the following?

Thin blue-black line on the margin of the gums

Which of these manifestations is least likely to present with the onset of asthma? a. Plugging of airways by thick mucus b. Hypertrophy of the mucus glands c. Thinning of the epithelial basement membrane d. Hypertophy of smooth muscle

Thinning of the epithelial basement membrane (should be thickening)

Morton's neuroma is a benign neuroma that causes a compression neuropathy on an intermetatarsal nerve, most commonly on which two intermetatarsal spaces?

Third or fourth inter metatarsal spaces

Wilfred, a 45 year old male, is diagnosed with tuberculosis. You order a common drug regimen for treating the infectious disease. You may suspect drug resistance if Wilfred's symptoms persist for approximately how long after initiating treatment?

Three months

etonogestrel implant (progesterone only) usually offers continuous birth control for how long?

Three years

What proportion of osteoporotic bone loss stems from hypoestrogenic states?

Three-fourth

The best screening test for detecting and monitoring both hyperthyroidism and hypothyroidism is:

Thyroid stimulating hormone

A patient diagnosed with hyperthyroidism is about to undergo thyroid surgery. Which of the following factors would be most reliable as an indicator of a euthyroid state?

Thyroid stimulating hormone is normal

What is the purpose of case management?

To mobilize, monitor, and control resources used by patients during illness

The Institutional Review Board (IRB) is an independent body established for which purpos

To protect the rights and welfare of human research participants

You have just transferred a patient showing classical findings of a transient ischemic attack to emergency care. You know that, in the course of imaging the condition, your patient will likely first undergo a computed tomography scan. Why would this diagnostic be performed?

To rule out the possibility of hemorrhage

Which of the following would a nurse practitioner be likely to prescribe for a minor bacterial skin infection?

Topical antimicrobial.

A male patient presents with a noticeable yellow tint to his skin. In the workup, which of the following lab values would most likely be of priority to be assessed?

Total bilirubin

The practitioner recognizes that all of these are pulmonary findings in the geriatric patient:

Total lung capacity unchanged Residual volume increases Vital capacity decreases The number of mucus-producing cells increases

With regards to pathology of diabetes, which of the following factors plays a principal role in causing type I DM?

Toxic environmental insult to pancreatic beta cells

If the sponge is left in place for too long, a patient is typically at serious risk for which of the following conditions?

Toxic shock syndrome

A 7-year-old male presents to the clinic with a limp from pain on his right side. You ask how long he has been limping but the boy's mother is not sure. She tells you that she noticed him limping a few days ago, but the boy did not complain of pain until very recently. He is unable to walk long distances. His temperature is 99°F, which the mother explains is due to a recent cold. Upon physical examination, you see no obvious signs, but an internal rotation of the hip causes a spasm. The radiograph appears normal. Which of the following conditions do you suspect is the most likely cause?

Toxic synovitis

A 20-year-old patient is brought to the clinic after crushing his finger in a door. The blow has caused throbbing in the finger, blue and black discoloration of he nail, and bleeding from he nail bed. Which methods of management would be least appropriate at this time? A. Hydrocodone and acetaminophen B. Naproxen C. Trimcinolone hexacetonide D. Ibuprofen

Trimcinolone hexacetonide

Sonia, a 23-year-old female, has lost the ability to move her eyes downward and inward. Which cranial nerve has been affected?

Trochlear CN IV

Cough lasting 3 weeks or longer. Pleuritic chest pain. Hemoptysis with fatigue, weight loss, anorexia, fever/chills, night sweats.

Tuberculosis

Neuraminidase inhibitors such as Oseleamivir can shorten the duration of symptoms of influenza by approximately how many days?

Two days

Which of the following findings is present in 90% of all cases of cholesteatoma?

Tympanic membrane perforation

A female presents to the urgent care unit with complaints of constant nausea, weakness, and fatigue. She reports that she has been feeling extremely thirsty since her symptoms began 2 days ago, and she has been urinating a lot, which she attributes to constantly drinking water. The physical exam reveals the patient has a normal blood pressure and heart rate. A urine test indicates ketones. For which condition should the patient be further evaluated?

Type 1 diabetes

The Union for International Cancer Control uses all of the following parameters to classify tumor development except: A. Size or extent of the primary tumor B. Type of tissue in which the cancer originates C. Amount of spread to regional lymph nodes D. Presence of metastasis

Type of tissue in which the cancer originates

During an office visit, Alex, a 27-year-old male, reports an adverse reaction to mesalamine suppositories and requests an alternative treatment. For which of the following conditions is Alex being treated?

Ulcerative colitis

Which of these statements is false in regard to ulcerative colitis and Crohn's disease? a. Ulcerative colitis and Crohn's disease may occur at any age in both males and females b. Crohn's disease may affect the mouth c. Ulcerative colitis can occur in any part of the gastrointestinal tract. d. Ulcerative colitis and Crohn's disease both have unknown etiologies

Ulcerative colitis can occur in any part of the gastrointestinal tract. False it is typically limited to the large colon

Joan, a 46-year-old woman, comes to your office with worries pertaining to a lump she found in her breast. During examination, it is clear she has a mass, although the type is unclear. What is the best diagnostic test to use at this time to distinguish a liquid-filled cyst from a solid mass in the breast?

Ultrasound

Johnathon, a 44 year old male, is undergoing an evaluation for his complaints of severe heartburn. He tells you that the pain normally occurs after he eats arg meals and is usually alleviated after vomiting. You noted that there is tenderness in the right upper quadrant of the patient's abdomen, and the the patient reports pain upon inspiration as you press down under his right rib cage with your fingers. Based on these findings, which of the following tests would be most effective in confirming the patient's likely condition?

Ultrasound

Which of the following tests is most useful in providing a specific diagnosis of ectopic pregnancy and ruling out other conditions?

Ultrasound

Non-Hodgkin's lymphoma is more likely than Hodgkin's disease to present with which characteristic? A. Unpredictable pattern of spread B. Reed-Sternberg cells (Hodgkin's) C. An unknown etiology D. Cervical adenopathy

Unpredictable pattern of spread

The best test of cure after treating a patient with Helicobacter pylori infection is:

Urea breath test

A 64-year-old male says that he has trouble "getting started" when he wishes to void, and the stream "keeps leaking" when he tries to terminate it. He also wakes frequently with the urge to urinate.He mentions with some embarrassment that he "let go" while celebrateing a victoy for his sports team. Which diagnostic test should you order initially? a. Prostate-specific antigen b. Gram stain c. Urinalysis d. 24-hour urine collection

Urinalysis

Donna, 25, took a pregnancy test a week ago and learned that she is pregnant. Her mother is currently on insulin therapy to control diabetes, and Donna has a body mass index of 31. Since she is in her first trimester, which of the following diagnostic tests should you first order to assess the risk specific to this patient?

Urinalysis

Which assessment finding is expected in a 3-year-old male with penopubic epispadias?

Urine leakage with stress

According to Title II of the Health Insurance Portability and Accountability Act, which of the following is protected by the confidentiality provisions of the Patient Safety Rule?

Use of identifiable information to analyze patient safety events and improve patient safety

All of the following factors are known to contribute to pregnancy-induced hypertension except: a. Diabetes b. multiparity c. maternal age d. Uterine tumor

Uterine tumor

Which of the following statements is typical of the heart murmur with which it is associated?

V/VI, Very loud murmur can be heard off the chest wall with one corner of the stethoscope.

Which of these is often the first sign of type 2 diabetes in females? Select one:

Vaginitis

What is the most useful for asymptomatic viral shedding of herpes simplex virus type 2?

Valacyclovir

For a pediatric patient being treated for seizures, continued use of which drug may induce spontaneous gingival hemorrhage or acute stomatitis?

Valproic acid

A 24-year-old male presents with the following skin rash on the torso. The presentation is most indicative of which condition?

Varicella zoster

All the following tests would be used to diagnose pelvic inflammatory disease except : a. Venereal Disease Research Laboratory test b. ESR test c. C-reactive protein test d. ultrasound

Venereal Disease Research Laboratory test

A 62-year-old male presents to urgent care saying that he has been experiencing trouble maintaining balance and coordinating his movement since early this morning. As he puts it, "every once in a while, I feel lightheaded and dizzy." Suddenly, he appears to lose his balance and states that "it feels as though the room is spinning." You notify the ER physician that you are transferring the patient based on which most likely diagnosis?

Vertebrobasilar transient ischemic attack

Auscultation of normal breath sounds of the chest will reveal:

Vesicular breath sounds in the lower lobe

A patient presents with complaints of a burning sensation in the left eye. A physical examination reveals redness and watery, nonpurulent discharge. The patient presents with no other physical symptoms or sensations. Based on the patient's signs and symptoms, which type of conjunctivitis is the most likely diagnosis?

Viral conjunctivitis

Jordan, 24, presents with complaints of painful, itchy lesions around her labia majora region. She adds, "I think I've had the chills a few times, and I've really felt sluggish for the past few days." Upon physical examination of the affected area, you note small, painful, fluid-filled blisters. Given her presentation and your assessment, which of the following tests should be ordered to confirm the most likely sexually transmitted disease?

Viral culture

Fever, cough, pleurisy, shortness of breath. Scanty sputum production. Myalgias. Breath sounds: decreased breath sounds, rales.

Viral pneumonia influenza, RSV

All of the following deficiencies are common in pediatric patients with celiac disease, except: Folate Ferritin Vitamin D Vitamin B12

Vitamin D

Components of metabolic syndrome

Waist circumference > 40 inches men (102 cm) >35 inches women (89cm) Blood pressure > 135/85 Fasting blood glucose > 100 High-density lipoprotein < 50 women < 40 men Triglyceride level >150

Jerry, age 64, has a history of a herniated disk. He is complaining of pain that radiates from his lower back down through the left side of his calf. He is also experiencing numbness in his back and left foot. He has difficulty dorsiflexing his great toe. which would be best to have Jerry perform for a proper assessment?

Walking on heels to screening for diagnosis of pinched nerve in L4-L5

You have just prescribed your 42-year-old bipolar male patient risperidone. The patient may experience which of the following?

Weight gain

A mother brings her 2-year-old to the clinic with nausea and vomiting, fever, and abdominal pain. The mother noticed blood in the child's diaper this morning. Upon physical examination, the nurse practitioner palpates a nontender abdominal mass extending from the flank toward the midline. Swelling is noted on the side of the mass, and the child has an elevated blood pressure. Which diagnosis is most likely?

Wilm's tumor (Does not cross the midline)

Incidence of viral gastroenteritis in pediatric patients usually peaks during which season?

Winter

Which of the following groups has been recommended to be screened for thyroid disease

Women 50 years or older

After identifying the possibility of Osgood-Schlatter disease during a physical exam, what test would best confirm the diagnosis

X-ray

Calcium supplementation

Yogurt and green leafy vegetables

A patient presents with epigastric pain and intermittent diarrhea that worsens with food. She has a history of multiple ulcers. The nurse practitioner reviews the results of a serum fasting gastrin level and finds there is no inhibition of gastrin levels. Which diagnosis is most likely?

Zollinger-Ellison syndrome

Which patient has the most substantial signs for developing osteoporosis?

a 5'4" asian female weighing 90 lb d.

Which first-line treatment should be administered when treating a patient with chronic obstructive pulmonary disease (COPD) category A?

a SABA alone

The most likely weight of a 1-year-old child whose weight @ birth was 6 pounds would be: a. 19-20 pounds b. 13-14 pounds c. 25-26 pounds d. Impossible to estimate

a. 19-20 pounds (An infant's weight should approximately triple by the age of 1 year.)

Adolescents who engage in risky behavior, such as driving without a seat belt, are displaying: a. A type of egocentrism b. A need for independence c. Role experimentation d. Low self-esteem

a. A type of egocentrism (The belief that one is immune to poor or bad outcomes (e.g., death, disease) is a form of egocentrism known as personal fable in which adolescents believe that the laws of nature do not apply to them.)

You are examining the serologic test results of a patient's hepatitis workup. The practitioner knows that the appearance of ______ and the disappearance of ______ indicate recovery from the hepatitis B virus, no infectivity, and protection form recurrent infection.

a. Anti-HBs; HBsAg

One of the major psychosocial tasks of infancy is: a. Development of secure attachment b. Separation-individuation c. Symbiosis d. Regulation

a. Development of secure attachment (The development of an infant-caregiver bond is key to the prevention of long-term psychological effects associated w/deprivation and/or failure to develop secure, stable bonds.)

The most common temperament profile is: a. Easy b. Difficult c. Slow-to-warm-up d. Intermediate

a. Easy (p. 36 Approximately 40% of children are described as having an easy (rhythmic, approachable, adaptive) temperament.)

With regard to hepatitis and prevention, the practitioner knows that all of the following are true:

a. Hepatitis A immunization requires 2 injections A low- or no-protein diet may be appropriate c. Hepatitis B immune globulin provides passive immunity d. The leading cause of fulminant liver failure is hepatitis

Which of the following findings would be indicative of chronic obstructive pulmonary disease?

a. Hyperinflated lungs b. flattened diaphragm c. Reduced FEV1 d. Increased residual volume

A risk factor that is common to many psychosocial pediatric problems, including failure to thrive, conduct or oppositional disorders, and childhood depression, is: a. Maternal depression or other psychiatric disorder b. Substance abuse c. Prematurity d. History of sexual abuse

a. Maternal depression or other psychiatric disorder (All of these problems have multifactorial etiologies; maternal psychiatric disorders that could affect parenting or development of a secure and stable bond are important factors to consider.)

A preschool boy whose parents have separated & are beginning divorce procedures: a. May think that he caused the divorce by misbehaving b. Should not be told of the impending divorce until the parents are sure of their decision c. Is likely to experience gender identity confusion d. Should be able to make a decision about which parent he prefers living with

a. May think that he caused the divorce by misbehaving (Preschoolers are characterized by egocentrism; they think the world revolves around them & that everything that happens is because they did or did not do something.)

The common practice of using "time-outs" w/young children is a direct application of: a. Operant conditioning b. Classical conditioning c. Separation-individuation d. Maturational reinforcement

a. Operant conditioning (Time-out is a practice of behavioral modification as promulgated by B. F. Skinner's model of operant conditioning w/a negative consequence for an unacceptable behavior.)

The stage of cognitive development that Piaget described as characteristic of the way preschoolers think is the: a. Preoperational stage b. Mental combinations stage c. Tertiary circular function stage d. Sensorimotor stage

a. Preoperational stage (Piaget characterizes preschoolers as preoperational thinkers.)

Which of the following strategies would not be appropriate to include as part of your management of a 9-year-old boy who is obese? a. Referral to nutritionist for weight reduction plan b. Increase physical exercise c. Behavior modification strategies to deal w/stress &/or reinforce treatment plan d. Involve family in management program

a. Referral to nutritionist for weight reduction plan (Treatment of obesity in a 9-year-old requires a multifactorial approach, including "eating healthy", exercise, & needs; not necessarily focus on weight reduction. The goal is to prevent weight gain & maintain weight until linear growth catches up.)

While listening to 2 1⁄2-year-old KL talk, you note that she frequently omits final consonants & her sentences are 2-3 words in length. The appropriate plan of care would be: a. Routine follow-up at the next WCE b. Referring for hearing screen c. Assessing for developmental delays d. Referring to a speech pathologist

a. Routine follow-up at the next WCE (Children aged 2-3 years have several articulation disfluencies, among them is the dropping of final consonants. 2-3 word sentences are normal for the 24-30-month-old child.)

All of the following are standard components in assessing cerebellar function

a. The patient stands with eyes closed, feet together, and arms at the side b. The pateient runs the heel of one foot along the shin of the opposite leg. d. The patient alternately points from his'her nose to the examiner's finger

GPL-1 Liraglutide (Victoza) and Exenatide (Byetta) has a warning associated with

acute pancreatitis and thyroid cancer

Of the following, which condition is most likely to develop as a result of pressure on the cornea in cases of chalazia?

astigmatism

Most cases of epistaxis occur

at Kiesselbach's plexus (Little's area)

Which adolescent would be at greatest risk for developing anorexia nervosa? a. 12 yo female who just had her first period b. 14 yo female gymnast c. 16 yo male runner d. 18 yo female college student

b. 14 yo female gymnast (There is a very high incidence of anorexia nervosa in ballet dancers and gymnasts.)

Most healthy infants are able to reach, grasp, & hold on to a rattle or other small toy by about: a. 2 months b. 6 months c. 8 months d. 10 months

b. 6 months (These are gross developmental norms associated w/the 6 mo child.)

Which of the following situations does not necessarily warrant immediate mental health assessment and/or referral? a. 13 yo girl who has been "down" for the last month w/varied somatic complaints b. 9 yo boy whose parents recently separated & filed for a divorce, who seems to be doing well c. 16 yo girl who has a history of longstanding depression & has started to have "slipping grades" at school d. 15 yo boy who expresses suicidal thoughts

b. 9 boy whose parents recently separated & filed for a divorce, who seems to be doing well (This child may be at risk for a mental health issue, but he appears to be coping well & immediate referral is not indicated.)

The first physical sign indicating the onset of female puberty is: a. Sparsely distributed, fine, pale pubic hairs b. Breast buds c. Menarche d. Peak height velocity

b. Breast buds (The first sign of female puberty is the development of breast buds, closely followed by the development of pubic hair. Peak height velocity & menarche generally occur at age 12 1⁄2 years.)

The principle that growth & development become increasingly integrated is best demonstrated by: a. Gaining head control before raising the chest b. Bringing cup to mouth, tipping, & swallowing c. Rolling over before sitting d. Grasping with fist before using fingers

b. Bringing cup to mouth, tipping, & swallowing (Infants must first develop hand-mouth coordination before incorporating tipping & swallowing, which is a more integrated function. Head control before raising the chest demonstrates the principle of cephalocaudal progression. Options C and D suggest proximal-distal progression.)

Susan, age 35 is interested in taking medication to ease her anxiety, but is very clear about her desire to not take a "tranquilizer". Which of the following medications would be the best choice for Susan? a. Lorazepam b. Buspirone c. Imipramine d. Selegiline

b. Buspirone

While examining 10-year-old RM's teeth, you note that the upper incisors slightly overlap the lower incisors. The second & lower first molars are absent. Your assessment is: a. Malocclusion b. Delayed mandibular dentition c. Normal dentition d. Hyperdontia

b. Delayed mandibular dentition (The mandibular (lower) molars usually erupt between ages 6 & 7. Even allowing for individual variation, this is a considerable delay. Hyperdontia refers to supernumerary teeth.)

A preschool child who says that the sky is blue b/c it is his favorite color is illustrating the concept of: a. Symbolic thinking b. Egocentrism c. Centration d. Imaginary audience

b. Egocentrism (Egocentrism is the hallmark of preschoolers; there is little they think they do not control, from the weather to the color of the sky.)

The mother of 5-year-old DW is concerned that her son often cheats when playing board games with his older sister. What is the most appropriate response to DW's behavior? a. Encourage the parent to use 5-minute time-outs when cheating occurs. b. Explain that DW is developmentally unable to comprehend rigid rules. c. Make sure that DW understands the rules before starting to play the game. d. Explain to DW that cheating is like lying and is not acceptable behavior.

b. Explain that DW is developmentally unable to comprehend rigid rules. (Developmentally, the concept of cheating is not well understood until 7 years of age. The idea of playing fairly to ensure everyone an equal chance occurs with maturity & the ability to differentiate among moral choices.)

In most states, which of the following conditions is not required to be reported by the nurse practitioner to the department of health and Human Services? a. Chlamydia b. Herpes c. Tuberculosis d. HIV

b. Herpes

HO is a 5-year-old Vietnamese child who has fallen off of his growth curve. The best intervention would be to: a. Suggest high-calorie breakfast drinks as supplements b. Incorporate traditional foods into a management plan that will provide increased calories & nutrients c. Educate the family on the need for increased calories & nutrients d. Refer family to a growth clinic for evaluation

b. Incorporate traditional foods into a management plan that will provide increased calories & nutrients (It is important for healthcare professionals to understand the cultural norms & perspectives of others. This often helps in compliance with suggestions for improved health. Asian families, out of respect, often do not ask questions or challenge advice. By understanding their food patterns & incorporating that into a diet plan, the healthcare professional may increase compliance.)

Which behavior would you expect to decrease during the preschool years? a. Rough-and-tumble play b. Instrumental aggression c. Hostile aggression d. Cooperative play

b. Instrumental aggression (Preschoolers strongly defend what is "theirs," be it a toy or a space or a special privilege; this aggression wanes as they begin to understand sharing & appropriate impulse control).

Good communication among families, schools, & primary care providers is an example of which ecological concept? a. Microsystem b. Mesosystem c. Exosystem d. Macrosystem

b. Mesosystem (A mesosystem is the link or relationship between the various settings (microsystems) w/in which a child exists (i.e., home, school, day care, etc.).)

In addition to specific academic skill deficits, learning disabilities are commonly associated with which of the following characteristics? a. Perceptual-motor impairments, normal motor function b. Perceptual-motor impairments, impulsiveness c. Perceptual-motor impairments, Down syndrome d. Lack of impulsiveness, perceptual-motor impairment

b. Perceptual-motor impairments, impulsiveness (Children with LD may demonstrate impulsive behaviors as attention-getting behaviors.)

You would be concerned about the language development of a child who: a. Repeats simple phrases at 32 months b. Stutters when excited or tired at the age of 7 years c. Has a vocabulary of 10 words at 12 months d. Pronounces words that are not understandable at 24 months

b. Stutters when excited or tired at the age of 7 years (Stuttering associated w/fatigue or excitement is not unusual in a preschooler but may indicate a more pervasive problem in a 7-year-old.)

TJ, 13 years old, reluctantly shares w/you that his "chest hurts." On physical examination, you note unilateral breast enlargement, which is tender to palpation. You suspect physiologic gynecomastia. Which Tanner stage would support that diagnosis? a. Tanner stage I b. Tanner stage III c. Tanner stage IV d. Tanner stage V

b. Tanner stage III (Physiologic gynecomastia is a common clinical finding in young adolescent males. It is usually present during Tanner stage III.)

The diagnostic criteria for autism spectrum disorder include: a. A noted lack of back-and-forth conversation b. Tolerance of flexibility with routines c. Fascination with light or movement d. Abnormal eye movements or body language

b. Tolerance of flexibility with routines (Children with ASD do not tolerate alterations to prescribed routines and may benefit from predictable schedules.)

Metformin use increases risk of vitamin

b12

A 25-year old presents with with the following lesions. which of these would be the most effective to treat the patients condition? miconazole clindamycin betamethasone Doxycycline

betamethasone

The pincer grasp is a fine motor skill that involves the ability to pick up a small object such as a raisin or piece of cereal w/the thumb & forefinger & that usually is mastered around: a. 4 months b. 6 months c. 9 months d. 16 months

c. 9 months (This is a developmental norm for a child of 9 months.)

The underlying emotion of an insecurely attached (avoidant) relationship is: a. Ambivalence b. Deprivation c. Anger d. Conditional love

c. Anger (Attachment is the bond that develops throughout the first year of life; underlying anger characterizes avoidance or an insecure bond.)

Which of the following findings would be helpful in distinguishing obesity from large body frame in an adolescent who is concerned w/her weight? a. Tricep skin fold measurement b. Weight-for-height ratio c. Body mass index d. Percent of ideal body weight

c. Body mass index (Body mass index is generally considered the best index for evaluation of weight; it correlates weight w/height. BMI = weight (kg) / [height (m)]2)

Which of the following best describes behavior associated with Piaget's concrete operational phase? a. Learning primarily by trial and error b. Interpreting events in relationship to themselves c. Categorizing information into lower or higher classes d. Drawing logical conclusions from observations

c. Categorizing information into lower or higher classes (Concrete operations occur during the school-age years as children begin to understand the characteristics of things & objects. Classification is a thought process that develops during this time.)

Which of the following issues or concepts is relevant to the school-aged child? a. Operational thinking b. Initiative c. Concrete operations d. Separation-individuation

c. Concrete operations (Concrete operational thinking is key to successful adaptation to school. It involves the concepts of reversibility, conservation, classification, & seriation.)

Which of the following clinical findings would not suggest an eating disorder w/a purging component? a. Sore throat b. Brittle nails c. Diarrhea d. Finger calluses

c. Diarrhea (Eating disorders w/a purging component are characterized by constipation, rather than diarrhea, resulting from chronic laxative use.)

You would expect a school-age child to: a. Grow 1.5 inches per year b. Grow 0.5 inch per year c. Gain about 6 pounds per year d. Gain about 3 pounds per year

c. Gain about 6 pounds per year (The recognized standard of physical growth of school-age children is to gain 5-7 pounds per year & grow about 2.5 inches per year.)

Which of the following substances is associated with pupillary constriction? a. Amphetamines b. LSD c. Heroin d. Nicotine

c. Heroin (Opiates cause constricted pupils. Amphetamines and LSD cause dilated pupils, and nicotine generally does not have any effect on the pupils.)

While taking the history of 6-month-old EM, you learn that she is not sleeping through the night & will not fall back to sleep w/o the parents rocking or feeding her. This is an example of: a. Somnambulism b. Pavor nocturnus c. Learned behavior d. Delayed sleep phase

c. Learned behavior (Sleepwalking (somnambulism) and pavor nocturnus (night terrors) are sleep disturbances that occur in school-age & preschool-age children, respectively. Learned behavior is a result of parents interfering with the child's attempts to return to sleep without stimulation from the parents.)

During a physical examination of 10 1⁄2-year-old Melissa, you note the appearance of breast buds. You tell her that she can expect which of the following in approximately 2 years? a. Growth of pubic hair b. Peak height velocity c. Onset of menses d. Axillary hair

c. Onset of menses (Understanding the sequencing of pubertal development is important, but it must be remembered that individual timing may differ. In the female patient, pubic hair, axillary hair, & the peak height velocity generally occur before menarche.)

The mother of 3-year-old GW reports that he has begun to stutter. Further probing reveals that the stuttering occurs frequently & lasts 1-2 seconds. GW does not seem bothered by the stuttering. The appropriate management would be: a. Referral to a speech pathologist b. Referral for an evaluation for an anxiety disorder c. Reassuring the mother that this is a mild problem d. Demonstrating to GW slow, deep breathing before talking

c. Reassuring the mother that this is a mild problem (This represents a mild stuttering problem but does not warrant immediate referral unless the child or parent is increasingly concerned or if it continues indefinitely.)

Which of the following physical findings in a 2 mo child warrants an immediate referral to a neurologist/neurosurgeon? a. Head circumference growing faster than height & weight b. Unresolved cephalhematoma c. Rigid & immobile sagittal suture d. Snapping sensation when pressure is applied to parietal bone

c. Rigid & immobile sagittal suture (Rigid & immobile sutures indicate premature fusing resulting in craniosynostosis. For proper brain growth, sutures need to approximate each other yet remain mobile.)

Which test would be least beneficial in making assessment of cerebellar function ? a. Finger-to-nose test b. Romberg test c. Roving's test d. Heel-to-shin test

c. Roving's test (appendicitis)

The most common form of child abuse seen in pediatric primary care is: a. Burns b. Fractures c. Soft tissue injuries d. Shaken baby syndrome

c. Soft tissue injuries (Soft tissue injuries, such as bruises, abrasions, & lacerations, are the most common form of abuse, occurring in all age groups.)

Which developmental theory best explains the multifactorial etiology of failure to thrive? a. Organismic-maturational theory b. Social learning theory c. Transactional theory d. Psychoanalytic theory

c. Transactional theory (Transactional theory explains risks and protective factors associated w/resilience & vulnerability. It may explain some of the environmental factors associated w/FTT.)

Which of the following findings would most likely be associated w/asymmetric intrauterine growth retardation? a. Weight, length, & head circumference ranging from 3rd - 5th percentile b. Heavy maternal smoking throughout pregnancy c. Weight at 3rd percentile & length at 25th d. Gestational diabetes

c. Weight at 3rd percentile & length at 25th (The weight & length are at significantly different percentiles; if the IUGR were symmetrical, these would be at the same percentile.)

The results of a patient's cervical cytology test indicate "atypical squamous cell of undetermined significance". Which test would you least likely order as a followup? a. human papillomavirus testing b. a second pap smear c. ultrasound d. colposcopy

c. ultrasound

Respect and spiritual needs are considered components of which principle that the NP must often consider in therapeutic communication with a patient?

cultural competency

Which of the following scenarios is suggestive of a child who may not be ready to enter first grade? An inability to: a. Recognize 6 colors & remember one's phone number b. Accurately use pronouns c. Empathize with others d. Count to 5 & draw a person with 3 parts

d. Count to 5 & draw a person with 3 parts (Children entering first grade should have the requisite skills to master the tasks they will encounter. This includes language, fine & gross motor skills, & personal and social skills. At this age the child should be able to draw a person with at least 6 parts & count to 10 or more.)

Jeffrey, at 8 years of age, has been diagnosed with ADHD & is receiving stimulant medication. Which of the following interventions would be least helpful? a. Monthly height & weight checks b. Small frequent meals & snacks c. High-calorie supplemental drinks d. Elimination of refined sugar from diet

d. Elimination of refined sugar from diet (Stimulant medication may decrease the appetite, so careful monitoring of growth & a nutritional plan that encourages adequate calories are important. There is no sound evidence that sugar or artificial additives play a role in ADHD.)

Which of the following diagnoses is not more common among males? a. ADHD b. Conduct disorders c. Suicide d. FTT

d. FTT (The incidence of failure to thrive has no predominance in males or females.)

Which of the following symptoms is not typical of a child w/ADHD? a. Easily distracted b. Difficulty playing quietly c. Doesn't follow directions d. Frequently angry & resentful

d. Frequently angry & resentful (Anger is not usually a manifestation of ADHD, whereas high distractibility & inability to sit quietly or follow directions are red flags that a child may have ADHD.)

Healthcare organizations must provide language assistance services, including bilingual staff and interpreter services, at no cost to patients with limited English proficiency. c. Healthcare organizations must ensure that language assistance services are provided at all points of contact in a timely manner during all hours of operation. d. Healthcare organizations must always rely on qualified interpreters and bilingual staff, and never rely on the patient's family and friends to provide interpretation services even if requested by the patient or consumer.

d. Healthcare organizations must always rely on qualified interpreters and bilingual staff, and never rely on the patient's family and friends to provide interpretation services even if requested by the patient or consumer.

Which of the following would you least expect to occur during menopause? a. loss of elasticity in the skin b. Changes in skin pigmentation c. Loss of muscle tone d. Increased sebaceous gland activity

d. Increased sebaceous gland activity

In males, Tanner stage III can be distinguished from Tanner stage II by: a. Fine, downy pubic hair at the base of penis b. Adult-like pubic hair not extending to thighs c. Penile growth in width d. Penile growth in length

d. Penile growth in length (Most penile growth in Tanner stage III is in length rather than width because of underdevelopment of the corpora cavernosa. Fine, downy pubic hair appears in stage II, & adult-like appearance occurs in stage IV.)

During 8 mo LB's physical examination, the father boasts that LB is going to be a left-handed batter since he prefers doing everything with his left hand. The appropriate response would be to: a. Ask if others in the family are left handed b. Suggest play activities that require using both hands c. Present toys more often to the right hand d. Perform a careful neurologic examination

d. Perform a careful neurologic examination (Handedness before a 1 yr is cause for concern & may indicate cerebral palsy. A neurologic examination is indicated. The examiner should carefully assess for increase in deep tendon reflexes & tone.)

A differential diagnosis for child abuse would include all of the following except: a. Birth marks b. Unintentional injury c. Inadequate parenting d. Prader-Willi syndrome

d. Prader-Willi syndrome (Prader-Willi is associated w/uncontrolled appetite & obesity; it has no outward sign that could be confused with signs of child abuse.)

Early reflexive responses that are not r/t survival include all but: a. Babinski b. Moro c. Swimming d. Rooting

d. Rooting (Rooting (i.e., moving the head to locate the nipple) is a key survival reflex.)

The privacy Rule of the HIPAA establishes that a patient whose rights are being denied or whose health information is not being protected can file complaints with certain entities. Which of the following is not one of these entities? a. Health insurers b. Healthcare providers c. U.S. government d. State agencies

d. State agencies

Which of the following is the least definitive test in diagnosing PID? a. sexually transmitted disease screening b. Erythrocyte sedimentation rate test c. Ultrasound d. TSH test

d. TSH test

An increase in which of the following behaviors is seen more frequently in late rather than in early adolescence? a. Value conflict with parents b. Focus on physical appearance c. Peer group involvement d. Understanding inner motivations of others

d. Understanding inner motivations of others (Late adolescence is characterized by increased autonomy & beginning to appreciate the complexities & motivations of other people's behaviors.)

Asking the patient with weak quadriceps or pain and numbness in the medial knee to squat and rise is a way to assess nerve damage

disks,

What two antibiotics can be given to a patient with a penicillin allergy that has syphilis?

doxy or erythromycin

Which of the following best describes the appearance of Auspitz sign when psoriasis scales are removed?

droplets of blood

What are common signs and symptoms of an initial outbreak of herpes?

dysuria, fever, malaise

Ephelis

freckle

Melissa, a 25-year old female, has yellow green vaginal discharge. She also has pain and labial swelling. Based on the findings , what is the most likely diagnosis

gonorrhea

The practitioner knows that which agent is the least effective in the treatment of candidiasis balanitis? Griseofulvin Clotrimazole Steroids Miconazole

griseofulvin

Thiazolinediones rosiglitazone (avandia) and Pioglitazone (Actos) are contraindicated in patients with

heart failure Class III or IV

Which of the following is best descriptive of trephination?

heating the nail end of a needle and pushing it into a fingernail

Which of the following poses the greatest risk of ectopic pregnancy among the general population?

history of infertility

Pharmacokinetics is defined as the study of

how the body interacts with drugs

Which antimalarial agent is used in the treatment of Rheumatoid arthritis?

hydroxychloroquine

ST-segment depression in leads I and aVL indicates which type of infarction?

inferior wall

What is first-line treatment for asthma

inhaled corticosteroids

A mother replaced breast milk with whole cow's milk. Which dietary insufficiencies might the infant have

insufficient iron

Sarah, age 24, comes to your practice complaining of sudden pain in her legs and a lowgrade temperature. A physical exam reveals her right leg is hot to the touch and erythematous. Sarah has been bed-ridden for the past few weeks due to a minor surgical operation. Her medical history indicates that she is a smoker and uses an intrauterine device for contraception. Given the most likely condition, which of the following is LEAST likely to be considered a risk factor? a. The patient is female. b. The patient is bed-ridden. c. The patient is a smoker. d. The patient has an intrauterine device.

intrauterine device.

An HIV positive patient develops a low grade fever. During his visit, he complains of fatigue, a reduced desire to eat, and a dry cough. He coughs in front of you, producing sputum that is tinged red. You order a chest c-ray, which reveals a small homogeneous infiltrate in the upper lobes. Given the most likely condition, what combination of drugs would be most effective for treatment?

isoniazid, rifampin, ethambutol, pyrazinamide (9months)

What statement is true regarding licensure

it is acquired through rules and regulations established by governmental body

What is the preferred reliever medication for asthma according to the Global initiative for Asthma (Gina, 2020) treatment guidelines?

low-dose ICS with formoterol (ICS-LABA)

Which of the following characteristics applies to type 2 Diabetes?

major risk factors are heredity and obesity

All of the following treatment options are commonly used to manage vertigo

meclizine (Antivert) promethazine (Phenergan) dimenhydrinate (Dramamine) transdermal scopolamine

Which plan of medicare not only offers coverage for prescriptions but also requires a copay for each as well as a monthly premium

medicare D

nurse case manager utilizes a comprehensive systematic approach to provide quality care by

mobilizing, monitoring, and controlling patient resources during illness while balancing the quality and cost of resources.

How frequently would a typical pediatric patient with mild persistent asthma use a rescue inhaler

more than 2 days a week

Which OTC agent is best known for decreasing renal blood flow, further reducing elimination of many durgs

naproxen

In the course of your research study, you determine the measure of the interdependence of two random variables to be -1. What research term best describes the value?

negative correlation

High alpha feta-protein could indicate

neural tube defects

incidence rate is

number of people contracting a disease during a time period

What is the primary purpose of professional licensure?

o ensure a minimal standard for competency

ou are treating a 67-year-old Latina. Based on her profile and family history, you decide to order a bone density test to check for osteoporosis. Which of the following is not a typical risk factor that would lead you to suspect osteoporosis in this patient?

obesity

Which age groups is the fastest growing segment of the total population

oldest old-85 tears and older

H. pylori quadruple therapy

omeprazole, bismuth subsalicylate, metronidazole, tetracycline

A 62-year-old man with chronic obstructive pulmonary disease (COPD) complains to the nurse practitioner that his prescription for ipratropium bromide (Atrovent) is not working. He reports that he still feels short of breath even after using it four times a day for 3 months. Which of the following actions is the next step for the nurse practitioner?

ontinue ipratropium bromide (Atrovent) and add two inhalations of an albuterol (Ventolin) inhaler QID

The findings of osteophytes, junta-articular sclerosis, and subchondral bone on an x-ray would most strongly indicate that a patient has

osteoarthritis.

Ant bites, elevated nevi, and common skin warts are examples of which type of skin lesion?

papule

Which of the following most closely resembles an instrumental activity of daily living for the geriatric population?

paying bills

Which subfield of pharmacology would be most explain different responses to a drug with different ethnic groups

pharmacogenomics

Work up for cystic fibrosis includes

pilocarpine iontophoresis sweat test pulmonary function test Sputum culture

what keratolytic agents are used to treat condyloma acuminata?

podofilox imiquimod sinecatechins

What level would most indicate nutritional risk

pre albumin 15 (16-35)

During a prenatal exam, your patient says: "I've been getting some really bad headaches lately. They start at the base of my skull, and then spread through my head. She is 36 weeks pregnant, and her history shows that she has been seen for high blood pressure on previous visits. You note that she's gained 5 lb since her last visit the week prior, and that she has edema of the hands and feet. These findings most likely indicate what condition?

preclampsia

A 48 year old male presents with difficulty maintaining a clear visual focus at a near distance. He notes that he has never experienced vision-related conditions until recently. You would know that which of these is the most likely diagnosis?

presbyopia

What two laboratory finding best characterizes the findings of restrict disease in pulmonary function test

primary reduced volumes and reduced expiratory flow rates

Luis, a 42-year old, presents with a painless indurated ulcer on his anus. What stage of syphilis are you most likely in?

primary stage:

Positive pregnancy test and Goodell, Chadwick, and Hegar sign are considered

probable signs of pregnancy

Syncopal episodes are most likely to occur in which trimester?

second trimester

What trimester is the fundus palpable at the umbilicus

second trimester

You are seeing a 24 year old female who states that she has been experiencing severe cramping pain during her last 2 menstrual cycles. Which of the following conditions is the patient experiencing?

secondary dysmenorrhea

Which of these statements istrue regarding aging skin

skin loses sensation and immune response

Which of the following best depicts Hegar's sign?

softening of cervicouterine junction (4-6 weeks)

The patient safety work product often contains

specific findings, procedures, and management methods recorded during a safety event

A febrile 5-year old patient with sickle cell anemia has continued bedwetting. You order a urinalysis. Which of the following findings would you expect

specific gravity of 1.008 (hypo concentrated)

Which finding from a patient's cervical cytology would best confirm condyloma acuminata?

squamous intraepithelial lesions

NovaRing acts by releasing

synthetic estrogen and progestin (etonogestrel)

Immunosenescence refers to

the gradual decline of overall immune system function due to age

Agnosia

the inability to recognize familiar objects.

sarcopenia

the loss of muscle mass, strength, and function that comes with aging

Kaitlin is 18 years old and has recently become sexually active. She requests a contraceptive prescription at your clinic. A physical exam indicates that she is 5'6" and 205 pounds. Based on her height and weight, which would be the least effective form of contraception? the patch (xulane, Ortho Evra) Diaphragm NuvaRing Oral contraceptives

the patch (xulane, Ortho Evra)

AN HIV positive pt has a CD4 count of less than 20% what does this mean?

the risk of HIV infection progressing to AIDs has increased

You are reviewing the chart of a patient with terminally ill bone cancer. A section of the chart pertains to the confidence interval (CI) of nausea for the patient. Which of the following statements is true about CI?

the smaller the confidence interval, the more precise the range of values.

In states that recognize living wills, what condition must be met so that the will is legally revoginzed

the will must be specific enough and address the problem at hand

In comparison to nonselective beta blockers, which of the following statements regarding selective β1 blockers is true?

they cause less bronchoconstriction. Selective β1 blockers such as metoprolol (Lopressor) or atenolol (Tenormin) are indicated in asthmatic patients. Unlike nonselective β-receptor blockers, they do not cause bronchoconstriction.

Acrochordon

tiny benign outgrowths of skin; also called skin tags

What is the standard purpose of a t-test?

to evaluate the differences between two groups

Which of the following treatments is a priority when treating for frostbite?

treat for pain and assess for hypothermia. than soak in water 100 degrees for 15 minutes

Which typically occurs in the first trimester but may also return in the third trimester?

urinary frequency

a male patient who was admitted to the hospital is diagnosed with a highly contagious, viral illness that could be fatal and potentially cause an epidemic. The decision to keep him in isolation and limit the number of health care providers who come into contact with him would most closely demonstrate which ethical principle?

utilitarianism

Which entity is required to comply with HIPAA guidelines?

value-added networks and other health care clearinghouses

What would be best classification for this lesion?

vesicle

What is the definitive test for diagnosing herpes?

viral culture

What is an expected finding in a pediatric patient with gastroenteritis?

watery diarrhea, hyperactive bowel sound, abdominal distention

Complications with the L5-S1 disks, which are assessed by having patient walk on toes.

weakness of plantar flexion of great toe and foot. Pain along buttocks , lateral leg, numbness to lateral aspect of foot and posterior calf. Ankle jerk reflex

An ultrasound examination for fetal survey is typically performed at

weeks 18-20

A positive purified protein derivative test would produce, which lesion

wheal

The practitioner knows that when a smallpox rash appears, the lesion usually turns into small blisters filled with clear fluid:

within 2 days

A human resources (HR) representative has contacted you regarding your 55-year-old patient, Carmen, who has gone through extensive treatment for alcoholism and is now recuperating. The HR representative says that, since the company she works for has paid for her treatment through its health insurance policy, it would like to know whether she has been receptive to the treatments. How do you best respond to the company representative from the choices below?

you say that any information regarding Carmen would need written approval from her.

Jessica, a 34-year-old female, presents with episodic, throbbing headaches that last for multiple hours at a time. She tells you that these headaches are usually accompanied by nausea and vomiting. Given the type of headache the patient is most likely experiencing, all of the following visual disturbances are usually

zigzag of lights, seeing stars, luminous hallucinations


Conjuntos de estudio relacionados

Chapter 4 Review (Data Communications)

View Set

66 management of pt with neuro dysfunction prepU

View Set

Lesson 06: Digestive System Part II

View Set